You are on page 1of 551
USMLE Step 2 CK Qbook Preface ‘Section One: TestTaking and Study Stateges Guide “Chapter One. Inside the USMILE Step 2 O Exam Chapter Two. Stuy Tecriques ‘Chopter Tvee Tackling the Exam ‘Chapter Foue Physical and Mental Preparation Caprese. Surmany Pointers Section wo: Qbook Practice Tests Invernal Medicine Test One sete Test One Answers and Explorations Test Tro . “est Two Answers and Cxplanations Test Thee “Test Tree Anewers and Explanations ‘est Four coe “Test Four Answers end Cxplenations Test Fie soe “Test Fe Answers and Explanations “est Su “est Sic Anewers and Explanations Test Seven Test Seven Answers and Epanaions Test Een “est Eight Answers and Bplanations 7 a 55 35 101 7 133 147 2185 179 195 208 2 22a 259 as Contents Obserics/Gynecalogy Test One “est One Annes ad Explanations TES THO sesso Test To Assners and Explanations Pediaics Tet One Test One Answers one sparations Test Two co Test Tuo Ansuess and Explosions Test Tree . Test Tree Answers and Explanations Paychiaty Test One “Test One Anwar end Explanations Test wo “Test Two Answers end Explorations Surgery Test One “Test One Animes and Emanatons Test Tao Test he Answers and Explanations 289 308 321 335 351 358 3m 301 495, sor 23 335 | SECTION ONE | Test-Taking and Study Strategies Guide Section One: Test-Taking and Study Stategies Guide | 5 Chapter One: Inside the USMLE Step 2 CK Exam ABOUT THE USMLE The United States Medical Licensing Examination (USMLE) consists of three steps designed to assess physicians ability to apply abroad spectrum of knowledge, concepts, and principles and to evaluate the physician's basic patient-centered sil ‘Step 1 (multiple-choice exam)—This exam is designed to test how well the examinee understands and applies concepts integral to the basic and clinical sciences. ‘Step 2 (two separate exams)—The Step 2 Clinical Knowledge (CK) ita multiple-choice ‘exam designed to determine whether the examince posesss the medical knowledge and ‘understanding of clinical science considered essential for the provision of patient care under supervision. The Step 2 Clinical Skil (CS) i a separate, “hands-on” exam that tests the examinee’ clinical and communication skills through hier ability to gather information from standardized patients, perform a physical examination, communicate the findings to the patient, and write a patient note ‘Step 3 (multiple-choice exams)—This exam asses the examinees ability to apply medical [knowledge and the understanding of biomedical and clinical scence essential for the ‘unsupervised practice of medicine, with emphasis on patient management in ambulatory setting. “The results ofthe USMLE are reported to medical licensing authorities inthe United States and its territories (state boars") for use in granting the inital licens to practice medicine. The ‘examination serves to provide a common basis for evaluation ofall candidates for licensure ‘The USMLE i sponsored by the Federation of State Medical Boards (FSMB) and the National ‘Board of Medical Examiners (NBME). DESCRIPTION OF THE STEP 2 CK EXAM USMLE Step 2 CK is nine-hour, computerized examination that serves as the clinical science ‘exam ofthe licensing pathway, Is purpose sto determine whether an examinee possesses the ‘medical knowledge and understanding of clinical science considered exeential fr the provision ‘of patient care under supervision. As the complemen tothe Clinical kills Step 2 CS exam, Step 2.CK consists of multiple-choice questions that will test you on the principles of clinieal science that are important to the practice of medicine in postgraduate traning Chapter One ‘Strategy Tip View the computer tutorial at home 0 the USMLE demo dik and skip the computer {uta on est dy for an ‘adefional 15 minates of break time Seaton One: Test Taking and Study Suatgies Guide What Is Tested on the Step 2 CK Exam Step 2K tert primary cae maine. Subjects covered on the USMLE Step 2 CK inloe bt are oot limited + Internal medicine + Obwttics and gynecology + Preventive medicine and public heath + pacar + Emergency medicine + Dermatology + Neatlogy + Gerais * Sugsy For a dete decipton of examination content ares sete USMLE website, Not that he dines ited inthe orline do at represent anal nce ist. Question ae generally, bt rot ext, faut on the lise ores neon nat al sted tpi aria on Cath examination. Remember la tht etegoiatons and content coverage ate Sbj hans Examination Structure ‘The nine-hour Step 2 CK exam consists of approximately 370 questions, divided into eight “blocks of one hour each: Fach ofthe eight blocks contains appronimately 50 questions from ‘he fll range of topics covered on the exam You wll be able to skip back and forth among test ‘questions, but only within ablock of questions. Once the hori up, you wil be unable o return to that block of questions. When you start the testing session, Your session clock will ein ‘counting down from nine hours, When the nine hours are up, the test wil shutoff completely. Within your nine-hour session ime, you will have to complete eight, one-hour section Step 2 CK at Glance Examination Length: Gg cornu "lod" aninered in one hour tesing session compute tra 15 minutes; breaks: 45 mines, sel scheduled Number of Questions: Apprcimail 370; 1-47 questions per I-Hour black Question Types: To types of muktplechoice question: single best answer and matching set ‘The additonal hour can be used to view the tutorial (up to 15 minutes) and to tke breaks, including lunch, Ifyou skip the tutorial (which you should because you can review it on the USMLE' demo dish), you will havea full hour of break time. ‘Whenever you begin a new block, the section clock will countdown from 60 minutes (the ses sion clock will continue te count down fom the orginal nine hours). When you complete # section (block, youl get information about how many sections you have lef, a well s your remaining session time. Ifyou completa section early, you can add those mites to your rest ‘time. You can never add minutes o your tet ime Scion One: Test aking and Study Stateies Guide |g Inside the USMILE Step 2 CK Exam Question Order “The questions on the Step 2 CK exam are presented ina random, interdscplinary sequence— there are no “conteat-specifi” sections. This makes the exam quite challenging, a you are required to possess the mental agility to jump back and forth among many topic ares. Test, ‘questions focus on topics that are relevant tothe practice of medicine, All the questions are presented in multiple-choice format. Question Length ‘The exon sens on the Step 2 CK exam tend to be considerably longer than those o Step [rast amas inuding ie history an laboratory dt. nerpretason of physic igs td symptoms, nia at personal histories, bls, aging sae, andthe resus of other ingot stoke te equi rere. MULTIPLE-CHOICE QUESTION FORMATS ‘There are two basic question formate on the Step 2 CK eam: one single best answer and extended matching. You will encounter both types throughout the USMLE Step 2 CK Qbook, whichis designed to simulate actual exam questions. Approximately 75-80 percent of the questions on the exam are in the one-bes-answer format. ‘These questions will be the frst one in each black. You wil be presented with a statement or ‘question followed by alist of 3 to 26 options from which you are requied to eelect the one best answer (Five choice is by fr the most common.) “The final questions (20-256) in each block ae in the extended matching set format. Fach of these sets consists of alist of letered choices urvally related to 2 common subject (@g renal tliseases or neurologic disorders). You mast choose the single answer that best corresponds to tach numbered question, There canbe upto 26 answer choices (although 10 most common) ‘There are usualy about two oF thre questions corzsponding to each matching st. A small ‘numberof matching questions in each block wil require you to pick more than ene correct Socton One: Test Taking and Study States Guide Chapter One. One Single Best Answer Questions ‘This isthe traditional, most frequently used multiple-choice format. It consists ofa statement ‘or question followed by 3 t026 options that are in alphabetical o logical order. The response ‘options in this format are etered. Examines ate required to select the best answer to the ques tion. Other options may be partially correc, but there is only one best answer. ‘ATS year-old man presets wth ce complaint fhe episodes offaining (er the past month. He ao relates several episodes of lesde chest pin that ocr with xeon and ese on esing. Furtesmoe he hs noted increasing leg edema and orthopnea. He has no pro aria hisory and is nt on any mecieaions. is lod pressure 130/70 mm He ands pulses min, Ping edema is present in both lower exrenies. The caratd upstokeis dele Cardiac examination irematale fo a nomal Si anda sot, singe S2. harsh te peang sso ejection murmur is ead atthe Fight upper sera border An ECG shows signs of let verricularhypertopty ‘Wc ofthe followings the mast ely agnosis? (A) Aarti insuciency @) Aatic stenosis (Q) Mita insufcency (0) Mita stenosis © Ticuspid insuiciency (answer 8) Section One: Test aking and Study Staregies Cue Inside the USMLE Step 2 CK Exam ‘Oceasionaly single vignette is wsed to introduce two questions. These are known as “linked “questions” An example js shown below. Mems 1-2 ‘A 26yearsld man comes to medial atenton because of sustained hypertension for the past year During the same period, he ao hed parongsma episodes characterized by profuse seating, headache, and a ‘sensation of increased heart ate. At hs time, hs blood pressure 160/95. mig, with pulse of 86min in the supine poston, On standing, is blood presse i 120/70 mm Hg, and hs pulse i 10m. A cpstick test eves fucose in the urine. |. Which ofthe folowing s the most appropiate next step in diagnosis) (8) Urinary 2557 for tee catecholamines and veiymandeli ci (6) Measurement ofevinephrne and norepinephrine in the blood (©. Measurement of esting glucose lvls in plasma or serum ©) Pharmacologic provocative tess (© CT andor MRI tutes ofthe abdomen (Ansver A) 2. Aer perorming the appropiate diagnostic stuties, the patent scheduled for surgery. Which ofthe following i the most appropriate pharmaccterapy? @ Insulin (@) Labetalol © Niedpine (0) Nivoprsside © _Phenonpewamine (Propranolol (Answer B) Strategies for Answering One Single Best Answer Questions, + Read each question carefully Is essential to understand wht is being asked. + Try to generate an answer first and ther look frit in the option lis + Ifyou cat come up with an answer, read each option carefully and eliminate those that are clearly incorrect. + Of the remaining options, select the one thats most correct. + Ifyou are unsure about an answer, guess. Unanswered questions are automaticaly ‘counted as wrong answers. ‘Strategy Tip ‘ou ae nt onking fora good answer, you at lokg for ‘he one best rane, Seaton One: Test Taking and Study Sbatgis Guide (Chapter One Matching Sets ‘This format consists ofa series of questions related toa common topic, All matching sets contain st-speific instructions, alist of letered response options, and a est two question. There will be between 4 and 26 response options. Each sti preceded by a box tat indicates the number of questions in the set asiociated with the response options that follow. You wil select one answer for each question inthe st. Questions wil be presented one at time, with instruction and response options repeated foreach subsequent question. The response opos res -2arethe same You wil be requied to select nears foreach tem inthe set (A) Aortic dissection (B) Nerosertic disease (© Cure Svaus syniome (0) Cryoglbutineicvacus (©) Giant cell arteritis. (F) Henoch-Schonlein purpura (@) Kawesaki syndrome (H) Microscopic polyangitis (0 Palyartests nodosa G) Syphiltic aortits (1) aay tenis (0) Thromboangits obitears (0 Wegener genlomatosis For each patent wth ins and symptoms of perpheral asia ease, select the most ely diagnos 1A 25yearolé women presenis with a 2-month history of recurent transient episodes of visual bising, dizziness, and focal neurologic defi, suchas lib weakness and paresthesias. She aso reports numbress and cles of her fiigers. Physical examination discloses marked weakening of the pulses in the upper extremities. An aoc angiogam reves Proximel narrowing ofthe brachiocephalic, fet common carotid, and lt subclavian arteries, The aortic ach appears normal (Answer K) 2. A Sé-yearold woman presents witha month history of sinsis ae ois media unresponsive to common antbioic and antrinlammatary treaimens. She ecenty developed increasing malise and 2 perssent Cough product of Bod inged spt, Her emperatures 38.2 C (1008 9), blood pressure fs 126/81 mmm Hg, pulse min, and respirations are 1W)min. Laboratory studies show mild anemia and levkoytoss, an erytracte sedimentation rate (ESR) of 87/min, and the presence of| Crulting artnet cytoplasmic atbodies of ANCA type. Urinalysis shows more than 5 red blood celishigh poner id and red cell casts (Answers) ‘Section One: Test aking nd Study Statgies Guide| Inside the USMLE Step 2 CK Exam ‘small narber of matching questions in cach block wll quite you to pick more than one | correct answer. Questions that require more than one answer often refer to patients wo need vitamin supplements or vacines oto stations in which ordering e series of laboratory oF ‘ther diagno tet i retin the workup ofthe patent. ‘The response options forthe items 1-2 are the samme. Each item wil state the ‘number of options to select. Choose exactly this number. (A) Antinuclear antibodies titer | (8) Blood cultures (© Bone abserpiometry t (D) Complete blood count i (&} Radionudide scanning: (P) Rectal examination (©) Rheumatoid facto iter | (9) Serum protein elerophoresis | Urinalysis: i 0) Keay fs For each patient with musculoskeletal pain, select the most appropriate dagostc tudes, |. A Gt-year-od man comes to medical attention because of pain inthe ‘wer back, weight los, and two episodes of bronchopneumenia in the past 6 months. He doesnot smoke. Examination reveals bone tenderness ‘n compression onthe lumbar spinous processes an pallor of skin and sucosee (SELECT 5 STUDIES) (nsiers:D, B Hy) 2. A 2I-yearold man wit a history of sickle ca disease presents wih the acute onset of pain and sweling ofthe right ower temperatures up to 40 C (14 F, and profound male fr 3 days. Bone tendemess on pressutis led onthe dial tia, SELECT 3 STUDIES) ‘Answers: BD.) Strategies for Answering Matching Sets Begin ac st by eating through the option ist to Bcome faa wth he vale response. + Read each question carefully. + Within st some options may be used several ties, whereas oer options may nat ‘be used at all. Respond to each question independently. + For matching ste wth age numbers of options try o generate an answer othe {oeton and thence the anower inthe oon It + yo cat come up wih answer, read ech option ctf an climinete those that ae dey incre. + Ofte remaining options seket the one thats most core + Ifyou a unsure about an answer, guess. Unanevered questions re automaticaly {ound as wrong answers Strategy Tip ‘A ananesed questions are courted as wrong ans, to your advantage to guess when you dnt know iittical Section One: Testking and Stuy Soategies Guide Chapter One UNDERSTANDING STEP 2 CK QUESTION TYPES ‘Within the basic multiple-choice formats presented on the USMLE Step 2 CK exam, there are tinct types of questions If you understand how Step 2 CK questions are written, you wil ind ieasirto understand what the intended answer shouldbe. A good USMLE tem testo se not only you know the requisite knowedge, but if you alo can think with that knowledge the way «physician needs to think. Can you use che information presented to solve the presented clini cal probleme Different types question structures tes diferent types of clinical reasoning sil Stimulus-Response Questions “This yp ofitem ask seightforerd question nde ansvered by imple simul eponie paling Gene ean"Xin the stem you have to pick the right sascaton fo th anon Thistye of question incayto wnt and yo have the owe iy ery snow Noe ‘hat othe roche wed foc mot USMLE questions and that thi spe of Hem nt what you hou be preparing enenx || Stem ‘Answer Figure 1-1.014 Style tem (One ofthe goals of your exam preparation must be to become a good consumer of questions so that you know what is and what is nota good question. Tht way you can train yourself for ‘the particulars of this exam, Double-Jump Questions ‘This question requires you to make nt one, but a series of corect decisions in order to elect the proper anser from the presented alternatives. The qoeston gives yu some information, “X,"and you must reason "IEX is true, then point umber one must be tue If point mimber ‘one struc then pot amber wo must be tre. pot number twos tr, then the answer fot tobe choice D® ‘This ain of esoning is sometimes called internal medicine log because internal medicine physica seem particulary adept a this thought process om | = ve Figure 1-2."Double jump” tem ‘The frustrating part about this type of question is that you can know some, but just not al, of the steps and stl get the question wrong. { Secion One: Testakng and Study Stategies Guide | 4 | Inside the USMLE Step 2 CK Exam Clinical Case Questions “The second common type of Step 2 CK question is the long clinical case. Clinical case questions ‘are more than just alot of words. They are made up of words selected ina particular way, with prticular information at particular places. You have to understand how they ate put together {E you want tobe clear on how to answer the {Long clinical case questions begin as texthook examples of some disease or medical situation Allof the base features youd expec forthe disease will be included inthe descriptive material ‘ofthe question. All he crucial signs and symptoms are there. Next, the question write lays out ‘isto option that include the correct answer, bu alo common diagnoses that are often com fused with the disease that is presented. The goal ofthese dstactorsis to mislead stadents way from the correct answer In clinical case questions, the frst ine i critical. The Fist ne sets up the question and gives key ‘demographic information such as sex, race, socioeconomic status, oF vocation. The lst ine of ‘the queion i also critical becase it gives the actual question to be answered. Everything that ‘ame befor it is merely information. To be correct, your answer mst entrespond to what is asked in the lst ine So now wehave a classic case with asetup, a question, and options that inclode the right answer sand some distractor, Forts item writer, fo conver this question into a good USMLE ques- ‘on, further modification i quired. The fst hing to dois go back into the case and add “ed ‘ervings thats, facts that donot change the answer, butby themselves, in isolation, might tend tolead you to one of the distractors for an answer Asa simple example, a patient is presented, As we read through the case it provides a textbook ‘example of Alrheimer dementia. However, the patent’ history also mentions a history of Ibypertension. Then you are asked for the patient's mos likely diagnosis. I you tke the case as ‘whole, the corect answer of Alzheimer dementia should be obvious. Bu if you fisate on the fact of the hypertension, you might well select the answer of vascular dementia and get the {question wrong, The examiners know what key words you are looking for. A simple way to write 2 more dificult question isto include these “buewords" that led to the wrong answer But we're not done constructing our clinical case question, yet. Remember al the casi signs and symproms we included in our case? Le’ go back and remove some of them from the ques- tion. Not enough to change the answer, but just enotgh to remove some key signs that cvery student knows. For example, if presenting 2 case of schizophrenia, we may not include the ‘pmuptoms of auditory hallucinations The examiners expect you to arrive at the correct diag ‘nosis from the athe information provide. ‘The intent her isnot merely to be dificult but to present the student with real world scenae fos. In real life, patients often lave out important information, or include information that’s cxtrancous Will you beable to figure out what is going on in these less than perfect situations? ‘That's what the clinica cae questions are intended to asses. 12 | Chapter One Note The Sep 2 K exam does at feature negate phrased uesions (ie, quesion wth the words NOT, EXCEPT, of LEAS nthe stem) Section One: Tes Taking and Sty Suategis Guide UNDERSTANDING DISTRACTORS Getting 4 question correct means selecting the best answer, Incorrect options are called “distractora” Their purpose isto distract, tha is, to get you to pick them rather than the best answer. Each distractor wil be selected by some examinees oF it would not be included as an ‘option, Every option fool somebody. Your job sw not be misled, In general, dstractors will seem plausible and few will stand out a¢ obviously incoret. Distractors may be partially right answers but not the best answer. Common misconceptions, incomplete knowiedge, and faulty easoning wll cause you to select distractor The question writers are told that their distactors mus fellow thes ive ules: + They mast be homogeneous. For example, they will be all laboratory tess or all thera pies, nota mix ofthe two. + They mus be incorrect or defintely inferior tothe correct answer Thee wl be enough ofa difference between the right awer and the datactrs to allow a distine- tion. For example, if extimating the percentage ofa population with a disease, the options wil differ by more than five peren They must not contain any hints to the right answer, Disteactors are meant to induce ot to an incorrect choice, not give you ces to the correct one. + They must seem plausible and attractive tothe uninformed. Ifyou are not sucieny familiar with atopic, you may wel find that all of the options look good. + ‘They must be similar to the correct answer in construction and length. Thus, trying to “psy out” the question by looking fr laws in its construction i not a useful strategy. “Answer choices that do not adhere to these rules are not used on the exam. All options ate ‘meant to distract you, but often one ofthe distractor wil soem better than the thers, the s0- ‘alled "preferred distractor” This isthe wrong answer chosen most often. Preferred distractors sre why you can often get yourself down to two choices: the correct answer versus the preferred distractor, Selecting the correct answer isnot a matter of splitting ais. The correct answer wil be clearly correct Ifyou think two answers are so close that you cannot reasonably choose between them then the odds are that nether one is correct you need to look carefully at a different option, Reading the Question ‘The examiners kno thatthe thing you lest want to do ona Tong cna cse question i read the qurston. However question ae constructed that if you do net etd them, youre ely together wrong ‘A sophisticated question writer can take advantage ofthis tendency. Istudens ae likely to stop reading the question part way down, the question writer an inclade key information in the lat ‘part ofthe question, knowing that many students wil never get thee, If you stop reading halfway down through the question, you wil offen mis important, if not crucial, information. So how should students approach long clinical case questionst Start withthe notion that you fre going to read the question. Stop tzying to Sind away around reading the question and jst ‘dit. Time spent reading the question is time well spent. Secon One: Test aking and Study Soategies Guide Inside the USMLE Step 2 Ck Exam 6 you find yourself confused by the question oi you are short on time, ty these strategies Think as You Read ‘Many students read the question and then try to figure out what is going on. By then it's 1 Int. You should be figuring out the question as you read it initially. This means stop at every period and tel yours whats going on. Summarize the question to tht point and decie what Jssues seem mest important. Create an ongoing hypothesis that you can confirm or disconficaa as you move dow through the question Read the Last Line to Set Up Reading the Question Te you do this it this cam be very etective strategy But be cael f you read the question fist and then read through the enti eas, that's great approach, However, many people read the question instead of reading the case and then go skim through the case looking for one crucial piece of information to answer the querson. Inver, they come across aed hein. and they choose the wrong answer, lf You Find Yourself Short on Time, Use the “Rule of Three” Here's the rule: Never piclean answer on the bass of one piece of information, You must have thee pices of supporting information to be sure that you are on target If you do find three Pieces of information tling you the same thing, then you can fel very confident that you now “understand what's going on. THE COMPUTER-BASED TEST Although you'l receiv a tutorial on using the eomputer on test day; sure yuite comfortable ‘with this format by running the tutoril and sample materials onthe USMLE website to become Familiar with it prior to your test date. Experiment with taking the test in blocks and know the screen and all the test features well. A normal laboratory values table including Standard Tnternational conversions, wil be available as an online reference when you take the examina- tion. Other computer interface features inelide clickable icons for marking questions to be reviewed, automated review of marked and incomplete questions clock indicating the time remaining, and help application, During the defined tine to complete the tem in cach block, you may answer the items in any order, review your responses, and change answers, After you exit the block, or when time expires, you can no Tonger review test items or change answers ‘SCORE REPORTING The USMLE program recommends a miniawum passing sore for each Sep. Curtently, the pass ing score as set by tne USMLE program is 182 for Step 2 CK. This corresponds to answering 60 {070 percent ofthe items correctly. The mean sore for first-time examinees from accredited ‘medical schools inthe United States isin the range of 200 220, andthe standard deviation is approximately 20. Your sere rport will include the mean and standard deviation for recent administrations ofthe exam, ‘There will also be a two-digit score on your score report. This two-digit sore is derived from the three-digit score and is used to meet requirements of some medial licensing authorities that the passing score be reported a8 75, ‘The graphical profiles which appear on the back of your individual Step 2 CK sore report are provided as an asessment tol for your benefit and will not be reported or verified to any thi 4 | chapter One iStiical ‘Section One: Test ating and Study Strategies Cuide party. The profiles summarize relative areas of strength and weakness to ad in self-assessment. Percentiles are not provided in connection with USMLE scores. ELIGIBILITY ‘To be cig to take the Step 2 CK, you mut be in one ofthe following categories tthe time of application and on test day: + A medical student officially enrolled in o a graduate of, a US. or Canadian me school program leading to the M.D. degree that is accredited by the Liison ‘Committee on Medical Education (LCME) + A medical student offically enrolled in, ora graduate of a U.S. medal school pros tram, leading tothe D.O. degree tat i accredited by the American Osteopathic ‘Association (AOA) + A medical student ofcilly enrolled in, ora graduate of, a medical school outside the U.S. and Canada an eligible for examination by the Educational Commision for Foreign Medial Graduates for its certificate USMLE EXAM SEQUENCING Students and graduates of LOME- and AOA-accredited medical schools may ake Step 1, Step 2CK, and Step 25 in any sequence. Students and graduates of medial schools ouside the U.S.and Canada rms pass Step ! before registering for Step 2CS, REGISTRATION FOR THE STEP 2 CK EXAM Students and graduates of medical schools in the United States and Canada accredited by the Liaison Committee on Medicel Education or the American Osteopathic Astcition may regis ter forthe Step 2CK throug: NBME Examinee Support Services 3750 Market tret Philadelphia, PA 19108-3190 Website: htpstwwenbme.org “Telephone: (215) 590-9700 Fax (215) 590-9457 E-mail webmailnbmecorg ‘You may use one application to apply foe Step 2 CK, Step 2.CS,and Step 1 a the same time. ‘Students and graduates of medical schools outside the United States and Canada may register for the Step 2 CK through: ECEMG 5624 Market St Philaelpia, PA 19108-2685 Applicaton materials: Website: hips /ww.ecing org ‘Telephone: (215) 386-5900 Pax: (218) 386-9196 E-mail: infosectng org Section One: FstTaing and Study Stategies Cuide Inside the USMLE Step 2.CK Exam Intemational students may also complete a paper application (Form 1068). You can download, Form 106S from the ECEMG website or request a photocopy from ECEMG. ELIGIBILITY PERIODS ‘When applying for Step 2 CK, you mus sect a thre-month period during which you would like to take the exam. A Schedaling Permit with instruction for making an appointment a Prometric Tet Center willbe sent to you after you registration entity processes your applica- tion and determines your digibiliy. The Scheduling Permit species the three-month eighility period during which you mast complete the examination. ‘The Steps to Take Step 2 CK ‘Obtain application mater ram the appropiate registration en, Complete your aplication materia sabi them to the reqszation ety. Receivea Scheduling Peri veriying your eit and authoring you to schedule the Follow the isrucons on your Seedling Permit to schedule your test date at 2 rome conte Cente cation are avaiable at wunspremeticom, Bing your Scheduling Perit and ierifcation as described on yout Sdeding ert tothe Pometic et ener on the scheduled date ad ime to tke the examination. TESTING REGIONS Step 2.CK is administered in the United States and Canada and in more than 50 other counties by Prometrie, You can contact Prometric’ central scheduling office known a8 the “Regional Registration Center” to schedule test dates st www.prometriccom. Use the Prometric Test Center Locitor for up-to-date information on the locations of Prometric Test Centers. ON EXAMINATION DAY rive atthe Prometric Test Center 30 minutes before your scheduled testing ime on your testing day f you arrive more than 30 minutes afer your scheduled testing time, you will not be admited. ‘When you arrive a the tet center, you must present your Scheduling Permit and the required identification described on your Scheduling Permit. Your identification must ‘contain both your signature and photograph. Acceptable forms of identification include the following forms of unexpired identiiation: = passport = drivers icense with photograph rational identity card ~ other form of unexpired, government issued identification IMG issued identification card + Your name a it appears on your Scheduling Permit mst match the name on your forms) of identification exactly. Section One: Testing and Stay Stategies Guide 16 | chapter one + Upon arrival at the text center, you must present the required identiiation, sgn a test ‘ante log, be photographed, and store your personal belongings in your assigned locker + You will ceive a marker and laminated writing surfaces for use during the text. If you fll he laminated writing surface provided, please inform the test center staf. Replae- ‘ment laminated writing surfaces wil be provide. You will not be provided with material {o erase the laminated writing surfaces; do not wipe them clean yourself, Center personnel wil exort you to your assigned testing station and provide brief instructions on use of the computer equipment. You may then tke a bie tutorial prior to starting the frst ws block. Fifteen minvtes is allotted to complet the tutorial and 45 mints for break time. The 45 minutes for breaks canbe divided in any manner, according to your preference. For example, you can tke a short break at you seat ater you complete a bock, or You can take longer break for a meal outside the test center afer you complete a few blocks. ‘Once you begin a block ofthe test, no breaks are provide during the block. Each block lasts approximately 60 minutes. During blocks, the clock continues to run even if you leave the testing room fora personal emergency, Each Block ends when is time expires or when you exit from it + Asyou progress through the blocks ofthe est, you shoul monitor how many blocks ae remaining and how much break time is remaining. Ifyou take too mvch break time and exceed the allocated or accumulated break time, our ime to complete the last lock(s) in the esting session willbe reduced, +The test session ends when you have started and exited all sections or the total time for the est expires. EXAMINEES WITH DISABILITIES Reasonable accommodations will be made for USMLE examinees with disabilities who are ‘covered under the Americans with Disabilities Act (ADA). Ifyou wish to apply for test accom ‘modations, you must send your afcal request and documentation a the same time that you apply forthe exam, Soe the USMLE website for more information. FOR MORE INFORMATION + See the USMLE Bullen of information andthe USMLE website at USMLE.org. ‘Changes can occur afte the Bulletin is released, so monitor the USMLE website for the most curcent formation about the test + Refer tothe website of your registration entity: BCFMG for students and graduates of international medial schools; the NBME for students ad graduates of US. and Canadian medical schools + Run the sample Step 2 CK tet materials and tora provided at the USMLE website, inedical Section One: Test Taking and Study Strategies Guide | 17 Chapter Two: Study Techniques ACTIVE LEARNING Active tse of material increases retention and facilitates recall. Repetition makes memories Each instanceof recall produces a new memory race, inking it to another moment of fife and increasing the chance for recall in the future. Memory is dynamic. Recall actualy changes ‘neuronal structures. Tobe truly usful,a piece of information needs to be triangulated, con- nected to a number af otber concepts or better yt, experiences, For the USMLE Step 2 CK exam, ‘meaning, not mere information, s your gol Rereaing textbooks from cover to cover and underlining —yet again, in a different color— every line on every age fs nt an efcient way to learn You need to focus onthe material most They to be on the examination, Studying that material through active application i the bet Meese ofthe study ‘way to enhance your understanding and retention of the information. materia isthe key to success studin ‘The following study techniques will help you develop better ways to prepare for the exam, but remember, learning for retention and use requires active involement. CHOOSING WHAT TO STUDY (AND WHAT TO IGNORE) | | Hw can you possibly know what is likely to be on your examination? There area sumber of | 1. Talk to medical schoo! faculty. They often have sen past exams or have reviewed an item analysis and can tll you the topics most likely to appear on the examination. They should beable to direct you to what i essential knowledge in ther feld and what is ess imporcant. “Talk to students and colleagues who took the examination in past years. Do they remember some topics being particularly "hard hit” Was there a"avor”to the exam? For example, did there sem to bea lot of Pathology? 4, Students who took the eam i the pst cannot tl you what wil be on the exam that you will tke, but they can direct you to the high-yield content arcis that you must make sure to | master Is highly recommended to talk with people who have taken the exam. However, be | cautious. Candidates typically overestimate how much of their weakest area was on the | eam. They are most likely to recall ested content that they got wrong | 4, Take advantage of the CD-ROM. practice disk sent with your Step 2 CK application | confirmation. These questions can also be downloaded from the USMLP’s website (nwwstlecorg). These practice questions will help indicate dhe content structure ofthe ‘tam. Every year new topics are added to the content oulline, and some older nes are eim= inated, These changes ae likely to indicate new questions that you will not hear about fom any other source, so be sure to check out these soutes. The good news is that this USMLE book contains a general oulline of everyting that can be tested on the exam. The bad news ‘is that as you peruse the outline, you will son realize that there is more content indicated than any one person can possibly master in detail be 18 | chapter two Remember You dort have to study eventing Use a numberof diferent sources of ifaton when deciding wha to tc Remember Yous beable to recgrize concep undersand tei importance, and apply them in presented sitions | Secion One: Test-aking and Study Stategies Guide 5. Certain topics are standards others appear as tends. Recent there have been many’ ques: sions on AIDS, infectious disease, genetic disorders, toxicology, pregnancy and prenatal are, ‘thice, autition, aging, and increasingly, molecular biology: Pay special attention so these topics when you plan your studying. In general, topics begin to appear on the exam two to three yeats afer they reach prominence inthe scientiieay community. Any interesting ‘medical topic that appeared in the literature atleast two years ago isa candidate for inch sion on the exam, 6, Beware ofthe trap of “studying forthe last exam”—exam content dilfers fom yest to yes. ‘This year’s exams will be ditferent from last years. And within any given year, you exam ill be diferent from that of other. Tiss especially true because the exam is computerized, CHOOSING HOW To STUDY ‘Mastering the material you must learn for this exam is a three-stage proces. These stages parallel the functional organization of memory. Basic Terms and Definitions You must learn basi terms and definitions This provides the core vocabulary to understand the content being tested. This stage ia mater of simple recognition and memorization, Terms and 4efntions are learned bythe wse of associational memory. Tiss the level where mnemonics can be useful Central Concepts ‘You must lear central concepts foreach ofthe seven subject ares, This is a mater of being able to explain the meaning of conceps, haw they re used, and how they connect with other concepts Understanding the ero-linkages within subjects and across subjects will serve you ‘well over the course ofthe exam ‘Your basic mental task here i that of reconstructive memory, learning to recall concepts in terms of how things ft togsther. At this sage, the practice of recaling one concep facilitates the recall of other elated ideas. Patterns begin te emerge. Thisis the evel at which diagrams, tables, and pictures can be most helpfl Application You most be able to apply the concepts in presented lca setings an recogni wha om Cepis ars most portant in micas presentations. This the hadet stage of pepaaion, tnd te one that met students neglect Acevng your best pou score deers om know” ingot nly what concepts mean but lo how ie are apple ina gien meal tation, ‘The tsk at this level is that of reasoning, understanding the implications of presented infor ‘ation, and being abe to choose the appropriate action from the avllable options. At this eve, study/discussion groups and doing practice questions can be most help. ‘Your method of study and your study schedule should be arranged to allow you to master each ‘ofthese tages in tur, Asyou make your decisions about how you wll sted, the following sug- sestons may be helpful: Section One: Test Taking and Std Statgies Guide + orga Sup an xu stay shee and adit The big ange wierrpar re cln pnd ws mane cron sonar gogo | SSRESESERR lithe sh tayo wl sug rh ag pe yor ie iniensei kbd purity | «Daiwa your nek ay ling ete doc asinine Uontoarcoueneor wegen tok ep ou pe ilu oa seal pr cr atest tebe + Denote nel ors aay but alec sinew em Tin be tia Rocco ato mes sae ya oy roe Shand irs ns toabjtha mae ncn ce ce ‘Sicpallicmosttineon cab tne out noc + fp ite by ering abe ery nye, witgeaty sé our pgeraton Sep Coch cps enero teiesdoues ths efor s ote ns pap its pe Cher moka eon ma ify hn youd ep oe yorperhe nd wenn «Recent nla css For eample keane and ever ow Sha poem bse penlgy ees wo itt he ein chro peso ch sg tt wou ‘Peston dap ht cv sera Se ey + Kept morn nou oan otra sh wi te 15mm aurea dingy ean or Sng tari ony inal xe oot eat esl he ‘ow cm memory ing ner + Donate boas Ueno ft on infomation 0 + inte mambo fran sue fom nih ou sta. Set oe main tees ea fre cyber bos woe ry tui ccs cape ay point se {uns oon anodes mp HIGHLY EFFECTIVE STUDY METHODS ach person has his or her own preferred way of staying, You will have to decide what will ‘work best for you. High yield study methods all have one feature in common: The more active ‘sou are withthe material the more content you will ultimately retain. Remember, your goal in studying i nt just to putin the mos time, but tobe efliient. Many ofthe hes stodents make use ofthe following techniques ‘Ask Yourself Questions ‘One ofthe best study techniques isto pose questions to yourself es you review material. Perhaps ‘youll want to jot them down on index cards to share with others and to practice later By ask- ing yourself questions, you ae framing the materi challenging yourself to focus on key areas, land preparing for questions you may well see on the examination, Your goal is not to lean ‘ovledge for general use, but tobe able to answer multiple-choice xm questions Study Techniques | ™® Note his eay to dsever your en areas by watching your cain reaction to each be Whatever subject or subject eas you te the eas are probably the onesin wich you are the wees. Remember Do nc entirely eget your sug aeas, but pay ks 2terson to them when studying lang he ater foc teview ‘Study Tip ‘The fey nt how long you spend suing, but geting ‘he most otf the te. 20 | chapter two ‘Study Tip oul see los of tables, srs, and chats on the fram <0 pracice using these oo Study Tip Pasting content in your ‘own words wil make you beter test ake. Remember Ifyou can teach someone a top then you know wel enough forte xa, Section One: Test aking aed Study Strategies Guide This steategy will move you from thinking like a student answering questions to thinking ikea faculty member whois writing questions, ly this proces, you "get into" the head of the ques tion writers and begin to understand what makes a good question and the basi science issues likely to beat the core of presented questions Use Graphs and Charts Many common graphs and carts appear reps onthe exam, Practice reading graphs hart and ables Try abstracting the alent facts quick fom a graph or chart. This tay be txpedied by using a pain set of paper fo cover unneeded information and oft your Stenton on eeted information. Drawing the graph yourelfscems to hp you remember it more than just looking st it lt ple times, Drawing a graph from memory will give you the confidence that you've tuly mas tered the material Again, the more active you are with the material, the more kely you are to ‘oth remember it and understand important nuances. Paraphrase Practice paraphrasing material to highlight important information Paraphrasing means po Cen th neal out rn ling youre what portant a ant 2 ‘ead throgh ind summariing the Ky content in your wn wort. Pretend shat you ate the teacher who i in charge of presenting the content. What would you ‘choose to emphasize? What would you leave out if you were short on time? How would you ‘explain the concept to someone new to the field? Remember, if you can say it in your own ‘words then you realy know it ‘The at of paraphrasing wil allow you to answer questions with extensive information in the stem, such as ease histories, much more efficiently. Many students sy the mos dificult part of the exams is getting through the large volume of reading requited for each question. When you sare paraphrasing, do not treat every piece of information with the same emphasis, but decide ‘what is important and what is not. Developing this skill wil also be helpfl as you progress through your medical cacer. Summary Notes (Creating summary notes ia great sy technique and wil reinforce your parapirsing sk Samsmary ote are yout personal representation of ey pont inthe mater writen na way that makes sense to you, Summary notes should run parallel to yout primary sty mse nd ‘Boul serve to ainott usta aad amplify the hey points of that materi. The physical Seton of simply writing the notes nds to reinforce learning and al long-term retention, Once competed summary noes provide «ready guide for hoe ines when you review te materi Study Groups Sty with ends or colleagues in groups of our or five. The best groups comprise people with 2 range of experts. Ty to form «group where each perons weaknes is complemented by fomeone ees strengths Secon One: Test Taking and Study Steg Guide ‘The goal of these study groups is not to show your colleagues how much you know. Rather, is to find the hoes in your knowledge while you sill have time to correct those gaps. Dont be afraid wo tackle the tough tics Wit the aid of your study group things will make sense much Sooner than they will on your on. Challenge eachother. Pose hypothetical situations and sek agreement as the best answers, Plan Your Study Time For moe cit studying. eo cramming and plant reteviw hey material on a regular bess Repeated exposure to mater ve ie lends to move thorough retention than one mas se concatrted pone Jn your inal eviews remember tha active earings est. This meas siding simply read ingthesame pge of nue ve and over Inn, wat ey words s ment ggereand tel our sefas much you can about the topic ou are studying. Fr example, don spread he cbs eye Rather tl yours about tas if you were expaning to someone ee and then Check your explanation spins! your nots Rereview it alo the ime to hein to mak inks among different sections of our materi ‘What dows your understanding of the physoogy of te tadioscaar ate el you about Common pablogy or pharmacologic inferventton? The threade of common dates wee throug ech of the bu sence ube them together and rove a famework ht ads intention Practice with Test-like Practice Questions Doing practice questions is exential in your preparation for taking a multiple-choice exam. Your goal here sto test yourself and also to learn good question-answering habits As you do questions, examine whether you got them right, but more importantly, look a why you got the «question right or wrong Did you not know the conten? Then that’s your cue that more study {sneded, Did you misread the question? Then evaluate how you mistead it and lean how the question writer wants you to read it Sources of Multiple-Choice Errors Problem Type Source of Errors Format problems Particular question subtypes Anxily problems Questions containing numbers or done ealy in the review session Fatigue probleme {Questions done lt in review session Reading errors ‘More common in Fong questions Directionalityerors _Quetions that ak prediction of consequences Group delineation errors. Questions that present material a unique context ‘When you do your practice questions, do them under a time limit similar tothe actual exam In general, your rule should be one minute per question. Tis is roughly the amount of ime (72 seconds) you will have during the real exam. Get used tothe time constraint. Iris one ofthe unchangeable realities of the USMLE, ‘When doing practice questions, avoid these common mistakes: 1. Do not do questions without preparatory studying, Review material fst until you fe you ‘know it, and then ase questions to text youre I you study by doing questions before you ae ready, you will erode your self-confidence and fail to develop key linkages within the ‘material. ‘Study Techniques | 7? Remember Pan your wok and wok the plan, Remember Your efeny athe ang process deaeases rapidly as the amount of study tne Remember "You do no ave pasa of the subsecins ofthe eam, butyou donot wat o be ‘weak otra rvery wean ‘wo or three oes 22 | chapter two. Stady Tip Learing the arses to hated of questo tat you may pt see an the exam vil ot help you prepare Review materi unt you know hen do questions Remember ‘ein passive earig wl not increase your rteion of information; you rust work withthe meter itiedical Seaton One: Test Tong and Study Stargies Guide 2. Donnot get into the habit of lingering over « question. You do not have this luxury on the real exam, Remember that you have just over one minuce per question. You should spend ahout75 percent of that time reading and analyzing the question stem and the other 25 per Cent selecting an ansver. e honest when you do not knove an answer, move on, and look it up witen you are finshed, 5 So-called “retired questions” and many published questions in review books are not rep- resentative of questions featured om the current USMLE Step 2CK. They ae a reasonsble ‘way to review content, but often do not reflect the length or form ofthe questions on the 4, Donnot do questions individually. Do them in clusters under time pressure, with 5 to 10as a miniorn. This will gt you used to’ moving from question to question. Do not Took up answers after each question, Instead, check yourself after you have done te ful se of ques- 5, When you start working on questions do not panic if you do not get the correct answers. {earn from your mistakes. Questions are apart of the study process they help you sce what ‘dee you need ta learn You will et beter at questions as your studying continaes TRY THIS QUESTION-MASTERING EXERCISE ‘Coverup the options tothe question and read the question tem. Pause teach peviod and pars- ‘phrase what you have read. When you finish reading the question, cove the question and reveal ‘he options Select from the options without looking back at the question ste, With practice, you will get faster, and this strategy wll become a habit. This strategy frees you +o get the information out ofthe question as you read it and does not allow you to waste time by Boing back and re-reading. Remember, you only have tite to read each question once. Learn to make your reading time as efficent as possible Section One: Test-Taking and Study Strategies Guide | 23 Chapter Three: Tackling the Exam STEP 2 CK TEST STRATEGIES ‘Some students have their own personal strategies for dealing with multiple-choice exams. Ifyou hhave & method that you are comfortable with, stick with it Hostever, many people find the Following set of recommendations bell. Our experience shows that your best strategy is (0 Follow this vice: + Start with the beginning ofthe question block and work your way tothe back of it. “This means tat withthe frst question and do each question, in onder, until you come tw the last question. The idea here iso get into a rhythm that will carry you through the exam, Tis rhythm wil help erate what one paychologist calls 3 "low" experience. ‘The low experience isa state of optimal concentration and maximal performance + Do not skip any questions. Ifyou don't know the answer when you come tot you ce not likely to know i ater, Skipping around wastes time and ean end up confusing ‘you.as to where you ae in the exam. Deal with each question as yo come tit ‘Answer itas best you can, and move on to the next question. + Limit your wse of the question-marking feature to no more than two questions in each block. The marking feature lets you return to review and reconsider questions ‘where you would ike more time. However, marking more than a couple of questions makes it hard to keep track of how many you want to revisit You simply may not have time to go back and look at questions you have marked, especially if you mark alo of them. Keep track of the questions you do mark by writing down the question number and any option you hae been considering on your whiteboard, Use the macking fea ture wo keep yourself feom geting bogged down, not to hold yourself back. + Remember that you can only return to questions within the current question block. (Once the time as expired on any given question bloc, you ean no anger access any ofthe questions within it. + Re cautious about changing answers. In geneva, your odds of changing a correct answer toa rang one are so much higher than the everett it is simply not worth ‘the risk Ifyou chenge an answer, you are mos likely making it wrong! You fst impulse i usually the correct one. Stay with it unless some clear insight occurs t you you are not sur, leave your Best answer + you finish a question block with time lft over, go back and “check” only those answers that you have previously marked, Checking almost away leads to changing and tends to reduce your score. Ifyou have « spare moment, make sure that you have fentered an anawer for every question inthe block and then relax. Stake a Break, and ‘mentally prepare yourself forthe next block of questions Focus on the questions to ‘come, not the ones that ae past Test Taking Tip Used comeaty, question making wil you double check quesions whe you have high proba of tinge ansver coer. Misused, making can cause yout nt give 2 question you ful tention heist, fee around 24 | chapter Three Remember Eee te management is key to geting your best core, Remember Keep moving foward thoush the ean. There is ays anther question coming strategy Tip Rea think answer Seaton One: TestTaking and Study Stategies Guide Segment your time sothat you know how much you have left, and so that you do not find yourself rushed atthe end. You have just over one minute per question (72 seconds), Some questions will take more time and some less. Work on your pacing fom the beginning ofthe question block Check your watch every 10 questions 10 rmatke suze you are on the correct pace to finish. you pace yourself throughout the block, you should not be squeezed for time atthe end. + Do not spend alot of time on individual questions. Research has shown that st ents spend the mos time on questions that they get wrong, Ifyou find yourself spending fot of time on a question, this is your indication that you do not know the answer How will ou know you are spending too long on any one question? Ifyou {ind yourself thinking, “Maybe Tm taking oo long on this question” you are. As soon 1s you think this, stop, mark your best gues, and move on tothe nex. + During the breaks between question blocks, try to relax and not think back over the ‘exam. The desc to recall questions is strong but not helpful. Those questions ae in the past you will never se them again. Focus on relaxing and making the most of your break: Remember, you will always tend to remember those questions you got ‘wrong, Thinking back over these questions will hr you self-confidence and make the temainder of the exam more difcn. Be glad one set of questions is behind you, Forget about them, and think about something more pleasant A PROVEN ROUTINE FOR HANDLING EACH QUESTION “The Step 2 CK exam isnot just about regurgitating fics, but about applying those facts in 2 clinical content. Ta handle these types of questions, Your response pattern must nt simply be “read, answer” but instead “read, think, answer” ‘Take a Moment to Think ‘You must tran yourself to take this racial moment of thought on each question. Give youself time to reflect and cal to mind the key facts tat will help you answer the question. Many ques- tions require you to make multiple determinations to arrive ata correct answer. You must take ‘reflective moment on cach question to allow yoursl the time for this cognitive processing Read the question cll mind what you know and then proceed to select an answer. Approach ‘ach question with th assurnption shat you know the answer, and then muster your knowledge toattackit, ‘Make Your Choice in the Time Allotted “The mental task of sclectng an anower on « multiple-choice exam is different than the ask you face when you make decisions in clinical medicine. In the day-to-day practice of medicine, ‘when sclcting a Iaboratory tet, arriving at diagnosis, or finding the best reatment option, ‘you want to carefully consider all ofthe options and be sure about your choice. patient’ ‘health, and perhaps lie ia your bands, Before making choiee, most people report that they ‘want 19 fel 2s lose to one hundred percent certainty a they cn, For the USMLE exams you do not have the time to wait for ths level of certainty. You must ‘rain yourself to make your choice within the time allowed. This often means choosing an answer, even when you are not completly sure. If you wait for that absolute feeling of inner ‘certainty, you wil take to long. Make your choice as soon as you have identified a clear best ‘gues, Then move onto the next question, Remember, theres always another question coming. Secion One: Testaking and Study Strategies Guide ‘THE KAPLAN METHOD: THREE TRIES FOR AN ANSWER ‘You have thre chances to get each question right. you cannot get a clear answer sing these three attempts, you donot know the answer. Mark your favorit letter and move on tothe next ‘question, The hey to this strategy is that you always know what you ar going to-do next. This helps you fein contol and reduces anxiety Step 1: Read the Question. ‘This may seem civil, but studies have shown that most students look atthe answers fis. (Questions cause anxiety and answers provide te solution, so many people go right forthe sl ‘ion. However you cannat pick the correct answer untl you know what you are being asked, Superior students generally spend about 45 seconds reading the question and about 15 0 20 seconds choosing from the given options Poorer students tend to reverse this time allocation, spending ess time on the question and moze on the options, Time reading the question is ime ‘well spent. More time onthe question means moze time spent thinking. + Read the question and pick out key words. Key words are diagnestic information, albnormal values indications of gender of race, and any qualifying terms. + Read caeflly enough 0 that you only have to read the question once. Going back ‘over the question takes time. Read for comprehension the fst time Paraphrasing is the key to effective reading. Because the exam is administered on computer, you can no longer undertne key facts, circle abnormal findings, or make notes in the mains. “To compensite, superior test takers continuously summarize the key information in a brief {ashion while reading the question, This allows you tolook to the options with a sharp focus on the key elements and lessens the need to go back to reread it while examining the choices. Look at the following question to see how this is done A 7o.yearold smoker and alcohol abuser i hospitalized for evaluation of 2 squamous cell carcinoma of the am On his second hospital day, he complains of sweating, tremors, and vague gastrointestinal dress. On Piyscal examination, hes antious and has temperature of 101 Fear rate of 104/mi, BP of 1Sq/!00mm Hg, and a respztoy rate of 22 breaths pet minute. ater that da, he has three generalized tonicconic seizures. Which of the folloning isthe most ely cause of his seizures? (A) Aloholwithdcwal (©) Brain metastasis © Febrile seiawe (©) Mypoaleemia (© Subdurl hematoma Anower: A) ‘The mental paaphasing for this question might be as follows ‘An old man with laryngeal cancer who smokes and drinks too much hat sweat, tremor, a and some GI problems—he has three seizures afew days after admission, What's causing the seizures? Tracking the Exam | 25 Remember You cant give a coer answer toa quesion tat you havent red Scion One: TestTeking and Stody Stateges Guide ‘Chapter Three Paraphrasing isthe mental equivalent of underlining. Ithelps you selec what's important and kecp thot key fats in mind while you are evaluating possible answer. Like mos test-taking Skills it also takes practice. If you practice paraphrasing material when you study, you wil ind that you have developed the basics to answer questions. Paryphrase each time you work with questions to gain skill and confidence. Step 2: The Prediction Pass. ‘After eating the question, stop Before looking atthe option, try to come up with an answer ‘We cll his the preiction pass because you are trying to think ike the question writer and pre- dict the correct answer. By the USMLE’s own rules, questions are written so that any expert in the field can come up withthe correct answer without having any options present. (With the correct answer in mind, you ar les likly tobe led astray by distractor, Remember, they are supposed to distract you and convince you to pick the wrong answer. Ifyou se the answer you thought of, scan the other answers to be sure that itis the best. Then pick it and ‘move on tothe next question. Step 3: The Selection Pass. ‘Air reading the question, look down through al ofthe distracts in order (A,B, CD, ,1,G, tc). you se a correc ase pckit. This the selection pas. Ifthe answer seems cbvious and {iret good, Do not rick yours into thinking the question must be ricky o* more diffu Most answers wil be clearly correct. Ifyou find yourself making up along story why ane option is better than another, stop yourself You are probably wrong, The correct answer should be leary correc, two aswers sem tobe almast the same, then neither one correct. Once you hhave identified what looks tke the best answer choose it and move on to the nest question Step 4: The Final Pass. 1, after reading through the opGons, you ae sil not sure ofthe answer, you have one final try the fina pass. At his tage, rater than trying for a correct answer, you ae eliminating those you ow tobe incorrect. Using this strategy, you can usualy eliminate all bu tw ofthe options. ‘When you have narrowed your choles down to only two options, you have now arrived atthe ‘most crucial moment of the exam. The enerect action at this pont isto pick one of the wo answers and move onto the next question, I'you are really unsure ofthe correct anser, which ‘one you pick does not mate, With two options to choose from, you have 250 percent chance ‘of getting the question correct, rather than the 20 percent chance you started with ‘Make a choice. Many people waste time at this point by not choosing. Some people, when they have eliminated all but two answers, go back and rereed the question in hopes of finding some information that wil help them choose between the two options, Time spent talking with sti- dents and watching their thought processes during the exam suggests tht this the wrong ‘strategy. When students read a question a this point, they tend to add tito pick out single ‘atures that help them fel better about choosing one of the answers. Howeve,it does not help them pick the right answer. BY adding assumptions tothe question, students may feel more ‘confident, but they ee rally mentally rewriting the question tobe one that they fel more com- fortable answering. The answer they picks then the right answer tothe question that they env sion, but not fr the actual question presente. fae these three passes (1, Predition Pas, 2. Selection Pas, 3. Final Pass) you sil ate not sure ‘ofthe answer, your best option isto guess. At this point, mark any eter and move on to the ‘next question. No answer counts the same asa wrong anne. ‘Section One: TestTaking and Study Stategies Guide {Questions with large numberof options should be handle the same way asl ne-best-answer -Sestons, The only ciference stat you should not do a Final Pass because it would et up too ‘uc time. That means these questions usually take less ise to handle if dane correct emember: the key to doing well on this exam ito train yourself make choices. fyou donot ‘snow an answer, admit it make your best guess, and move on to the next question. TIPS FOR THE WEEK BEFORE THE EXAM Daring the last fev days before the exam, you should be tapering off your studying and geting v0 mental and physieal sbape - This is not the time for ramming in new material, but time to organize and integrate what you already now. Work on making what you know more accesible, Review keywords, phrases, and concepts. Look over your summary notes one more time. This i the time to dill yourself on esental information The key i to practice recall, not simply read over the material again. What you need to know is probably already in your head, Your tsk now isto train youself ro access the information when you need it. Doing practice questions ia good way to reinforce your recall skills. Remember, practice questions fre fen harder than the questions onthe ral exam, so do not panic yous do not gt them ‘ight Use them to clarify your understanding af ke details, 53. Have an honest conversation with yourself and decide what you do not know. No one can ‘know everything that i asked on this exam. Be honest with yourself about what you do and a not know Krawing that you do not knove somthing gives you more ofa sense of control fn the exam and makes you les likely to panic when you encounter the material and/or ‘este time on questions you ate not likly to get cornee. When you come to question that you know that you do not know, simply mark your favorite letter and move on! 4. Get youself onto the right time schedule. Wake up every day at the same time you will need to on the day ofthe exam. This will get your circadian Fhytam coordinated with the exam schedile. Do not nap between #00 41% and 5:00 Pm. Otherwise, you will accustom your body to shutting dowa during critical exam hours, 5. You shouldbe getting sufficient amount of slep, For most people that means atleast 6 10 7 hours night Sleep isan essential time for your brain fo consolidate what you have learned, You need sleeps it makes you a more efficient leaner when you are awake. 6. Take some time each day to relax. Have a good meal Take a walkin the fresh ai. Find time for exercise. The change of pace will refresh you and the physical activity will help you relax and sleep at night Consider the impact of your personal relationships on your preparation, Family responsi- Lilies and obligations can be very distracting. The week before the exam you should avoid family confrontations and any stressful relationships. Your focus should be on the exam and nothing ele. The other parte of your life can wait, 1. you vent done so already, visit the Prometric Center whece you willbe taking the exam. lt willbe indicated on your exam entry ticket. This will ensure you knw how to get thece and how much time you shoul allow forthe commute, You can see where you should park, and see what the computer set-up is ike 8. Ifyou have not yet done so, review the tuloral on the official USMLE CD-ROM, Recome {amiliae with the interface, the location of Key information on the screen, and how to navi- {ate between screens f you walk ito the exam familiar with the exam, you wil not have to _use any of your valuable break time to do this on test day. Tackling the Exam | 27 Remember Use te time before the eam for even, ot for tyng to Jean new material Section One: Test aking and Study Stotegies Guide {Chapter Three ‘THE DAY BEFORE THE EXAM 1. Take the ayo om all tadying. Theis your day to relax and gather your stengh before themain eet, Gt ou of bd at the same ne you wil have to et up he net day and then trea! this aya a vcation day to reward youre fo all your hard work. If you mst ty, limit yourse to reviewing your own notes and fasard. 2. Have some fan. Go fora wal. Listen to your fvorite muri. Go sce» goo comedy or an action movie that will llw cathartic reese. Go shopping. Spend time with sigiicint other Do whatever you ike You have worked bard and deserve it 2. Make se tat you hav checked ou the bass forthe exam: + Have you worked through the USMLE CD-ROM roi? + Do you know where the exam is bing given and how to ge thee? + Do ou he atemative transportation if for example, your ar dost not art + Do you trast your alarm clock to wake you up in time? If not, make arrangements with frends a a backup. You want tobe sure to wakeup ested, reeeshed, ad on tine 4 Lay out what yu!ll need for your exis before you go t lep. This ncades your photo ‘denutcation,sehedlig permit and confirmation number aswell x any persona ems Tike eyes Whyte a ity dot forget to pack hncht 5 Call your frends and casmates and make some plas to celebrate after the eam is over. “ui need to blow off some seem anyhow, and aking with clleagus wil remind you tat yo are notin this by youre. 6 Bese to do some physi activity Just taking a walk for an hour will hp relax you. 1. Geta good nights sep. To lp you sep consider aot bath or wm milk, Avoid taking sleping medication a ay lve you grog nthe morning ‘THE DAY OF THE EXAM “This isnot the most important day of your carer, but just another hurdle on your way t0 ‘becoming a icesed physician. Keep it in perspective. Treat the exam lke what i i a routine ‘mechanical exerci. You ate not doctor for this da, but an assembly line worker. Rather than ‘making cats or toasters, you aze answering questions. Deal with each question as you come to ty make your choice, and then move on. "No mater how well prepare you are forthe USMLE Step 2CK exam, you will gt many questions ‘wrong. This satan exam were you should expect to know every answer. Remember, 70 percent ‘correct puts yo well over the mean! Knowing this, your test-taking strategy should be some- ‘what diferent than it may be when you take other exams. ‘Doing wll on this exam means spending your time on those questions you are most likely to get correct, and not wasting time on questions you ate likly to get wrong. Two minutes spent ‘om 2 question that you get wrong is two minutes wasted. Approach every question assuming ‘that you will be able to answer it correedy Ifyou discover that you can not arrive at a clear answer, admit i, make «choice, and move on to the nest question, The core idea isa simple ‘one You know you will gt some questions wrong. Therefore, admit which ones they are and spend your time on those questions for which your probability ofa orzect answer is higher Section One: Fest-aking and Stay Stotegies Guide TEST-TAKING TIPS “Try to arcive 30 minutes carly to the Prometric Cente 0 yu are aot rushed and have time to get ‘organized. You wil be given alocker to stove your personal items and then assigned «computer station, Remember that you have ¢ total of & hours to complete 350 questions in 50-question Docks You will have 1 hour for each block of questions, and total of I ou tobe used through: ut the day for breaks and lunch You will be equied to sign out when taking breaks, To cope with fatigue, you wil need to schedule breaks. Our recommended schedule forthe Question Block Break Time at End of Block Block 1 No break Block 2 S:minute break Block 3 S5-minute break Block ¢ 30-minute lanch break Block 5 No break Block 6 10-minute break Block 7 Done! This allows you 10 minutes extra to we a8 needed. You should also be aware that if you leave the exam room during bloc, it willbe marked as an ereglarity in your testing session. SO consider after each lock whether you want wo take a bathroom bresk during your break time. AFTER THE EXAM Take the day off and have some fun! You deer it! Forget about the exam and pay attention to ‘the rest of your lf that you have put on hold while preparing, You will have plenty of time to ‘hinleabout the exam agnin when your scores come in, For now, however, take a holiday! Tackling tre Exam | 79 iiédical Section One: Test-Taking and Study Strategies Guide | 31 Chapter Four: Physical and Mental Preparation PHYSICAL AND MENTAL PREPARATION FOR THE USMLE STEP 2 CK EXAM The USMLE is big event. Like an athlete preparing fora bg race, you want to approach flly prepared-—both physically and mentally. The folowing information consists of traning tips to ‘help you prepare for your test-taking marathon. PHYSICAL PREPARATION Allay examinations are physically stresful What can you do to help decreas this tes? Get in training. At east one month before the exam, begin the plan described below. 1 Get enough rest Alough you mayb mpd ti atintthe night ad ight wp theme fore he eaminaton hissy canbe and rt yo The rin eed rex By shugo cua ipa foster te anation of informatio to lng ter ‘memory and allow time for making connections that help the integration, retention, and TeStTaking Tip real frman. Inthe week brent, you hoe genet es 612 bos of sleep a night, yen emia ‘Avoid heavy breakfasts to 2. Eariht. Whicpreing or texas hnve good oi mls Poin cabhydaey and SOM be gl‘ some fats ae all important. However, donot overeat or eat very late, because this wll impai at many people experience ‘your sleep. Breakfast on the big day should be light; complex carbohydrates like fruit and at a cuca time inthe fst or ‘cereal work well. Avoid anything greasy or unusual. Likewise, keep sugar or simple carbohy- second question block. drate equivalents, such as candy, to a minimum at this time, Hard candy, not chocolate, can_ Sousa imep dung camiation Iu normal ink cof in the mar Inparink cote te mraing of am Tou dont wan fo Wg ace witha Indice, at ight anche alas simple snd and wouscohole ds avoid dterfunch eyes + Some stents po relucion + Schoulean ext mass prt of our td ie. These mes srw as res hat gheyous chance recharge and be more effin in your sud 2. Getsome exci, The exam eis out ein god phys codon, since you wll besiting for long pera tin and want avoid Tate Tel inne bck ortega As pu of our preprton, strane programa sk wth no Mat ter how ifialYourteginen shoul ince srching and sericea such sk waking or be ing The ight Bee the exam, cree, ona more mart sae ‘ould conde. ees te psa bene ere can ele elt deca our ney at vitamin B complex, especially B, can help with stress 2 Sedion One: Testking and Stuy Stotegies Guide Chapter Four MENTAL/EMOTIONAL PREPARATION People ilfer tremendously in their reaction to test stations. Some appeac to sal through— confident and caln-—while others experience mental and physiologic symptoms of test anxiety such as insomnia, nausea, muscle twitching, or increasing inability to concentrate. If you have terious concerns about tect anxiety, don't wait until the day before to seck help, Deal with the problem so anxiety doesn't increase as exam day approaches and your options for dealing with st decrease to zero. This i stuational problem, and effective treatment i available Here are some suggestions for how to deal with test anxiety and some exercises that may help you to cope. 1, Avoid negative thoughts and feelings. Negative seif talk such as, “Ther no way T can ‘ass this eran” can be distracting and produce more anxiety about the tes, Focusing on voiding failure ie a recipe for flue, Focus instead on echieving succes 2. Make anxiety your friend. A degre of anxiety on the day of the examination is not only ‘natura, but also beneficial. How often have you heard of people responding well beyond their dell bites when pushed tothe limits psychologically? However, there i no need for you to push yourself to the limits, you simply ned to bea litle tense and anticipatory on ‘exam day. Incapacitating anxiety will destroy your ability to pass. We have heard many individuals say," knew everything, T even taught my peers, bur T didn't pass. T don't understand ie” Ansety may do this o you! Seak help if necessary. If you find that your studying i being overwhelmed by your anxi~ «sy the most usefl thing you can do may be to seek counseling. Talking o someone sbout your anxiety it noe a waste of time if your study time i unproductive due to that anxiety. Get help tobe more productive 4, Be aware of obsessive thoughts Ifyou find yourself focusing on your inability to answer ‘questions, the volume of material or doubis about your ability to pass the exam, try giving Yourself something els to think about. Keeping your mind focused on a smal task at hand, ‘uch a6 the information you are currently studying wll help you avoid letting obsessive thoughts distract you. 5. Makea study schedule. Purchase a calendar and write out which subjets you will sudy and atime frame foreach. Leave room for breaks and "fe time” This helps you gain con tuoloveryourlifeand your studying. Fllowinga schedule givesyou structure, which makes you mor efficent and reduces sess. 6, Improve your attention control. Awareness ike «searchlight. Whatever you direct your attention to is pretty clear, but other thins and events tend to fade int the periphery. Try directing your attention separately to sights, sounds, the felings in your hands, fet, et. ‘Your awareness can sift very quickly from one foes to anothers however, you can only be fully aware of whatever i in your realm of focus at one patiulse moment. Use this fatto redirect your wandering thoughts and feelings by focusing your attention. In time this wil Allow you to quickly de-emphasize extraneous thoughts by redirecting them to simple bod- Ay functions. Keepin mind how attention works: + You can attend to only one thing a time, + Astention i voluntary and immediate, focusing on the now. The future has no role to ply. You can monitor your own attention, + Attention can be redirected + Focusing on other things that are more relevant can reditectielevant attention, + You cannot pay attention continuously to one thing without breaks. Sedion One: Testaking and Sty Suess Guide Physical and Mental Preparation | ‘Use visualization, Spend a few moments thinking back to crisis situation that you han- Aled beautifully. Perhaps it was @ medical emergency ora family quate where you inte ‘rened and helped salve the issue. Iecan be any typeof exss in which you took charge suc- essfily, Recall how strong and in control you ft, how effectively you controlled a bad Scene, even what the setting wa ik. Reflect on the event to real it in as much detail as you can. Each day while you ae studying, spend 5 to 10 minutes e-visiting tis past event Until you are able to bring it back n your mind with great clarity and deta Now when you fool an anxious feling welling up, take a mental time out and re-visit this scene. When you do, all the emotions of that day wil also retuen and replace those anxious feelings. ‘Use an Affirmation Card. Take 30 minutes or soto write serie of statements on an index card. The statements should describe what you believe ar your greatest personal strengths, Character traits and talents. You might sate tht you ae 2 deeply empathetic pesoo, or that you are very good at solving problems. It doesnt mater what you write down a ong 4 you believe each statement is true an that you are proud ofthat sil personality aspect, for talent. Once you have created the card, keep it with you as you study and practice with test questions. When anxious feclings begin to intrude, takeout the card and read through it slow, realizing the truth of what you have written. With practice, this proces will help focus you and lessen negative though ‘Take a mental “time out” If anxious or angry thoughts interfere while you are practicing wth questions, ry this exercise. Close your eyes, take some slow, deep breaths, and lex, ‘hen eelax first the muscles in your nec, then your shoulders, then your lower timbs. Use a 1-2-3 count for each inhaled and exhale breath to keep your breathing dep and even. The wile proces will any take «few minute and will help you reduce the physical symptoms ‘of anxiety and allow you to return tothe test witha calm, focused state of mind. ‘Learn and practice deep breathing techniques. It can be difficult wo focus your tention nen you are anxious, Dut deep breathing ean help. With enough practice during stressful ‘vents this technique will become second nature during the exam. ‘Use “time out" during the exam. If necesary to break the ensity loop daring the exam, bck away from the mouse and keyboard and take afew deep breath. Thirty seconds of rest wil seem like 30 minutes in the middle of the ear. Ty timing yourself and se how long. itfels. By taking very bite time away you gt alot of mental rest. When the exam becomes too much, the best strategy isnot to push yourel to concentrate harder, but 10 Back off and rest for afew moments, You wil find that when you return tothe exam, your anxiety vill be reduce, andthe questions will make more sense. 3 | Section One: Test-Taking and Study Strategies Guide | 35 Chapter Five: Summary Pointers SUMMARY POINTERS the USMLE Step 2 CK isan all-day exam, The day can seem long and tring. You need to have : clear gol in mind and a lear plan for reaching that goal. You need tobe in good mental and ‘physical shape forthe exam. 1, Not every question counts (although yo can't tell which ones do and which ones do not). 2. You need to be prepared inal f the seven core subjects: Anatomy, Behavioral Sciences, Biochemistry, Microbiology/immunology, Pathology, Pharmacology, and Physiology. A ‘question in one subject counts just as much 3 one in another. 43. Booonte falar with the basic single best answer question types: positively worded, tw: ‘Hep bait and switch, trues, clinical case, and conjunction questions 4, You must beable to answer questions regarding material presented as ithe basic know!- edge or as applied to clinical tasks, Increasingly, for this exam, application of knowledge i the key. 5. Spend some time becoming familiar with what slikly tobe tested, but be careful ofthe mistake of studying for lst year’s exam” 6, Me organized and plan your study time. Kaplan as 2 variety of resources ranging from home study material olive lecture review courses that you may fn useful i this regard, Make a study plan and stick oi | 2. strongly consider forming a study group and helping exch ether review information. Presenting to each ater is one ofthe best ways to lar to use the material you have studied ‘only ater mastering the underlying material Always do questions in | | 8. Practice question clusters and within time limit and physically 10. Dac the exam, read each question and answer them in order. Never change an answer. Segaent your Hime to be sre you are no squeezed atthe end of the exam. 1 Do not finger over questions you do nat knove Move on and use the time to answer ques | tions you do know i |. Taper off your preparation before the eam to give youre «chance to rest up mentally | . i | 12, You have three chances to get every question right: cll, seloction, and final passes. Ifyou can't gota good answer afer these three tres, guess and move on. 13, Consider using any ofthe time-tested behavioral strategies, xpecaly taking atime out” for coping with anxiety and distracting negative thoughts during the exam. 1M, Take care of yours Suffering doesat help anyone 36 Secon One: Tsang and Study Svatogis Guide Chapter Five ‘THE USMILE STEP 2 CK EXAMINATION: DOS AND DON'TS Dos DO remember the content of the computer-based exam (CBT) willbe similar to previous ‘exams, 2s will the amount of time per question {jas over one minute), DO tae advantage of the CD-ROM computer-based testing practice disk that will be sent with your Step 2 CK application confirmation. DO be organized, Setup an organized study schedule and adhere tt DObe sure to make up questions while you study (use index cards) and forma study group to ‘eviw important content 'DO use graphs and chart in texts to speed up yous comprehension ofthe material. A picture isworth a thousand words, and the exam will st you using similar images DO practioe questions under atime limit similar tthe actual exam. In general, your rule should be one minute per question. DO understand that there is material tested that you will ether be unfamiliar with or never rea: ly flly understood. Don't pani! Guess and move on, DO know that you are not required to pass each subsection (physio, biochem, etc.) ofthe exam separately, but only to answer enough questions correctly to attain an overall passing sore. ‘DO understand that you will only need to get about 70 percent ofthe questions right to receive passing score. DO realize you probably sce more pictures than in past exams, They are only intended to est you on coe base scence content. DO ow that not al ofthe questions on the exam will count. Anywhere from 30 0.35 ques= tions on the exam are included so they can be pretest and evaluated for ase in later exams, fo ‘dition, as many as 10 to 15 other questions may be eliminated from the scored pool after the exam results are reviewed, DO mate sure your sclcted answer matches al the information presented inthe question. All answers are somewhat likely; you want to pick the one mos likey DO be prepared forrwo-step questions, where you will ned to make two correct decisions to arrive atthe correct answer (eg. come up with a diagnosis, then decide what the treatment shouldbe). DO remember when you bave the question down to two choices, you need to pick one and ‘move on. Lingering over the question tends to res in making the wrong choice Section One: Testking and Study Suategies Guide DON'Ts DON'T just proctice questions without preparatory studying. Review material fst until you feel you know it and then use questions to test youre DON'T do practice questions individually. Do them in clusters with five to ten as 2 minimum to get yourself used to moving from question to question, and DON’T look up answers ater cach question DON’T getinto the habit of lingering over a question or thinking about foram extended peri ‘od of time. You DON'T have this luxury on the rea exam. DON'T just memorie material! Learn and understand ow to apply the content in presented soe natis. ery ite ofthe exam wil test rote memory for basic facts an that’ not enough to pas. DON'T reread your textbooks from cover to cover You need to focus on the material most ike Iytobe on the examination, Talk to others who have taken the exam to see how they prepare. Talk to faculty, Download the Step 2 CK Content Description and Sample Test Materials document. DON'T stsume commercial review hocks and practice exams are current. A good rule of, ‘thumb is that whatever appears in most ofthe review books is probably important, and what- ‘ever appears in jst one book is mos likely peripheral DON’T be afraid to face your woak or last-iked ares. Take pretest and/or diagnostic exams tohelp you narrow down strengths and weaknesses, Begin your study plan with your weak areas, land plan to cover those at leat twice before the exam, DON'T entirely neglect your strong, ‘eas, but eave them for atime closer tothe examination, DON'T expect traditional exam “ticks” shortcuts, or buzz words to point you tothe correct answer The USMLE bas put reat effort into climinating these cues from the exam. DON'T get caught by distracts. They may be patially right answers, but not the best answer. ‘Common misconceptions, incomplete knowledge, and faulty reasoning will ause you to select a distractor DON'T substitute reading the last Hine ofthe question for reading the whole question, This can ‘cause you to miss important information of point you to dstactors intended to confuse the DON'T skip any questions. F you dont know t when you come tit, you are no key to know it later Deal with ach question as you come ot, answer it asbest you can,and move on to the ext question, DON'T change an answer. Your odds of changing acorect answer to wrong.one are so much higher than the reverse that it is simply not wort the risk. ‘A FINAL COMMENT This is not atest of your intelligence or even of how good a doctor you willbe. Thisisa test of your capacity to identify and apply core principles within the constains ofthe multiple-choice format. Planning, preparstion, practice perseverance, and patience wil lead you to your best score. Good lc and remember, were here to help! Summary Pointers a | SECTION TWO | Qbook Practice Tests Section Two: Qbook Practice Tests | 41 Internal Medicine: Test One |A 23-year-old woman comes to the physician for 3 health maintenance examination, She enjoys good Dhealth and exercises regularly, Her height is 172m (68. in) and weight is 66 kg (145 Ib) Her blood pressute is 120/80 mm Hg, puke is 78'min, and respirations are 12/min. Physical examination is unremarkable except for heart auscultation, which reveals an isolated tnidsystolic dick. Which of the following is the most ‘common cause ofthis auscaltatry finding? (A) Bicuspid sortie valve (B) Congenital pulmonary stenosis (©) Miteal valve prolapse (D) Ruptared papillary muscle (2) Trcuspid regurgitation A 50-year-old man comes to the physician because of singival bleeding, epistacis, and fever for 2 days. He appears acutely i His temperature is 39 C (102 P), blood pressure is 120/70 mm Fg, pulses 120/min, and resirtions are 22/min, Bilateral thonchi are heard on chest examination. He is admitted for further evalua- tion, Chest x-ray shows bibaslr inflates consistent with bronchopneumonia. Blood tests show 12,000 Teukocytesmm with numerous myeloid Bass, Platelet ‘count is 15,000/mm’. A bone marrow biopsy demon: strates hypercelular marrow, with 35% blasts Elongated eytoplasmie inclusions consistent with Auer ‘ads are appreciated in peripheral and marrow blasts ‘Which of the following isthe most likely diagnosis? (A) Acate lymphocytic leukemia (ALL) (B) Acute myelogenous leukemia (AML) (€) Chronic myelogenous leukemia (CML) (D) Leukemoid reaction (E)_ Myelodysplastic syndrome |A.48-year-old man comes to the physician because of a 2-day history of severe low back pain. He states that he has had periodic low back pain for years, but this is sore severe than usual and radiates to the buttock and ‘down the right leg, His temperature is 36.8 C (98.2 F) Examination shows some cgity of the lumbar spine The pain is exacerbated by applying pressure on the paravertebral region in the lower lumbar spine and by passively raising the leg at 45 degroce wile the patient Ties supine. A reduced Achilles tendon reflex is noted. ‘Which of the following i the most appropriate next step in management? (A) MRI examination of vertebral column (8) Nonsteroidal anti-naramatory drugs (NSAIDs) and 2 days of bed rest, (€) Plain x-ray examination of the lumbosacral spine (D) Radionuclide bone scanning (8) Surgical consultation A previously healthy 30-year-old man i injured in an automobile secident. He is taken 10 the emergency department, where he is noted to have multiple lcera- tions of his extremities some of which are Bleeding profisely, His blood pressure is 7O/palpable mm Hy. “The decision is made to tranafse 2 units of blood after pid ctoss-matching. No reactions are detected in the blood bank, Ten miautes after the tansfusion, the patent develops a severe ease of hives. The development ‘hives in this setting would be most ikely tobe seen in ‘patient with which of the following syndromes? (a) Adenosine deaminase deficiency (B) Ataxia telangiectasia (©) DiGeorge syndrome (D) Selective Iga deficiency (©) Wishow-Aldtich syndrome Scion Two: Qbook Price Tests, Internal Medicine Tet One ‘A Ti-year-olé man presents with a complaint of dysp- nea forthe past week. The patent has a history of di. betes and hypertension and was recently diagnosed with cancer He is currently on multiple drug therapy. On examination, temperature is 37.2 C (99.0 F), his blood pressure is 140/90 mm Hg, pulse is 90/min, and respirations are 22/min. His lungs havea few crackles at the bases with no wheezing. A chest xray film shows bilateral difuse interstitial markings. Which ofthe fl: lowing medications slike responsible fr the patients dyspnea? (A) Bleomyein (8) cisplatin (©) Mithramycin (©) Verapamil (6) Vincristine ‘A 30-year-old woman complains of fatigue and dysp- ‘ea for the past2 months. She reports that she has also Jost 15 pounds during this time. She has been previous. Iy heathy and snot taking any medications. She is pale and thin and has a ow murmur on her exec exami nation. She also has mildly enlarged supraclavicular lymph nodes. Laboratory results ate notable for a hhematoerit of 309, mean conpascular volume (MCV) of 78 ym’, decreased transferrin ion binding capacity (TIC), and increased fereitn. Her blood sme shows rmicroctic red el. Which of the following isthe most ikelydiggnosist (A) Anemia of chronic disease (B) Aplastic anemia (C) Pyridenine deficiency (D) Sphorocytosis (©) Thiamine deficiency 2 ‘A.50-year-old man comes tothe physician because of san unusual appearing mole on his upper back. He sy that his wife has noted a recent change in its color and shape. The lesion measures 0.7 cm and has l-lefined ‘margins and irregular pigmentation. The patient i oth ‘erwise healthy and takes no medication. Which of the following isthe most appropriate next step in manage- (A) Follow-up examination in 6 months (B) Topical application of Podophyllum resin (©) Cryotherapy with iid nitrogen () Shave biopsy () Incisonal biopsy (Excision biopsy ‘A.52-year-old man with a history of chronic low back pain comphins of 3 days of a cough productive of purulent sputum, fever and left-sided subcostal pain worsened by breathing. A single episode of shaking 0.6 em, Bleeding and ulcerations ae malignant sins, leit far less frequent. Melanoma is the most common ‘use of death duet skin malignancies. Physicians can play crucial sole in prevention by refering to derma tologists patients who have moles with such suspicious features. The inital approach to a suspicious mole or clinically obvious melanoma consists of toa exision (excisional biopsy) with a small margin. Ifa diagnosis of melanoma is confirmed pathologically, wider mar gins ae excised ona second operation. Follow-up eramination in 6 months (cholee A) would result in a dangerous delay in diagnosis and trearment. Topieal aplication of Podaphyllum resin (choice B) and cryotherapy with liquid nitrogen (choice C) are treatments use for common and genital wars, as well as fr other common benigh lesions, suchas seborrhe fc keratosis, These methods should never be used on pigmented lesions. Shave biopsy (choice D) is applicable to many types of superficial skin lesions, ineuding baal el carcinomas, but is inappropriate for melanomas. Proper diagnosis ‘and evaluation of depth of invasion in melanomes ean be achieved only om full-thickness biopsies. Incisional Biopey (i.e, patil sampling: choice E) isnot appropriate winless the lesion is too extensive (such as fant congenital nevi or lentigo maligna). However, there seems tobe no fourdation forthe bli that i sional biopsy feiitates cancer spread ‘The correct answer sD. "This patient is demonstrating the dlsisic picture of pneumococeal_ pneumonia ‘Streptococcus pneumoniae s the most common cause of ‘community-acquired pneumonia in this age group. The ussal presentation is sudden onset of shaking chills, with rigors, high fever, and eificlty breathing Dleurte chest pin is ften present an signifies bate ‘ial infection. A white blood cel count, not provided in this cate, most often ie significantly elevated with a let shift (predominantly bands and polymorphonsclear alls). Chest xray fins usally reveal a lobar distribu. tion ofthe prewmonia.Pleualefusions are presenti. up to 3086 ofthe cases. Gram stain ofthe sputum com ‘monly reveals grars-posivediplococci in chains, Gram-negative diplococc (choice A) would be present in pneumonia due to Moraxella catarrhalis (formety Branhamelia catarrhalis). This pathogen may produce acute pneumonia and usally occurs inthe elderly or in those with «history of chronic beonchits or obstructive lung disease Gram-negative rods (choice B) are nota usual cause of ‘pneumonia inthis population of patients. Gram-nega- five rods causing pneumonia include Klebsiella, Enerobacer, Serratia, and Proteus, which oecur more ‘commonly in patients who are debilitated or resding in ‘nutsing homes or similar institutions. These bacteria are often responsible for nosocomial pncumenias and, infrequent community- acquired pneumonia. Gratn-positive cocci in clusters (choice ©) hat cause ‘pneumonia are sually Staphylocceus aureus, S. aureus js an uncommon ctuse of community-acquiced pneumonia. When it does cause disease, tis usually ‘ring or just following an epidemic of vel influenza 'S aureus may be seen year-round in the hospital, ‘ecauseftisa common cixse of nosocomial pcumonia, Gram-positive rods (choice £) would likely be (Corynebacterium diphtheria dphiberia). This patent presents with pneumonis, not diphtheria (an infection that occurs in the pharyn, middle ear, larynx, skin, oF bronchi ‘The correct answer is C. Hepaits E virus (HEV) is rarcin the US, but outbreaks of acute hepatitis E occur in some counties, namely Mevco, Inds, Afghanistan, tnd Burma. The infectious agent is transmitted by the foral-fecal route, usvally from infected water. HEV hepatitis generally sl imited, but itis important to note that it carries 10¥6 to 205 mortality rate in preg nant women, Similar to HAV, HEV infection does not ‘cause chronic hepatitis or acarir state. The onset of IgM antibodies to HEV is concomitant with the appearance of clinical symptomatology, whereas IgG will become detectable after the acute phase. RIBA (recombinant immunoblot assay) for ans-HCV antibodies (choice A) is used to conti a disgnoss of hepatitis Cin patients with positive anti-HCV antibodies by the more conventional enzyme immunoassay ts. The latter has 25086 specificity, and false postive results are ikely in patients with ypergammaglobu- linens. Occasionally ICV RIBA is lo used when the ‘enryme immunoassay is negative, but there are strong, clinical grounds to suapect thatthe patent hes HC hepatitis There is no reson to suspect HCV hepatitis fn this case ‘Anti-HDV antibodies choice B) ae foand in association ‘with hepatitis D virus infection, HDV i a “defective” RNA virus that causes hepatitis only in the presence of the surface antigen of HBV (HBsAg) In the US., HDV ie arly astciated with IV drug abuse. HDV may coi fect of superatect paticnts with HBV. Superinfecton is associated with increased risk for fulminant hepatitis or rapid progression to cirthosis, vo. AntisHGV antibodies (ehoice D) are found in $026 of IV drag abusers and 30% of patients receiving hemodialysis. HGV is a Aavivirus transmitted by the parenteral route. The infection is fllowed by viremia Tasting for at leas 10 years, The pathogenic role of HGV is aill uncertain, HBcAg (choice B) detection implies ongoing HBV repli ‘ation inthe organism, HBsAg appears fst in the blood before HBV infection becomes clinically evident Persistence of HBs4g is asocated with infectivity. HBV infection would be most unlikely inthis ese, since the patient developed ent-HHBV antibodies ter vacintion, ‘The correct answer is C. Traveling abroad often entails sbrupt changes indict and climate as well es exposure te conditions of poor sanitation, al of which results in a high incidence of diarhea, This ie self-limiting and manifests with watery diarrhea and dehydration, but no fever or other signs of systemic infection. The mos fe quent cause of troelers diarthea is enterotoxigenic Escherichia el The remaining infectious agents listed hore are all potential causes of noninlammatory darthes, whichis not associated with blood and mucus in the stool fever, systemic signs of infection, or eal leukocytes Bacillus cereus (choice A), Clostridium perfringens {choice B), and Staphylococci aureus (choice B), slong, With enterotoxigenic Ecol, are the most common agents associated with food poisoning due to produc- ton of toxins, All these pathogens produce similar clinical picture of watery diarhea, sometimes with ‘nauses and vomiting, bit no fever Rotavirus (choice D) is one of the most important infectious causes of diarrhea i infants and young chil- ‘ion/reinnervation in a muscle biopsy is confimatory ofthe clinical diagnosis. The patent will later develop cvidence of corticospinal and cortico-bulbar (upper ‘motor neuron) degeneration as his disease progresses. smal recessive demyelinating disease of per ‘that manifes in children or young adults with marked _sophy of thecal males and distal muscle weakness For this reason, the disorder is also known as peroneal ‘muscular atrophy. Guillain-Barré syndrome (choice ©) manifests with ascending paralysis (Grst the lower, then the upper ‘extremities are involved) and ceslts from a chronic inflammatory response leading to demyelination of peripheral nerves. Is often preceded by an upper res Piratory tact infection. Myasthenia gravis (choice) is characterized by uctu- ating muscle weeess that usually begins inthe ocular ‘muscles, resulting in diplopia and ploss Since the dis- order is due to impaired cholinergic transmission st the neuromuscular junction, skeletal muscle biopsy is within normal imits at the ight microscopic evel. Spinal muscular atrophy (SMA; choice B) is the nfm tle counterpart of ALS. SMA isa group of hereditary Alisorders, the mos: frequent form of which i Werd Hoffmann disease (SMA type 1), which lads to death by th 3d year of life ‘The correct answer is B. The speifc signs tha suggest the correct diagnosis include the wide discrepancy between the blood pressure in the upper extremities and lower extremities, the systolic murmur heard on the bck, and the notching ofthe ribs appreciated on x-ray. CCoarcation ofthe aorta in its most fequent (adult) ‘ype, consists ofa stenotic sortie segment just distal the origin ofthe left subclavian artery. Hypertension {develops inthe branches proximal tothe stenosis, and hypotension inthe aocta distal to it In tbe most severe forms the patients may develop left ventricular fallure Section Tw: Obook Practice Tess “answers and Explanations | © ‘in infancy, but the most common presenting picture is that of a young adult with hypertension, which may lead to let ventricular hypertrophy or cerebral hemor- tage. Aarial septal defect (ASD) (choice A) is generally ‘symptomatic. A large ASD usually lads to right ver Iecular fata in middle age. A systolic murmur is heard atthe pulmonary area, and $2 s widely split ‘Congenital aortic stenosis (choice C) gives rise to a harsh systolic murmur heard along the lft sternal bor ‘der and radiating tothe nec. Its due to congenitally abnormal, usually bicuspid, aortic valves. Congenital pulmonary stenosis (ehoice D) is a rare form of congenital valvular disease. Mid-to-moderate ‘stenosis is usually asymptomatic, but severe cases result in right-sided heart failure or sudden death. A systolic ‘murmur is heard atthe second left intercostal space, ‘often preceded by an jection click. ‘Adults with small o medioo-sive patent ductus arte- ‘owvs(choive E) are usually asymptomatic unt] mid “dle age. This anomaly is associated witha characteristic continous “machinry-like” murtus, which is maxi- ‘mal atthe pulmonary area and ofen accompanied by 8 thsi “Tetralogy of Fallot (choice F) isthe most common form of cyanotic congenital heart disease. The four features include subpulmonary stenosis, ventricular septal defect, overriding aor, and right ventricular hyperto- py. The degree of subpulmonary stenosis isthe single ‘must important determinant of the clinical severity and symptomatology, Most infants prescat with early cyanosis Ventricular septal defect (VSD) (choice G) isthe most frequent congenital cardiac anomaly. Most cases axe symptomatic. Large VSDs lead fo tight ventricular ‘overloed and are asocated with a harsh paneystolic ‘murmur along the left sternal border associated with a thal. ‘The correc anawer is Porphyria tana tds the tos common ofl ofthe porphyrin ard is cn ‘ently lily get nthe USMLE. tenses conc Wistering and crusting lesions on sun-exposed kin “The defective cozy in heme synthe oropor phyrinogen ecard. reptating actors tron even n normal owns nome ces), trogen se lebol nd chronic hepa Cinton Ski biopsy can he eign comely pe- Gif ihe “caterplar bois” in the question sem are lumps of basement membrane material). Porpyein sales demorstate he inings inte gustion em. | Seaton Two: Gbook Practice Tests Internal Medicine Test One Acute intermittent porphyria (choice A) is one of the ‘mioze common forms of poxphyia and typically pre- sents with severe abdominal pain, Delt-emiolevulinic acid dehydratase deficiency (Choice B) is 2 rare form of porphyria that can cause abdominal psin and hemolysis Enythropoietc porphyria (choice C) is one ofthe more common forms of porphyria and typically presents ‘ith acute, rather than chronic, photosensitivity with pin and sweling ater sunlight exposure, Hereditary coproporphyria (choice D) is rate por: pyria than presents with abdominal pain ‘The correct answer isEThe key data to make acomrect fagnosis ince the following: severe thrombocyrope- aia, which results ina bleeding disthess elevated indi rect bilirubin and high LDH with schistocyes in the blood smear, indicating microangiopathic hemolytic Anemia; renal dysfunction (high creatinine and new- ‘logic and systemic symptoms (headache, confusion, and fever). Negative findings important to rule out similar condition inclade a negative Coombe test and shsenee of fibrin split product. Thrombotic thrombo- eytopenic purpura (TTP) is a disorder of unclear pthogeness, perhaps related to circulating plate aggluinaing factors. Ie presents with a characteristic combination of mictoangiopathic hemolytic anemia, fever without infection, neurologic symptoms, bleeding dliathesis secondary to thrombocytopenia, and renal impairment. This condition may be precipitated by pregnancy or wse of estrogens, Disseminated intravascular coagulation (DIC; choice {A canbe diferentiated from TTP because of abnormal ‘coagulation tests. In DIC, microangiopathic hemolysis is alo present, but prothrombin time (PT) is pro: longed, fibrinogen levels are reduced, and fibrin split products ae dlevated. vans syndrome (chotee B) refers to coexistence of autoimmune hemolytic anemia (positive Coombs tet), and autoimmune thrombocytopenic purpura (see choiee D). Hemoiyticuremic syndrome (HUS; choice C) is not sigaiicantly diferent from TTP. The to conditions, i fact, ate considered manifestations of the same patho- genetic spectrum. However, the vascular bed of the NS is not involved in HUS; thus, mental status changes are not pat ofthe clnial picture. Idiopathic (autoimmune) thrombocytopenic purpura (OTP; choice D) is an immune disorder caused by autoantibodies o platelet antigens. Systemicillnesis not present in ITP, which s characterized by isolated throm bocytopenia without other hematologic sbnormaiis ‘Ten peroent of cses will manifest in asiociation with autoimmune hemolytic anemia (Ens syndrome), Malignant hypertension (choice E) may cause rmicroangiopathic hemolytic anemia, However, blood pressure values would be extremely elevated. “The correct answer is D. The porphyris are due to metabolic defects in heme synthesis. Although they occur in variety of forms the most common ae acute nermittent porphyria (which this patient has), ery thropoietic protoporphyria (which presents with painful skin and acue swelling) and porphyria cotanea ‘tarda (which presents with cheonic blistering skin Iesions) Acute intermittent porphyria characterstial- ly presents with neurovsceral symptoms, which may mimic an acute abdomen, The sbdominal pain pro- ‘doced isa nerve problem rather than an inflammation, ‘which is why exploratory surgery in these patents fs rsually unrewarding, Patents with long-standing cases may have demonstrable damage to motor nerves a8 ‘wel, which typically begins as weakness in the shoul ders and arms. The condition is relatively rare (although it i the most common acute porphyria; therefore, itis suspected more often than itis con firmed. The combination of complains of severe pain, distraught behavior, and absence of physical findings ‘may Tad clinician to suspect the patent either is abu ing rugs or has psychiatric problems. Failure to make ‘the diagnosis als raises the risk of potentially danger: ‘ous complications because of drug interactions with the disease (barbiturates ae notorious offender). The biochemical defect in acute intermittent porphyria ea block in porphobilinogen deaminase. Determination of urinary porphobinogen levels, which are best mea- sured in’ 24-hour urine collected during the period when the patient is symptomatic, isthe most important screening. test for acute intermittent ‘porphyria Aminolevulinic aid (ALA), which isan early precursor in heme synthesis, isalso elevated in the urine. IV heme «an be giver fr therapy Erythrocyte porphyrins (choice A) can be used t0 screen for erythropoietic protoporphyzia. Fecal porphyrin (choieB) area second-line choice for screening for porphyria cutee tard, Plasma porphyrins (choice ©) can be used to seen for ‘either porphyria cutanea tarda or erythropoietic por- phys ‘Urine porphyrin (choice E) can be used to sereen for porphyria cutanes tarda, ‘The correct answer is B.A small ventricalar septal defect (VSD) isusually asymptomatic, manifesting with ‘systolic murmur sometimes associated with 2 til slang the left sternal border. Patients with the typical ‘murmur as the nly manifestation have # normal life txpectancy but are more prone to develop infective endocatdits. Thus, antibiotic prophylaxis mandatory Doone dental procedures or other procedures that might produce bacteremia. Arthythmias (choice A) do not constitute a particular risk for patients with VSD. ‘A large VSD leads to a significant lf-o-rght shunt, ‘which increases the right ventriulr loud and rests in ‘pulmonary hypertension (choloeC) and right ventric lar yperteophy. The longterm effect ofthese hemady- namic alterations i right-side ear flue (choice D). ‘A shunt associated with a pulmonary-to-systemic ow ratio of less than 1.5 is hemodynamically inconsequen tial and should not be repaired surgically. Large shunts should be reprired to prevent late-onset pulmonary Inypertension and heat fre Shunt reversal (choice ) develops when the right ves tricular pressure exceeds that inthe lef ventricle and the shunt becomes right-to-left. This isa long-term complication of unrepaired large VSDs “The correct answer is F. Recurrent episodes of inflam ratory arthritis absence of urate crystals, and speck Ting (due to caeification) ofthe articular cartilage are virtually diagnoetic of prcudogost. The knees the ‘mort common joint involved. Kentifcation of caleurm pyrophosphate crystals in joint aspirates is diagnostic (teaky birefringent on polarized microscopy). It may be hereditary, may develop 24-28 hours after surgery, or may be asociated with metabolic diseases, such 28 hyperparathyroidism, hemachromatosis, hypomagne seinia, acromegaly, Wilson disease, hypothyroidism, and gout. Recurrent hemorrhages into joints, especialy the knees, are characteristic of poorly treated hemophilia (choice A) Minor or unappreciated trauma may precipitate ini vidual events Healing i associated with inflammation and pelifeation of the synovial membrane, and can lead to significant joint destruction, Widening of the Intereondylar notch ofthe femur is characteristic. Other clinical features of hemophilia are invariably present Cronie monaticular of oligoartcaar involvement, especially of the knee, is a feature of tyme disease (choice B). The primary stage of the disease may be ‘unrecognized. Caefcation or erosions of the articular cartilage do not cur. Section Two: Qbook Practice Tess | ‘answers and Explanations | & Rheumatoid arthritis (choice €) is uncommonly Imonarticular and enters the differential diagnosis Rheumatoid factor in the joint Dud may be postive, even when the serum cheumatoid factor isnot. The pt tern of presentation is chronig, rather than recurrent acute, monarticular arthritis, Ostcoarthits (choice D) at age 33, in the absence of prios major tama, would be rae. Crystal arthropathy ‘ay coexist with degenerative arthritis, and the latter fan progress more rapidly in the presence oferta: induced damage. Acute exacerbation of stable ‘ostcoarthrits should he evaluated for coenstent crystal arcritis asthe management strategy may be altered ‘The corect answer i E, Primary sclerosing cholangitis is a condition in which fbcosing inflammation of the intrahepatic and extrahepatic bile uct system evensu- ally lead tothe obliteration ofthe bile ducts and devel- ‘opment of cithosis. The underlying etiology of the damage is unclear, although toxic, infectious, andor futoimmne mechanisms have been postulated. The ‘nical presentation illusteated in the question stem is typical The asocation with inflammatory bowel dis- ce, particulaly ulcerative colitis, may provide a help fl dle. In some patients, AST and ALT may be mildly increased. The liver biopsy pictare may be similar in primary sclerosing cholangitis and the related cond tion, primary biliary ciehosis. The antimitochond:ial antibody test can be helpful, because itis negative in primary sclerosing cholangitis and positive in oughly 95% of cases of primary biliary sdlerosis. The most zation (coe B) in this case. The Findings donot suggest ‘syphis: however, both an HIV test and an RPR (choice D) would provide useful information, although neither wold aid inthe diagnasis of ABE lical “The correct answer is C. This patient is having a hyper tensive hemorrhage, The caudate and the putamen are the most common sites fr sch Bleeds (70%) hich can lead to dangerous devations in intracranial pres (OCP), atin thi patent. The signs and symptoms of Increased ICP, when present, portend imminent hemi ‘ion ofthe brain and certain death These patents require urgent intervention 1o lower thr bod pressure ‘The cerebellum (choice) isan uncommon ste (<59%) for hypertensive hemorchage. When cersbelar hemor- ‘hages cur urgent intervention x required because they ‘ean cau brainstem compression and/or obstrucive ‘nydeocepalus, Epidural hematoma (choice B) i usally the result of trauma tothe squamous portion of the temporal bone ‘ofthe skull and fs no associated with hypertension, ‘Subarachnoid hemorsnage (SAH; choice D) is inte ‘quently associated with severe hypertension and is ‘ually accompanied by meningismus. Once the SAH is Identified, neurosurgical imervention to sop the beeing ‘an be performes and the patient thereafter asa normal Iie expectaney. The most common nontraumatie case for SAH is berry aneurysm in the anterior potion of the citele of Wil ‘Subdural hemtoms (cholce F} results from tearing of the bridging subdural veins, mor offen due to traumna forshearing forces. Iris uncommon without taumaand, cerca when present, doce not tend to produce headache and increased ICP less very severe ‘The correctanswersB. The patent has unstable angina Unstable angina with ECG changes is associated with tial coronary artery stenosis in most cases. One goal ‘of therapy it prevent thrombus formation on complex atherosclerotic plaques: heparin is the most effective proven trestment to prevent progression of unstable fngina to myocardial infarction. Heparin is also required to maintain vessel patency when using rela- Lively fibein-spectic thrombolyties, such as «PA. Heparin may cause delayed thrombocytopenia in about 10% of cases, Nifedipine (choice A), 2 calcium-channel blocker, has no proven benefit in the therapy of acute myocardial infarction. The drug decreases afterload and may cause reflex tachycardia, ‘The patient does not meet the criteria for thrombolytic therapy (choice C). Te best candidate for thrombolytic therapy is one in whom the ECG has distinct regional ‘ST segment elevation or new let bundle branch block, ‘Thrombolytic therapy has not been shown t0 benefit patients with inverted T waves, ST segment depression, ‘nonspecific ST-T waves changes and chest pain. ‘The patient may eventually require catheterization {choice D) tose the extent ofthe coronary artery disease, butts not the appropriate next step in management Angloplascy (choice F) should be considered if chest pain rfactory to medical management persis, but it {snot the appropriate next step in management ‘The correct answers A. This patient has benign parox- smal positional vertigo. The pathophysiology appears to involve granular masses (tiny rocks) that ston the cupola in the inner eas, pushing the ci (ais) on the sensory cells down. Certain postions compress the calls ‘more, producing vertigo, Patients should be instructed to avid the peston that sets ff the vertigo. A canalith repositioning maneuver ie effective in most cases, but chronic cases may require sugical treatment. Some ass resolve spontaneously within a year Cholesteatoms (choice B) isa rumor-like, benign lesion that can destroy the middle ear and occurs inthe sting of conic ots medi. Herpes zoster oticus (choice ©), or herpes infection of ‘the ganglion of CN VI, causes severe ea pain, vertigo, ‘hearing loss, ad sometimes facial nerve paralysis. Meniere disease (choice D) causes the cluster of vertigo, ‘innit and factuating heating loss but is usually not triggered by positional changes. Presbyzcusis choice E isa progressive los of semiivity ta high frequencies with age ‘The correct answer i F Progresive supranucese ply {sa degenerative disorder that predominanly ales the midbrain and basal ganglia The clinial hallmarks are ‘rmmetric parkinsonism with vertical gaze limitation and axial sgiity. These patients tend to have falls 3s thei ears sympeoms. There isan ascociated mil moderate dementia that usually involves frontal lobe functions more than hippocampalimemory systems. These ptints tend wo show a very modest response ian to L-dopafarbiopa Imaging istypiclly unremarkable. ‘The basal ganglia (chofee A) i a common sit of small lacunar stokes in patients with hypertension and/or slabetes. Most of these tend to be asymptomatic, and vascular parkinsonism requires hewvier burden of is: ease in the basal ganglia In addition, a unilateral left pputamenal lacunar infarct would result in right-sided, rather than symmetric, symptoms. ‘Carbon monoxide poisoning (choice B) results in bik eral pallidal (globus palidus) necrosis. These paiens develop symmetric parkinsonian symptoms bat would not be expected to have vertical gaze problems. In addi- tion, an MRI would demonstrate tlsterl lesions in the putamen, ‘Section Two Cock Practice Tess ‘Answers and Explanations | 87 Cervical stenosis (hole ©) with impingement on the spinal cord can present with fll anda spastic gait but should not affect eye movements or cogoition. These patents should have hyperrefleca and upgoing toes iopsthic Parkinson disease (choice D) typically begins asymmetrically, with resting tremor and rigidity worse on one side of the body. Vertical eye movements ae not usually affected, Falls ocer but normaly a few ‘ears ito the course ofthe disease, Patients with iio pathic Parkinson disease will usually experience a pronounced improvement in symptoms when started ‘on Sinemet ‘The correct answers C. Thisis the typical presentation ‘of gonorrhea in men. The infection may regress spont- nevusly, progres to iavolve the epididymis and prostate, fo become chronic, resulting in urethral strictures. In ‘women, the infection is more often clinically sent, but when symptomatic, the manifestations frequently begin ring menses, with fequeney, dysuta, and urethral discharge. Chronic cerviciisisan important reserve of gonococei. IF gonococci cannot be demonstrated in smears ofthe discharge, cultures become necessary. For “uncomplicated urethritis or cerviiis a singeitcarns- cular injection of eeftragone, 25 ma, i the treatment of choice and guarantees compliance Amosiilin (choice A) and pencilin G (choice F) are ro longer recommended because of the increasing prevalence of penicilin-restant tains of gonacoccas, Chlamydial infection develops lrequenlyinasociation with gonorchea. Thus, therapy with ceftriaxone should be combined with a drug effective against chlamydia, Erytheomyein, 500mg. times daly for 1 week, or alter- natively azithromycin (choice B) ina single oral dose of Ig. may be used, Doxycycline (choice D) ils effective against chlamydia and should be administered ata dosage of 100 mg twice Sally fort wesk. ‘The correc answer is D. This woman bas carcino syn drome The asic triad of this disorders ashing (present fn 850), watery dartea, and valvalar heat esease The first tet for screening carcinoid syndrome isthe descr- ation of S-HIAA (metabolite of serotonin, S-HT) in a 2shour urine sample (arnold patien’s ‘may excrete more than 25 mgday)-Catcinoid syndrome i also sesocieted with hypotension, bronchospasm, telangiectasia, nd shdominal cramps duc tothe release of serotonin and vasoactive peptides, expecially in bronchial carcinoid. Thete may also be a secondary niacin deficiency, causing decmatts, depression, and «arta. Sympromaticweatment of carcino syodrome consists of giving the synthetic peptide octreotide wo. Section Two: Qhook Practice Tests Internal Medicine Test Two Barium examination ofthe bowel (choice A) will often not demonstrate the primary carcinoid rarer, most commonly located inthe distal ileum. Measurement of serum lipase and amylase levels (choice Bs indicated in the diagnosis of pancreatitis. Ulteasound examination (choice ©) would be indicated if she had symptoms pointing tan abdominal mass or gallbladder disease Small bowel biopsy (choice B) is invasive and would not aid inthe diagnosis The correct answer is E, The tumor i malignant melanoma, which is 2 neophstic proliferation of melanocytes The clin these tumors are related to new- rocndoctine cells and often sain immtinohistochemical- ly for $100. The prognosis in malignant melanoma slosly relat to the depth of the lesion, since shallow Jesons are auch ss itely to metastasize than are lesions (of L mm or more thickness, which have reached the rich Imphal plexus ofthe superticial dermis. The rests that shallow mielanomas have cose to 100% care rae with wide (typically 1 cm) excision, whereas deep melanomas have a dreadfal prognocis since they typical ly have already metastasinnd by the time of saga emoval and usualy flo respond to chemotherapy. Alpha-fetoprotin (choice A) is a marker for testicular ‘nd ovarian tumors with a yolk sac component, a well fs hepatocellular carcinoma. CAL125 (choice B) is a marker for some ovarian LCA (chotee C) is + marker for some Iymphoid neo- plasms PSA (chatoe D) is a marker for prostatic carcinoma, ‘The correct answer is D. Ths patient probably has siardiasis, caused by Giardia lamblia, The diagnosis of ardiais can often be made cinically on the bass of symptoms of flatulence and bloating eppearing several days aftr a trp to Meco. Metronidazole isthe treat ‘ment of choice for giardiasis Supportive cae with IV uid (choice A) is used for patients with Eschrishia coi “travele’s diarrhea” who fare severely dehydrated, Pacients generally have di thea the day after they return fom their tip, rather than t week later Gproflosacin (choice B) is effective for Shigella and Salrionella infections, Patients usually have fever and blood or leukocytes in the soo, since these agents are Mebendazele (choice C) is indicated for infection with helminths Timethoprim-sulfamethoxazole (choice E) is not elective against Giardia lamblia, The corect answer iC. Diabetes melitus isthe most common ease of cron tena are in the US. (and probably ia all indusilzed counties). Diabetic ephropatiy one f the mos severe complications and rants on average 10-15 yer afer the onset of di ‘rs. The east expression of diabetic nephropathy is ‘icroabuminura while the patient i ohereise asymp- tomatic. Tis isthe rationale for screening diabetic Fallen foe miclbuminur, which should be p= formedby 24-hour wine aletion or onan ety m0 ing urine sample In the later css, dipstick screning (Choice B) may not be sulicetly sensi. The aie rican rai ina etl moving urine sample i ‘Teomenint alternate to 2choutealletion. A ratio. {35 Enormal and >10 is abnonl: betwen hese two ‘ales, re-evalationircommended During the phase ‘f microatbuminura, aggresive treatment inling {ec geemic and blood presse cont, in ode Measurement of serum creatinine levels (choice A) ‘would not be Yauable in detecting pretinical real damage Theamicat with ACE inhibitor (choice D) has been shown to slow progression of renal nephropathy, pos bly bese of the reduton of intaglomerlar pres sure This tetent isnot widely sed if micoabu- ‘nara absent and the patient i normotensive. Renal biopsy (choice E) is not indicted in asymp tomate diabetic patents a2 method fo prevention of renal disse. The correct answer is B. The patient has pernicious anemia in which gustric atrophy is associated with imegaloblarc anemia due to vitamin By, deficiency. ‘The gastric atrophy characterstcllyivolves the cor pos, with sparing of the antrum. Most cases appear to have an autoimmune bass, with antibodies to parietal calls detected in 90% of patients; antibodies to intrinsic factor and the proton pump (H™/K* ATPase) are also commonly present. The lack of parital calls and the damage tothe proton pump lead to markedly mmen) aged 20-40 years. Bleeding, epistaxis, oral bleeding, or menorzgia ‘an occur and isolated thrombocytopenia (1,000) is characteristic Ten percent of patents wll have coexistent autoimmune hemolytic anemia (note the reticulocytes and anemia). The first-line therapy ie prednisone if the patient is not atively bleeding Patients eho ae Bleed ing may require IV immusoglobalin (choice B) to block phagocytic activity in severe cases, splenectomy (choice E) may be required Gryoprecipitate (choice A is effective treatment for von ‘Willebrand disease, Plasmapheresis (choice D) is the teatment of choice for hemolytic uremic syndrome (HUS) ina coagulation disorder sing, The mos striking fates of HUS are fever, fragmented RBC, and real failure without neu rologic signs. HUS is often sen afters diarheal lines, particularly after infection with Escherichia coli ‘The correct answer is E. Papillary structures within fl- Ticks that have al eels with mucei with cleared ‘enters Conphan Annie eyes") indicate the presence of pupillary carcinoma of the thyroid. It does not matter whether Une papiary stuctures are present ia oniy percentage ofthe follicles the condition i sill consid= ered to be papillary carcinoma. OF all thyroid cancers, 60% to 70% are papillary carcinonsas The condition i rote frequent in younger patients but tends to he more aggresive in the elderly. There i usualy a single domi nant nodale that js"eold” (doesnot take up cadioactve fodine) on thyroid scan, Among the diferent types of tyroid cancers, papillary carcinoma tends to be the ‘one with the best prognosis overall and smaller lesions «an be treated with thyeoid lobectomy alone, Large of mote diffusely spreading lesions require complete thyroidectomy, sometimes with ablation of any residual ‘thyroid tissue with lange doses of "1 Follicular carcinoma of the thyroid (choice A) would rot exhibit the papillary structures or oxphan Annie mace seen in this case, Well-iferentated folicolar carcinoma can be very dificult to distinguish fom nor smal thyroid issue iédical 1. Section Two: Qbook Practice Tests Internal Mesicine Test Two Graves disease (choice B) would be characterized by prominent hyperthyroid symptoms and would show smaller than normal amounts of colloid on biopsy. This disease is not characterized by a single hard ole: a symmevic, diffusely enlarged goiter may be found. Hashimoto disease (choice ©) would produce difuse foiter and would show an intense lymphocytic infl- tate with destruction of flies on biopsy. Symptoms of hypothyroidism are often apparent Nontoxic goiter (choice D) produces goiter that may be eithersmooth or multinodula, but does not usually have a single dominant nodule. On biopsy, the Flies se typically ofa wide range of sizes, and the patient is casually inically euthyroid ‘The correct answer is B. The dose of traditional ‘unfractionated heparin required for anticoagulation can be determined by following the pata thrombo- plstin time (PFI). Heparin prolongs the PTT. This testis performed by adding particulate matter to a patients blood sample to activate the intrinsic coal tion cascade; the PTT therefore reflects activity ofthe intrinsie coagulation pathways. Bleeding time (choice A) reflects the interaction of platelets with the vascular endothelium leading to the formation ofan initial cla. An abnormal bleng time usualy recs abnormal or diminished places Factor Xa levels (choice B) are used to follow the dos- ing ofthe newer, low-molecalar-weght heparins. Platelet count (chotce C) may be followed wile giving 1 hepari, since significant minority of patients will develop heparin-induced thrombocytopenia. The platelet count, however, snot the test used to monitor cffcacy of heparin therapy and any dosing changes Prothrombin time (PT; choice D) i a measure of the estrinsc coagulation system, This valu, and the core- sponding interntionsl normalized ratio (INR) of patient and normal PTs, is particularly sensitive to defi tdencies in factor VI. I is usually used to help guide {Coumadin therapy. ‘The correct answer is A, Acoustic neuroma, also ‘known as neurilerimoma or schwannoma isa beng, tumor that typically arses from the neurlemmal ‘sheath ofthe vestibular portion of the acoustic nerve in the auditory canal. Spmptoms are produced by com presion oF displacement of the cranial nerves, bran stem, and cerebellum, and by obstruction of CSF flow. “The trigeminal (CN V) and facial (CN VID) nerves are ‘often affected because oftheir anatomic location and relationship to the acoustic nerve, Clinical findings 1. {ncude insidious onset of hearing loss, nnitus and a sensition of fllnes in theca. Fail pain facil weak ness headache, and gat ataxia may alo be present; ver~ tigo ultimately develops in 20% to 30% of patients. The ‘most useful diagnostic test MRI of the erebellopon- tine angle Treatments surgical excision of the lesion, In Bell pal (choice), only the fics nee is fected. Patients with benign positional vertigo (choice C) ‘experience veriginous symptoms only when the bead ie in a specific position, Simptoms are usually most severe when the patient isin the lateral decubitus pos tion with the affected ear doen, Hearing loss is not @ featute ofthis condition, Ther sno specific reason at this time to suspect Lyme disease (choice D), although it should be Included in the differential “Meniere disease (choice H) is characterized by repeated episodes of vertigo lasting minutes to days, tinnitus, tnd progressive sensorineural hearing ls. ‘The correct answer is C. Obesity can be defined by ‘using the nomogrems based on the statistical studies of the National Center for Health Statstis on large popu lation samples inthe US. These data allow estimating ‘the body mass index (BMI fom height (in inches) and ‘weight in pounds) [To calculate his inéex more rigor ‘ously, BMI = weight(height), where weight is in llo ‘rams and height isin meters}. Most authors agre tbat ‘obesity is presont when the BMI is higher than 30 kgf? This patient has a BMI of 33 ky? Once obes ty is identified, the most appropriate next step is t0 determine whether obesity is secondary to underlying ‘pathologic conditions, Physical examination and histo ry shold focus on detecting signs and symptoms ofthe three most common causes of secondary obesity, name- Iyhypothyrodism, Cushing syndrome, and genetic con dltions. In this ese, menstrual regularities and slight hirsutism ace eatuses that suggest thence to undertake ‘xldivonal investigations to rule out endocrinologie or genetic causes. Stin-Leventhal syndrome (in edition fo those meationed) should be considered in this case [No intervention at this time (choice A) would not be appropriate. Active diagnostic and therapeutic interven tions are needed for all patients whose BMI is 230 ‘nd patents with BMI 227 kg/m? should be ccouraged 0 lose weight Tn fact, it is wel established that obesity i associated with increased risk of develop ing hypertension, diabetes, cardiovascular disease, cholelithiasis, palmonary dysfunction, osteoarthritis, and some forms of cance “The pysician should explain the rts of obeity and benefits of weight loss (choice B) to all overweight patients, although most of them are already psinfully avvae of the disadvantages inherent to obesity. ith regards to both heath is and social stigma, ‘Treatment options fo obese patients include decreasing caloce intake andior inereasing energy expenditure, Variably combined with techniques of behavioral modi Feat. & weight reduetion program for a very-low ‘alorie diet (choice D) is an aggresive approach that replaces the whole daily food intake with a diet or a beverage containing ne more than 800 kealday, 1g proteinikg of body weight, plus all of the necesary vitamins and minerals. This diet is used as part of 4 comprehensive weight-loss program, in which the patient is closely monitored by a medical specialist to prevent possible serious adverse eects Pharmacologic treatment with the serotonin reuptake inhibitor sibutramine (choice B) has been approved by the FDA for treatment of obesity. However, the long ‘erm effect of this therapy are stil under investigation. Hypertension has been observed in some patients. In any eas, thistypeaf treatment should be used only after secondary obesity hasbeen exclude ‘The correct answer is F. Canoes involving the upper third ofthe ciophagusare usally squamous cllin origin; thote of the lower esophagus can be ether squamous call carcinomas or adenocarcinomas (sully arising in Barrett esophagus). sophageal cancer has avery poor prognosis (mos patients die within 2 years, because the cancer has usually advanced through the esophageal wall by the time the patient presents with dysphagia, Because the esophageal walls thin, itiseasy forthe cancer to penetrate tothe level of the lymphatics (lee thar 1 mim), where metastasis can occur, ot completely through the esophagus (2-4 mm) t9 diretly involve rnonresectable mediastinal structures, such a the aorta, Inert or the vicinity ofthe carina ofthe bronchial tee. Predisposing fctrs for squamous ell carcinoma ofthe csophagus include alcohol and tobaceo use, human papillomavirus, esophageal scarring (lye ingestion, ‘nradiaton), sclerotherapy, and chronic achalasia Adenocarcinoma (choice A) of the esophagus usually arises in the distal exophagus, often in an area of Barrett disease Both large cell (choice B) and small el ( Iymphomas ofthe esophagus can occu, but ae rare Ost cell carcinoma (choice ©), analogous o oat cellear- cinoma af the lng, isa rare form of esophageal eancer D>) a1. 2, Secon Two: Qbook Practice Tess | “Answers and Explanations | ‘The correctanswerisC. This patients experiencing on aplastic ersis due to parvovirus infection. Parvovirus ‘exposure is common in daycare centers. Individuals with sickle ell disease, ike those with other chronic hemolytic diseases, are susceptible to infection with parvovirus, Patents usually present with weakness, lethargy, and severe anemia that solten preceded by & few days of nonspecific symptoms. These patients have {intense reticulocyrapenia, and the bone marrow con tains no erythroid precursor call, despite a normal ryyeloid series. A transient aplastic crisis due to par- voviru infection may produce life-threatening anemia and may requir urgent transfusion. Note that Howell. Toll bodies are consistent with asplenism. Inluenca virus (choice A) and parsinluenza virus (Choice B) both present with fever and systemic symp- toms, including myalgia, headache, and_ malaise Sudden development of severe anemia with veticulocy- topenia is not sen in these infections. Salmonella infection (choice D) is more common in ‘sickle cell patients because of splenic bypofunction, but it presents either as typhoidal illness or diarrhea, Selective red cell aplasia is nota feature of Salmonella Infection. Note that osteomyelitis due to Salmonella is ‘more common in sickle ell patients Infection due to Sieptocceus peumsoniae (chee E), although common in sickle el disas patients becuse of ‘splenic hypofuncton, more frequently manifest s pct ‘moni or ess comment as meningitis. “The correct answer is D. The clinical picture i virtually pathognomonic of migraine expecially considering the remonitry visual symptoms (the aur), the throbbing quality of the pain, and the postive family history owever, migraine does not always present casa. A ‘common form of migraine may be more frequent with diffuse (not unilateral) throbbing pain of moderate Intensity which isnot preceded by visual disturbances for associated with nausea, Acute treatment is bused 08 the administration of antimigraine drugs atthe onset of symptoms, fective drugs include ergotamine tartrate or similar compounds, and sumatriptan. Prophylactic treatment it aimed at decreasing the frequency of attacks, Cslcium-channa blockers (choice B), for exam ple, have been used for prophylaxis. Acetaminophen (chofee A) is very unlikely to provide relief in classic migraine, especially if aspirin and ibuprofen have already fled to do so, Carbamazepine (choice ©) has been found particularly helpful in the treatment of trigeminal neuralgia (i doulowreus), bat notin migraine 23. Seaton Two: Qbook Practice Tess Internal Medicine Test Two Prednisone (choice E) should be used to treat patents manifesting signs and symptoms strongly suggestive of sian cell arteritis (tomporal arteritis), which may lead to blindness in the absence of effective anti-ndamme tory therapy However, temporal arteritis is much more ‘common inthe elderly and would not be inthe difer- ‘ential diagnosis in this 20-year-old “The correct answer is A. Ductal catcinoma of the pancreas soften devastating becaus, in roughly 96 of ‘ass it present ate i the clinical course, when i is no longer resectable Therefore, the frst step when this dis ‘esis suspected on dina grounds is a CT scan, which ie noe recommended asthe most cost-effective method ‘ofboth diagnosing and staging the tumor Le, determin- ing whether i is potentially resectable), If unresectable disease i detected, then definitive tissue diagnosis (or choice of chemaxherapy) canbe madeon thebass of CT- tided perewiancous needle aspiration or biopsy: If a Potentially resectable lesion is identified, endoscopic Ukrasound (not yet widely avalable) can be wed to search for small metastatic lesions not visible with CT MRI (choice B) ie no more accurate than CT in detect ing and staging pancreatic eancer, andi more expensive. Ultrasound (choice C) is stil frequently used, bat usa- ally i followed by CT sean. Therefore, the current thinking sto eliminate this est and go direcly to CT, since the ultrasound is es sensitive. Acterigraphy (choice D) is rarely used and is primari- ly for determining vascular invasion and tumor ressctabiiy Endoscopic retrograde pancreatography (choice B) is stil commonly used in working up pancreatic cancer However, this is an invasive procedure and would not bethe frst step in diagnosis ‘Thecorrectansweris A.A sensation ofa curtain passing cross the visal fed can be characteristic of either amiauross ga o etinal detachment choice). a this ‘se the phenomenon is ransient, so amaurosis fuga is ‘more likely than retinal detachment. Heting bindessis 110 11) and often associated with mild hyperbilirabinemia Bone marrow biopsy (choice) isnot necessary in clssie ‘cases of microti anemia seconéary to iron-deficiency It is sometimes helpil to rule out anemia of chronic diseate which is associated with normal ot elevated serum ferritin and increased ion stores in the marrow. The correct answer is A. The aate onset of tenosynovitis is often the inital manifestation of gonococeal archi, Rise factors specific to this patient indude female sex ‘menses, and pregnancy. Signs and symptoms include fever and involvement ofthe knee and ankles. Obtaining ‘ultutes from the cervix rectum, pharyas, and blood wil ‘give a higher probability ofa postive rest than cultures from the joint uid alone. Gonococeal arthritis i con- firmed by prompt response to antibiotics, Blood cultures (choice B) are postive in only 4096 of patients “The immunologic reaction to nonviable fragments of the organist’ cell wal wil interfere with culture ofthe ‘organism from skin or joints (chotee ). These cultures wll be positive in Tes then 50% of eases synovial uid eel onan (choice D)ishelpl in making the diagnosis of septic arthritis bu notin identifying the pathogen causing the infection, ‘Synovial Buid Gram stain (choice E) i typically post= tive in only 25% of eases. “The correct answer is B. This is hydatid disease, de to infoction with Fehinococcus granulosus (rarly Echinococcus muller). The fe ece ofthis parse site worm usually alternates between sheep and canine Camivores (including sheep herding dogs). Man is an ‘sccidentl host, Endemic stea corespond tothe major sheep-herding areas ofthe weld: the Mediterranean the ‘Middle East, Australia, New Zealand, South Africa and South Ameria. Smaller foci of the disease are found in (Califia, Canada, and Alaska The ingested eg hatches inthe intestine, nd te lrva migrates tothe hurman liver, Jungs om les common ater body sites Over years the larva forms the hydatid cyt, which salary, Hui bladder that develops multiple brood capsules in its periphery each of whic contains numerous small infec tive seolces. So long as the st does not rupture, the palient may be asymptomatic. However, rupture including accidental surgical rupture) can cause an snaphylactic reaction (the est Sui is highly antigenic) ‘ora “metastatic” infection, as the upto milion of infec tious scolies are released. Treatment canbe ether with ‘very carefl surgical resection or wih percutaneous asp ration under CT guidance followed by instillation of a scolecoidal agent and then reaspiation. I the case is inoperable, o if an intraoperative spillage occur, abe drole canbe used to suppress growth or Kil the ests Ascatiasis (choice A) can case biliscy obstrction by the adalt worms and granulomas inthe liver by the lar- Fascioliasis (choice C) can acutely cause tender hepatomegaly with fever and eosinophilia, and can chronically cause cholangitis and biliary fibrosis Schistosomiasis (choice D) can cause a. severe granulomatous reaction 10 the ova, producing hepatosplenomegaly, pipestem fibrosis, and portal hypertension Toxocarass (choice) can cause visceral larva migrans and hepatomegaly with granulomas ‘The correct answers B, This cae raises the problem of the clnieal approach toa solitary pulmonary nodile radiographically detected in an otherwise healthy sub ject. In large survey, 609 of solitary pulmonary nod. ules are beniga, and granulomas represent the most common benign lesion, However, there is no infallible clinical or radiologic set of evteria that can discrimi fate between benign and malignant ksions Factors favoring a benign lesion include young age (<40-45), small size (<2 em) and smooth margios of the lesion, absence of symptoms, and slow growth on siccesie ‘lms. Generally, calcification is aot a malignant feature, and presence of ‘popoorn-lke” caleifications definitely favors hamartoma. A hamartoma i s malformative lesion resulting From random admixture oftisues nor- rally present in the lung. nctuding cartilage, bronchial rucoss, and smooth muscle, Iti usually discovered incidentally Bronehogenic carcinoma (choice A) may present as a solitary nodole, but patents ate typically older and Ihave a much longer exposure to cigarette smoking. CClofcations are usually not seen ia malignant pul- Pulmonary abscess (choice C) manifests a¢ 3 pul- ‘monary infiltrate with cavitation and, frequently, an air uid lve, Accompanying symptomatology, suchas fever and cough, is usally present Sareoidess (choice D) affects the lang froquenty. A fase moltinodulae infiltrate is sen on chest xa ‘most commonly associated with hilar ymphadenopa- thy. The patent may have sigas and symptoms of estitive pulmonary disease Secondary reactivated) tuberculosis (choice E) manifests With mutiplenodoar an cavitary inflates inte upper lobes; the patient has low-grade fever, malaise, weight loss, and cough. Howeves, primary tuberculosis may result ina calcified nodule within te ling parenchyma, ‘whic the remaant ofan old Ghon complex Section Two: Qbook Practice Test ‘Answers and Explanations | 7 ‘The correct answer is E. The patient has sigas, symp: toms, and laboratory findings consetent with gmt ell arteritis, which isa systemie disase overlapping with olvmyaigirheumatia. 1 affects the sypefcaltem- poral artery most commonly, but may involve any ‘medium or large caliber artery inthe body. The most characteristic ements in the diagnosis inciade scalp tenderness (sometimes associated with palpation of 3 nodular and tender temporal artery), elevated ESR, & frequently normal lekoeyte count, and age older than 55 years. The major risk af ths conlition is blindness Seriving from exension ofthe inflammatory proces to the ophthalmic arcery. Prednisone therapy: must be Immediately started, and biopsy of the temporal artery should be obtained to confirm the diagnosis CCT scan of the head (choice A) would be useful obtain information in ease of suspected intracranial lesions especially bleeding infatt, or space-oceupying ‘Visual evoked potentials choice B) are especially wef in evaluation of optic nerve involvement in demyelint ing diseases, Lumbar puncture (choice ©) for CSF examination ‘would have no diagnostic value in this patient Meningitis would develop more rapidly, usualy with rhuchal headache and rigidity. Subarachnoid hemor rhage results in thunderclap headache followed. by ‘hangs in mental status. ‘Muscle biopsy (choice D) i use in investigating ms- cular disorders. Mylgia, arthralgia, and tines in the pelvic and shoulder girdles are often present in this condition, which may mime a myopathy. The correct answer iE. Uricara or hives isa common «condition that can be quite dificult to manage because it ‘may have such wide variety of trigger. The under basis ofthe condition is mediator- (histamine, srotoni, Teukotrienes) driven vasodilation with accompanying ermal edema. The tigers for mediator reese from mas als or basophils may be either allergic (Fg bound ‘0 antgen) oF nonimmunologic cites pharmacologic effec. Although theres a wide range of posible riggers for urticaria, i is important 19 note that up t0 25% of asesof acute hepatitis 8 present with urticaria. is thee fore wel wort checking the sclera (the mos visible site in tanned or dark-skinned individuals) for jaundice in Patients with newly diagnosed hives. Other ‘riggers include contact cherials, drugs, fod allergens, pres, sunlight, inset tings, and hereditary predispositions, Allhe other choices can produce acute or chronic hep atitis but do not have a sigaiicant association with medical | Sesion Tuo: Gbook Proce Tests 120 | intemal Medicine Test Thee 7. The correct answer is E. A persistently wide split S, is typically seen in patents with right bundle branch block, pulmonic stenosis, pulmonary embolus, and ectopic or pacemaker beats originating inthe left ven- tec All these conditions produce delayed function of the ight ventricle Ina Brazilian patient, Chagas disease (caused by Trypanosoma erat) should be considered as a possible cause of heart block. This trypanosomal infection is endemic in Cental and South American ‘counties, and is a cause of rhythm disturbances, ca diomyopathy, and thromboembolism. ST segment elevation (choice A) is seen in acute anterior myocardial infarction. ‘A short PR interval (chee B) is associated witha loud 5 (when the mitral valve slams shat). Early repolarization (choice €) will produce ECG changes only with no specific findings on auscultation. Although modest T wave changes can sometimes be associated with right bundle branch block, marked T ‘wave inversions (choice D) are more typeal of right oF Jett ventricular hypertcophy and myocardial ischemia oF infarction '& The correct answer is C.Hyperpaathyoidiom is one of ‘the most frequent endocrinologi conditions, found in 1 {in 1000 adults, ta most cases, hyporparathyridism is feympromatic, manifesting only with hypercalcemia ‘often discovered incidentally inthe course ofrotine lab- oratory investigations conducted for other reasons. The ‘most common exse ia parathyroid adenoma. Calcium levels should be corrected for albuminemia, since most of the cium is bound to serum albumin. i hypercalcemia isthe ony cinicl sig, without asocated complications such as renal stones, bone disease or cataracts abundant ‘ud intake isthe only measure recommended to prevent formation of cleum stones in the inary ser, Bone x-ray films (choice A) are not necessary forthe iagnestic work-up of hypercalcemia an hyperparathy- roidiam, unless bone pain or pathologic fractures are pre- sent. X-ray films may reveal bone rarefaction, which is flea more pronounced inthe phalanges and in subpe- ose locations, Extensive cancer serening (choice B) would be appro- priate if kypercalcemia were sspected t be secondary to neoplasms (e.g Iung, breast, repal cel carcinoma, ‘multiple myeloma). Hypercalemia of malignancy ean be due to secretion of PTH-like substances or bone destraction by metastases In either case, plasma levels ‘of PTH detected by IRMA would be low. In fact, the PYDke peptides produced by some tumors (ung can- et) are not identified by IRMA. Allaline phosphatase ‘would be high in the presence of steals, 10. ‘Teatment with bisphosphonates (eg, alendronate choice D) may serve a an alternative eatment to pre ‘ent excesive bone resorption, Surgical exploration of the nock (choice) is aimed at finding the source of increased PTH production, which ‘is usally a parathyroid adenoma, Remavsl ofthe ade ‘noma is recommended when patient hae symptomatic Ibyperparthyroidim, with recurrent renal stones ot bone disease. Indications for surgical treatment in symptomatic patients inchude very high calcium loves, high urinary excretion of aium, eareme bone loss, oF dificlty in medical follow-up. ‘The corectancwer is D This man has glacose-6-phov- phate dehydrogenase (GoPD) deficiency (as do 10% of Afeican American males), GOPD serves to protect the RBCS from oxidative damage by maintaining high ints calular levels of NADPH, People of Mediterranean ‘descent can ako have GOPD deficieney but to a much iteater degree. Therefor, hemolytic episodes in thie population are moze severe (and can be fatal), as com- | pared with those in the African American population, Which are wually mild and self limited, Common oxida tive stressors that initiate hemolysis are drug ceactions (especialy sua drug), febrile ines, and fava bean ingestion. Acute infectious hepatitis (cholce A) would more likely present with fatigue, fever, abdominal pain, hep- stomegaly, and high elevations of AST and ALT (often into the 100), CCholestatic liver disease (choice B) more often presents ‘with elevation of alaline phosphatase, alongwith mild evations of AST and ALT. This patient has elevated unconjugated biliubin levels, ab in hemolytic disor ders. Both hepatocllular (hepatitis) and cholestatic liver disease cause moze conjogated (as opposed to "unconjugated hyperbiirubinemia ‘The most common side effets of uoxetine (a selective serotonin reuptake inhibitor; choice C) are anxiety a tation, and insomnia, “The correct answer is B. Eichericho coli, a coliform Deters is the most common cause of urinary tract infection (UTD) and is therefore seen much more often ‘than Klebsiella (choice C) and Proteus (choice). colt ‘auses shout 8044 of UTs in patients without urologic abnormalities. Coliform bacteria colonize the urethra, fand ascending infection may lead to cytitis or pyelonephritis Chlamydia trachomatis (choice A) is part ofthe dir ‘ential Gignosis for UTL ina sexually active patient with ysuria However, the patient often hes a vaginal dis charge. Urine culture usually shoves <100 organisms. On physical examination, the pationt with Chlamydia may hive cervical motion tenderes i ceric i preset. ‘Staphylococcus sprophyticus (choice B) is another com- mon cause of UTIs in young women, often after heterosexual intercourse, The correct answer is D. This question exernines the 3 giday. The history ‘of heroin abuse makes focal segmental glomeraloscle rosis (FSG) the mos likely diagnosis, FSG may occar in an idiopathic form or in association with three condl- tions: morbid obesity, HIV infection, and heroin abuse A renal biopsy il reveal sclerosis occurring in some, but not all, glomeruli (focal) with each glomerulus showing partial involvement (Segmental). Electron microscopy shows detachment of epithelial podocytes from the glomerular basement membrane, an alter= ation also seen in minimal change disease, Note inthis inical case, the coexistence of nephrotic features (marked proteinuria and edema) with nephrtc signs (hypertension and microheraturia), which i offen present in FSG, 126 2. Section To: Qbook Practice Tests Internal Medicine Test Three Acute proliferative glomerulonephrtis (choice A) is characterized by proliferation of endothelial and ‘mesangial al with inlux of leukocytes. The glomeruli ate hyperceluar, and immune depois areepimemira- nous in location. This patter is associated wi nephritic syndrome, ie, proteinuria <3 piay, hema turia, hypertension, and pedal and periorbital edema, Te prototype of this glomerular disease is postiniec- tiows glomeralonephrits Crecente glomerulonephits (choice B) owes its desig- natin tothe crescent-shaped mases of cells (epithelia and inflammatory) that accurmolate within the urinary Space of Bowman capeule, obliterating the glomerar tf. This resus in a rapidly progressive renal failure, equiing aggressive immunosuppresve therapy ‘Minimal change disease (choice D) is mostly a disease of childhood! It manifests with fll: blown nephrotic syndrome, On light microscopy, the glomerul appeat normal, only to reveal alterations ia epithelial podocytes on electron microscopy. ease: choice E) is pathognomonic of diabetic nephropathy. Round PAS-positive (ie, glycoprtein- rich) globules are seen within the glomeral. This fe ture along with diffuse mesangial sclerosis due to aceu- ‘ulation of altered glyeoproteins of plasma origin, ‘constitutes the pathologic substrate of diabetes-related ‘renal dysfunction, ‘The correct answer is B. A basic understanding of blood product and blood component replacement ie crucial The use of such producti extremely common, tnd there is misuse, Patents with liver disease hae deficiency of one or more cating factors produced by the liver A blood product that specifically raises such factors is indicated for treatment. Fresh frozen pasta (PEP) generally increases plasma anticoagulation factors bby 30% Like all blood products, itis type specific. There isa correlation for prothrombin times greater than 15 and the risk of Bleeding seit invasive procedvres sich ts paracentesis, Fortis reason, FFP is usualy indicated in such patients prior to undergoing their procedure Ceyoprecipitate (choice A) is prepared from FEP and contains concentrated factor VI, factor XI, fibrino- gen,and von Willebrand's factor. Indications fr use are hypoiibrinogenemia (DIG), von Wilebrand’s disease, and hemophilia A Packed red blood cells (choice C) are prepared from all the red call mass ina pint of donated blood, It has no plasma or btiy coat and therefore no proteins ( patients and soften the fist pre ‘eating sign of AIDS. Patients with CD44 counts <200 should receive prophylaxis against Purest arnt pneumonia (PCP) with double-strength trimethoprim- ‘ulfamethorazole (TMP-SMX), In sulfa-allergic patients, ‘dapsone or pentamidine may be used, ‘The correct answers D. Dog bites are the lest key to become infected compared with cat and human bits. The infection rate for dog bites is nly 59% for ca ites 30't 50%, and an intermediate igure for human bites. The pathogens accounting for infection are different depending on the biting animal and thetimeof onset of infection. Barly infections (in te ist day) due to dog ‘occat bites are usally secondary to Paseurli multe: ‘a, This agent is sensitive to penicillin or tetracyclines, however, the response is slow and treatment should be continued for at leet 2 weeks, Ea infections fllow- ing human bites are usually due to mixed aerobic and snaerobic bacteria (choles), which are normal com- ponents ofthe oral fora 130 2, Section To: Qhook Pracice Tests Internal Medicine Test Three Capceytophaga canimorsus (choice A), Fikenllacorr- ens (choice B) nd especially staphylococi (choice E) and steptococc! (cholee F) are responsible for late infections, occurring more than 24 hours afer a bite Capnocytophega canimorsus is 2 gram-negative organ- ‘sm ofthe canine mouth flora, whereas Fikeellacora- diens is @ saprophyte of the human mouth. The pathogens involved in human bite infections are so ‘variable that therapy shouldbe adjusted once antibiotic ‘seastvty has been determined onthe pathogents)o- lated from cultured wounds. “he corect answer i FE. This patent has pomp *euratica, which affects older individuals witha 2101 femal o male predominance, The disease character Jaed by bh severe muscle pain with stifnes and us ally prominent systemic symproms, suchas malaise, eve, and weight loss. The muscle pn and sine: tent involve ares near the tunkad ar not acon panied by muscle wasting or evidence of muscle dam ge on clecromjoaraphy oF biopsy. This condition i asccited with temporal arteritis (which can cause Blindness). Patients should be warned to inorm tele lyin prompaly if they develop severe headaches, ‘sual changes, oF significant jw pain 08 repeated chewing. ‘Cro cscs (choie A) i ascited with aniosing spondylitis and a peripheral atrits Dermatomyositis (choice B) is associated with polymyositis and polyartrigassometimes wih joint Selig and efusons. Disco pus erythematosus (choice C) is ocesonally sccompanied by relatively mild artralgias. ‘Sjgren syndrome (choice D) may be accompanied by ‘heumatoid aie o opuslike symptoms. "The correct answer is D. Nitroprusside is « nonselec- tive veno- and arteriodilstor that works via teleate of nici oxide. Te intrnsie ability ofthe lungs to match ventilation with perfusion via vasoconstriction in rel tively underventlte lung areas is abolished with this therapy. Therefore, large areas of ViQ mismatch ate create that result in often profound hypoxia. Hlevation of earboxyhemogiobin levels (choice A) is incorrect. Carboxyhemoplobin is the result of binding ‘of carbon monoxide to hemoglobin This moiety can not bind oxygen and results in a drastically reduced ‘oxygen-carying capacity. This patient has not been ‘exposed to carbon monoxide. The byproduct of stro russide administration is cyanide. ‘Methemoglobin (choice B) is an avidized form of Jhemoglobin in which the inon isin the Fe configure ical 45, tion. This often results fom blood being exposed to strong oxidizes such as nitrites. Methemoglobinemia is “suspected in a cyanotic patent wis normal oxygen te son, This form of hemoglobin cannot catry oxygen. Hypovenilation (choice ©), whichis one ofthe four ‘mechanisms underlying hypoxemia, is wsually quite apparent on clinical inspection, Pulmonary embolism (choice F) would certainly pro- dace hypo, but inthe absence of any clinical or his torical evidence for this diagnosis i is very unikaly at this time. ‘The correct answers B The csicl pcre is ons tent with ARDS, a daoder tha may be tigger by 3 suber of different conditions, arnong which i ace paces Typically ARDS develops 12-8 hours fol lowing the nitating event (3-4 days after cute pancre ait) and js character by acute respiratory faire unresponsive to supplemental oxygen. Therapy inchudes treatment of the undeving condition and ‘mechanical ventilation with pesive ead expiratory presse PEEP) The overall moet rates 308 ‘Acute biel broachopneumonia (choice A) may result in shortness of breaths boweves, anally docs not case such severe rexprtory fae and is conte | ‘monly scompanied by productive cough and fee. Farthemore, chest x-ray finding, especially ise {nflrates and aie bronchogrars are camisten! ith [ARDS nd not wth bronchepeumoni. Cardiogenic pulmonary edema (choice €) must be ruled out, sce specie eatment avaiable. A nor ‘nal care shot and the pulmonary changes on chest a1ay do not support a disgnoss of eardagenic pulmonary edema. tn uncerain cases, it maybe neces $ary to measure the pulmonary capilary wedge pes sueby Svan-Gane etter. racerbaton of acute poncrsii(choiceD), pes, is ‘lode by the cnc symptomatology and laborsto= 1 findings. Tere sno hyperemia and leukocyis ate ony milly elevated; however, the symptoms of ‘cate parceits can appear to iprove while the ‘erapuncreatic complications (ie, ARDS) worsen, ‘The correct answer is B. The clinical presentation i characteristic of Prinzmetal angina, a form of reeurent myocardial ischemia due to transient coronary vazospasm, ST clevation on ECG during ischemic episode is highly characteristic, This condition is most often seen in women younger than 50. Coronary angiography frequently fis to disclose any stenotic sg ‘ments, but the ergonovine test (to be performed with great caution) triggers vasospasm and anginal pan, “Myocardial infarction (choice A) usually manifests with intense precordial pain that persists for more than 30 ‘minutes associated with characteristic ECG changes Psychological chest pain (choice C) may manifest with variable clinical patterns and is usualy associated with ‘other symptoms, such ae depression, ame, oF panic stacks, ECG changes, such as ST elevation, ae absent, Precordal pain due to stable angina (choice D), by def= ‘nikon, manifests with a predictable pattern, usually {allowing constant amoun's of physical exertion, emo tional stress, or exposure to cold temperatures, Unstable angina (choice B) usually follows a period of stable angina. The attacks of precordial pain become moe fequent and less predictable, and tend to occur at rest. Increasing, degrees of coronary artery stenosis andlor platelet urombi are thought to be the underly ing pathologic ubstate “The correct answer is D.The mos ely diagnosis in ‘thisyoung woman is multiple sclerosis. Trigeminal new ralga typically occur in patients olde than £0; howev- ce onic at young age, bilaerlity, and presence of “objective signs of sensory los on the affected side ra cer the suspicion of multiple sclerosis (MS). MS should De suspected ina patient with matiple nearologe find: ings that are “separated by time and location’ Acoustic neuroma (choice A) isa tumor atthe cerebello pontine angle. Is aswociated with sensory hearing loss, facial nerve pas, cerchellr dysfunction on the affected ‘de, and headache. Ii common in patients with new rofibromatosis. Amyotrophic lateral sclerosis (choice B) is 2 form of progresive motor neuron disease characterized by both ‘upper and lower motor neuron involvement; fiscicle tions, muscle wasting, and weakness are observed. Bes in the late tages of illness, sensory, bowel, bladder, and cognitive functions are preserved, Dementia is unusual The illness is relentlesly progressive, leading ultimately to death Bell palsy (choice C) is characterized by loss of facial nerve function, Facial paralysis, loss of taste sensation, and hyperacuss may all be produced, Myasthenia gravis (choice Bi an autoimmune neuro: museular disorder involving skeletal muscles. The nds ‘menial defect is a decrease in the umber of available ‘ceylcholine receptors atthe postsynaptic muscle mem- brane. Sensory changes, bidder and bowel involvement, snd loss of tendon reflexes are not usually seen a. ‘Section Two: Qbook Practe Tests ‘Answers and Explanations | 13? The correct answer is C. The rapid progression of a febrile illness, the stoic murmur, and the objective ev dence indicating intravenous drug abuse point wo infee tive endocarditis a thundering etiology. The current recommendation isto obtain thre diferent blood sam ples for culture over a 24hour period before inating antibiotic therapy, unless the patients condition ise. Thisallows identification of the infectious agent and ads inthe choice ofthe mos appropriate anbitic epimen. Intravenous drug abusers are pasiculay prone to right= sided infective endocerdiis, especialy Staphyloperes ture infections on te tricuspid valve. Note that aur ‘urs are infrequent in right sided infections. Broad spectrum antibiotic therapy (choice A) should be started immediatly after obtaining the necessary blood cultures. Une cultre results are avaiable, the ‘combination of antibiotics should cover the three most frequent organisms responsible for infective endocst tis affecting native valves: viridans streptococci, Staphylococcus aureus, and Enero _Echocardiogzapbic studies (choice B) may document che presence of vegetations. Transthoracic echocardiography has a senstiviy of approximately 60%, whereas tans: csopagea is 90% sense in identifying vegetations Toricologie studies on blood and urine (choice D), although generally useful to document use of iit drugs, would not be heipful asa diagnostic foo in this ‘Ventilaton- perfusion lung scans (choice B) are used for the diagnosis of pulmonary infarction, which may ‘occur af result of embolism from right sided vegeta “The correct answer is B. Patents with sovere aplastic anemia need bone marrow transplant. If the patient needs blood products, they should be leukocyte reduced to reduce allosestzation in likely transplant ‘candidates (even in bone marrow transplant patient). ‘The cause of ths patient’ aplastic anemia is not clea. Up 10 50% of cases are idiopathic, though many of these may be from an undiagnosed viral infection that induces bost immune response ori dretiy marrow toxic. A small subset of cases follows an undiagnosed hepatitis, though thee iso link with known viral hep itis subnypes Immune- modulating drugs, such x antithymocyte glob tlm (choice A), aathioprine, methotreate and corti ‘steroids (choice E) are used in different bone marrow transplant protools. fa older patents or patients with ‘ut good grat matches they may be used as definite ‘herapy. The primary teatment of severe aplastic ane ‘mia however, js bone marrow transplanation 12 ». Seaton Two: Qbook Pratce Tess | tntemat Medicine Tes Tee Broaé-spectrum antibiotics (choice C) are not indicate, as there i no evidence ofa bacterial infection causing this patient’ symptoms Colony-stimulating factors (choice D) do not teat aplastic anemia, which is primary marrow failure. ‘Transplantation is necessary ‘The correct answer is C. Acanthoss nigricans is a benign skin condition hstologially characterized by papillomatous hypertzophy of the epidermis with hyperpigmentation. The lesions involve exural ‘regions of the body, pacticulaely the axils and groin Although the lesions ate benign, the sudden appear we of acanthosis nigricans in an elderly woman ‘should raise the suspicion of an underlying malignan- cys most commonly gastric cancer Thus extensive din- ‘eal and radiologic screening must be undertaken to rule out this possibility. Acanthesis nigricans is also associated with endocrinopathies, such as acromegaly, Cushing syndrome, hyperthyroidism, and. glcose Intolerance, Laboratory tests to Screen for such disease, including fisting glycemia (choice C), are thus appro~ Prate. Glucose intolerance in elderly patients with acanthosis nigricans is due to insulin resistance resul: ‘ng from autoantibodies to inslin-necepors ‘The correct answer is H. The clinical history alone ie highly characteristic of systemic masoytosis (urticaria Pigmentsa), which isa proliferative disorder of mast calls, The infantile type is usually confined to the skin, Whereas the adult ype affects visceral ongans ae well The most telling signs include the brown macules that urtcateon stroking or rubbing (chokce H),tchingtrg- gered by asprin or alcohol ingestion, asthma, and splenomegaly. Development of a wheal on gente stoking with «blunt objet (for example the bandle of te reflex haramer) isa utful diagnostic test. Loss of weight and splenomegaly suggest systemic involvement. ‘Thebone marrow isthe most commonly involved organ (besides the kin), and x-ray investigations will reveal ‘osteolytic oc Skin biopsies show accumulation of mast calls inthe dermis, Metachromatic staining with tlie dine blue i used for visalication of mast celle in sue Allergen skin testing (choice A) and serum IgE assays (RAST or ELISA: choice G) are used in the diognons of allergic disorders and allow identification ofthe ining agents. Neither testis indicated in the diagnosis of urticaria pigmentoss (systemic mastocytosis). Bacterial cultures of skin scrapings (cole B) are use ful in identifying bacterial pathogens in skin lesions suspected to be of infectious nature. Neither of the above conditions is due to bacterial organisms Gluten-free diet til (choice D) is diagnostic and therapeutic approach to cases of suspected dermariis heqpeiormiselie disease, Dermatitis herpetiformis manifests with pruttic vesicles on the trumk, which shows charactorstic Iga deposition in the tips of der: smal papillae, whereas ella disease is associated with turn diarrhea and malabsorption. Often, thes dis ‘orders occur in the same patient and probably result from an abnormal immunologic ection to gliadin, « protein in the gluten, HIV testing (choice E) would be unnecessary in ciher cise. HIV infetion i asiociated with sumer of skin conditions, including opportunistic infections (eg, can- Alida, herpes simplex, herpes zoster, Staphylaancus, and bacilry angiomatosis) and neoplastic conditions (eg, Kaposi sara and lymphoma). Neither acantho- ‘Ssnigrcans nor mastoeytosishas ben reported in AIDS. Microscopic examination of KOH-teatdskin scrapings (choice F allows identification of fungal organisms in skin lesions, Section Two: Qbook Practice Tests | 138 Internal Medicine: Test Four 21-year-old man i brought by his roommate to the emergency department because of abrupt onset of shortness of breath, mild chest pai, and a sensation of ‘rapid heart beating, The patient says that inthe past he had similar episodes, which resolved with the Valsalva ‘maneuver or breath holding, This time, these measures ‘were unsuccessful. He does not take any mediation tnd is otherwise in good health. An ECG documents supraventricular tachycardia with a pulse of 200/min. Under EOG monitoring, gentle massage ver the right ‘oti snus iatempted, but the attack doesnot cease Which of the following is the most appropriate next step in treatment? (A) Further ead sinus massage (B) IV tidocaine (©) W procainamide (D) W verapamil (2) Ocal verapan [A 4S.yearold African American man is taken t0 the emergency department because he is vomiting fesh blood. His temperature ie 37 € (98.6, blood pressure js 65730 man Hg, pulses 120/min, and respirations are 24/min. The patient is stabilize, then taken for emer tency endoscopy, The source of beeing isa tortuous ‘ein nea the gastroesophagea junction. The beeding is succesflly stopped by banding ofthe vesel. Which of the following is the mort likely underlying condition predisposing this patient for this complication? (A) Alcoholism (B) Alpha,-antitrypsin deficiency (©) Hemocheomatosis (D) Hepatitis A infection (E) Hepatitis 8 infection 3 ‘A S2yearold man with AIDS develops right-sided ‘weakness over the course of 1 week. He is om a con bined drug regimen of zidovudine (AZT) and a pro tease inhibitor, and his CD4 cll count is 190 cells, [MRI ofthe brain cevealsa single 2130 mm Hg) is associated with sighs and symp: toms of both cerebral damage (headache, confusion, and optic disk edema) and renal damage (proteinuria) ‘Aggressive management is required 10 avoid serious ‘complications or death and is aimed ar gradually low- ‘ering blood pressure within 1 hour. 1V infusion of sod um nitroprusside is the treatment of choice, but its ‘administration should be carefully titrated to obtain the desired effect without excessively rapid eduction in ‘blood pressure ‘Clonidine (choice A) and nifedipine (choteeF) are oral antihypertensive agents that shouldbe reserved fr les severe cases of hypertension (co-alled hypertensive turgencies) in which signs of cerebral or renal damage sare not detectable, “Enalaprilat (choice B) is the active form of enalapeil Its antihypertensive action is delayed; thus, it should be ‘used only in conjunction with other, faster-acting agents Esmolol (choice ©) isa beta blocker drug that hasbeen sel in treating hypertensive emergencies in the pres fence of myocardial ischemia, However, it should be ‘combined with some other antihypertensive age. Furosemide (chote D), as well as ther IV loop diuretics, ‘may be useful inthe presence of signs of cardiac failure ‘oF id retention, but dheir action is slow to manifest Hyétalazine (choice ) is mainly used in hypertensive crises affecting children and pregnant women. It may induce dangerous reflex tachycardia "Nitroglycerin (choice G), administered by TV infusion, is les eflectie than nitoprassde bu is useful in patients ‘with manifestations of myocardial ischemia. a, 2 ‘The correct answer is E. The polylandular deficiency syndromes are autoimmune disorders that cause sub- ‘normal functioning of several endocrine glands concur rently. This patient has the type Il vaian, which has peak incidences ge 30 and always involves the adrenal ‘cortex. Thyroid and pancreatic islet involvemeat, pro- ‘ducing type 1 diabetes melita, are also common. In this patient, adrenocoetical insufficiency is suggested by the caster of serum sodium <130 mEq/L, serum potas. sium "5 mEq/L, plasma bicarbonate <28 mBaJL, and BUN >20 mg/dl. Diabetes mellitus is suggested by the polyuria and the blood glucose of 380. mg/dL. Hypothyroidism is suggested by the elevated TSE, Patients with polylandular deficiency syndrome, type ama also present wit transient hyperthyroidism sec ‘ondary 10 destruction of follies in the thyroid gland. (Other features of the condition include antibodies lircted against the target glands, particularly against ‘cytochrome Py. adrenal corticl enzymes, and reduced systemic T-cell mediated immunity. MEN type I (choice A) is characterized by tumors of the parathyroid glands, pancreatic islet cells and psu- inary gland, MEN type IIA (choice B) is characterized by medullary farcinoma of the thycoid, pheochromocytoma, and hnyperpurthyroidise, MEN type IB (choice C) is characterized by multiple ‘mucosl neuromas, arfanoid habitus, medullary caei= ‘noma of the thyroid and pheochromocytoma Polyglanular deficiency syndrome typeI (choice D) is characterized by onset in childhood or before age 38, Ihypopersthyroidiem, adrenocortical failae, and gonadal fulure; diabeter mellitus is not usually scen ‘with tis condition, “The correct answer is B. These tears, called Mallory= Weiss Iserations, account for about 59% of cases of upper gastrointestinal hemocthage. They occur when the proximal part of the stomach is teescaped into the cist esophagus (stretching and tearing it) by severe vomiting, severe retching,o severe hiccups. The hcer- tons are usualy superficial and often stop bleeding spontancously. IF the bleeding fils to stop sponta: ‘neously the lacerations may be controled endoscopi- cally. The condition was initially described in aleo holis, bur you should be aware that it occasionally ‘cars in many ater types of patients as wel, AIDS (choiee A) can predispose for esophagitis due to Viruses (CMY, herpes) or fungi (usually Candida) Chagas disease (choice C) can cause megsesophagus 33. Seaton Two: Qbook Practice Tess “Answers and Explanations | "57 Diabetes mellitus (choice D) can predispose for fungal (particularly Candida) esophagitis. Sclrodecma (choice E) can involve the esophagus with Arosis, leading to dysphagia, ‘The correct answer is E Liver adenomas an indroquent ‘benign tumor composed of hepatocytes aanged wih the same lamellar pattern as the normal hepatic lobule It is usualy clinically lent and occurs met frequently ia association with oral contraceptives or anabolic trod. ‘When symptomatic ts most common clinical presenta tion is rupture into the peritoneal cavity with consequent Jhemoperitopeum and shock. ‘Alndominel psi (choice A) ia symptom of many hep- stobiliary disease, including hepatitis, carcinoma, and cholelithiasis. The pain of hepatic origin is presumed to result fom stretching ofthe capsule as occurs in acute hepatitis. The pain of biliary origin derives from acute distention of the gallbladder, usually because of gll- stones blocking the cystic or common bile duct, Jaundice (choice B) snot « manifestation of Tver adeno- sma. Hemolytic disorders, dfs hepatocellular damage, and obstruction of the biliary pathways are the most common cnultions resulting in jaune. Metastatic disease (choice C) is not a consequence of liver adenoma, which sa benign tumor. Liver adenoma, however, may be extremely similar histologically to ‘well-dferentited hepatocellular carcinoma, so that ‘one may be mistaken forthe other Portal hypertension (choice D) may result from pre hepatic causes (thrombosis of the portal vein), hepatic ‘causes (cirrhosis or other dfs inftrative conditions), and posthepatic causes (obstruction of the hepatic veins or right-sided cardiac failure). Well demarcated tumors, such as iver adenoma, do not lead to portal Inypertension ‘The correct answer C. The casi presentation long with the hyperuricemia, makes gouty arthritis the mest Tikely diagnosis, Nonsteroidal antiinflammatory drugs (ONSAIDs) ate the weatment of choice of acute gouty srthrtis. They are somewhat less specific than colchicine for gout but are beter tolerated and work auicky Allopurinl (choice A) i used in the inerritcl peciod Iherween attacks, not in acute attacks, Colchicine (choice B) i effective for acute gouty ahr tis but is poorly tolerated atthe high, frequent doses required for an act attack. Steroids (choice D) can be used in an acute episode, but not before a trial with NSAIDs andlor colchicine is attempted. medical 35. Section Two: Qbook Pracice Tests Intemal Maine Test Four Probenecid (choice H) is uricosuric agent sed in the intercritical period, not during acute attacks, in patients who are under excrcters of uric ai, "The correct answer is C. Wegener granulomatosis is a vasculitis that affects the upper respiratory tract and Paranasal sinuses, che kidney, andl the lange ‘lated with « dermatologic symptoms (subeutancous rodules and purple papules). Antinewtopil cytoplasmic antibodies yielding 2 cytoplasmic immanofluorescence patter (CANCA) are found in more han 90% of patents with Wegener granulomatosis, levated level of serum IgE (chotce A) might be seen in acute bronchopulmonary aspergilos Goodpasture syndrome ie associated with ant slomeruler baseraent membranes antibody (choice B). This disorder affects the kidney and lungs, rather than the lungs or paranasal sinuses, ‘A postive p-ANCA (choice D) is seen in polyarteritis ‘nodosa, a nongranulomatous vasculitis Purple nodules can be seen in both polyarterts nodosa end Wegener sranalomatosis Rheumatoid factors postive (choice Bin many patients With sheumatoid arthritis, but this marker is nonspecific and may be postive in several ther disorders “The correct answer is C. Tis sa debated and mal- nourished patent with kypernatremi. The most prob ale cause stat she eno longer able to express thirst ‘oF hunger and cannot cooperate efficiently with care- fiers when faking water of food. The most immediate toa of therapy is to replace water. The usual choice in this seting is 596 dextose in water, which should be ven relatively slowly to prevent glucosuria, followed by normal saline This strategy would increase the sl five water exctetion and bypertoncity (Oral water administration (choice A) will work with physiclly and mentally competent people who have bhoen water deprived (eg., standed in the desert) However, this strategy has obviously already filed in this patient (the nursing home was probably not deli crate trying to dehydrate her) and is not the best choice. 10.8% sane (choice B) by itself would not be used in ‘this clinical setting 1V ello and 09% sane (choice D) is sed the iii hydrating lid in patients who are in shock secondary to dehydration, but is not needed in this patient with adegjate blood pressure and symptomatic hyperna istiled water (choice F) should never be given to patents—it wil caus red ell ysis o. 38, “The correct answer is A. Tiss Behcet syndrome. The laboratory finding are usually nonspecific indications ‘ofan inflammatory proces (such as thos illustrated in the question stem), 50 the diagnosis is usualy estab- lished by the history (and may take months to years since not all features are typically present from the beginning). Characteristic features include painful orl takers, painful genital ulcers (in men, in women they smay be painless, ocular disease (most often a relapsing icidocylits that causes pain and photophobia, skin lesions (papules, pustules, vesicles, or folliculitis), and mil arthets of large joints. Other Features that may be seen include CNS involvement and migratory throm bophlebits Herpes simplex infection (choice B) can cause oral and genital ulcers but the Tzanck smear would probably be positive and arthits would be unlikely [Neissrie sonorshoeae infection (cholee C) can affect both genitals and joins, but does not usually cause scr Psoriasis (choice D) can cause both skin lesions and arthritis, but the skin lesions ae charactersially scale ‘covered plaques. Treponema pallidum infection (choice F) can cause ler (the chancre) but does no usally cause arthritis ‘The correct answer is B. This patient may have colon cance The weight loss, pallor, and anemia ae indicative ‘chronic bleed, supperted bythe stool positivity for ‘occult blood. Risk factors for colon cancer include a positive history of colon cancer or adenomatous polyps {na firtlgree relative and a personal history of adeno- ‘matous polyp. Since our suspicion of colorectal cancer js high, i would be helpfil to detest the lesion and remove it if posible, Colonoscopy provides the most efficient modality with which to achieve this gol AKUB (choice A) is wually a good modality for evlu- sting obstruction, which sa potential complication of colon cancer. This patient is not complaining of obstruction, and a KUB would not detect a soft issue "Esophagoduodenosenpy (choice C) would be done to cevalate the patient fora source of upper gastrointest- pal bleeding, suchas wlcers, varices, or gastec cancer Sigmoidoscopy (choice D) would be an effective sceening text but only 25 to 309 of colorectal cancers are detected by rigid sigmoidoscopy; the rate increases {0-40 10.65% when flexible sigmoidoscopy is use. This patient has a high pre-test probability of having col ‘rectal canoes, and since visualization of the entire ‘olon will guide further ation, sigmoidoscopy would be inadequate (Open laparotomy (chotee B) might ultimately be need ce ifthe patent undergoes colonic resection; however, 't would be premature to immediatly proceed to this option. The correct answer is A. The patient probably has carpal tannel syndrome, which i caused by compres sion ofthe median nerve sit passes through the carpal tunnel in the wrist. The symptoms illustrated in the ‘question stem are typical ab is the history of exacerba ‘ion of symptoms at night following heavy west use ‘ding the day, The test describe isthe Tinel test, which can be used to assess other superficial nerves as well Milder eases of erpal tunnel syndeome may respond 10 rest and nonsteroidal antiinflammatory drugs; more severe cases may requite surgical decompression. Cubital tunnel syndrome (choice B) compresses the ulnar nerve at the elbow, producing numbness and parestesiae ofthe ring and ite fingers Radial tunnel syndrome (choice C) involves compres son of branches of the radial nerve in the arm or fore- ‘rm, Itcausee pain ofthe back ofthe forearm and hand, sometimes with wrist drop. Refls sympathetic dystrophy (ehoice D) is pain and Jimited motion ofthe shoulder accompanied by ipsa «il involvement of the hand, Scapholonate ligament rupture (choice E) usvally ‘cen during a fall onto an outstretched hand and ‘causes pain in the mid-west. ‘The correct answer is C. This patient presents with a Jateral ankle spain, Clinical arsesment is usualy uh cient to diagnose this condition, unless there are sigas suggesting the presence of facture. The drawer and tale tess are used to asses ligament instability and confirm a diagnosis. A postive tars test indicates injury to the akaneofbular ligament, and a positive drawer test indicates injury to the anterior taloGbular ligament. The history i also very important in clarity ing the mode of trzumatic injury. Currently recom ‘mended treatment for uncomplicated ankle sprains of mild-to-moderate degree includes protection of the injured joint by splinting o: immobilizing boot; rst of the injured joint; ice aplication several times daly; ‘compression by elastic wrap, and elevation ofthe limb to reduce edema (othich gives the PRICE mnemonic) Adjuvant ant-inflammatory of analgesic treatment ‘may be used. Patients wih Intra ankle sprains should start rehabilitation ely, usually after 24-72 hours of ankle rest. Treatment for acute ankle sprain with reba Uilitaion after 10 days of ankle rest (choice D) hasbeen found to be unnecessary compared with shorter pet ‘ode of immobilization 4 Scion To: Qbook Practie Tess ‘Answers and Explanations | 159 Referral to orthopedic specialist (choice A) is rarely necessary unless there are complicating factors, such as Associated injuries to other joints or factres. Treatment with nonsteroidal ent-inflammatory drugs ‘without ankle immobilization (choice B) is definitly inappropriate in any case of ankle sprain, however mild X-ray examination (choice E) i indicated when there ae signs suggestive of fractures, such as history of a Snapping or popping sound at the time of injury, pain in the malleolar region, and tenderness on presure on the lateral malleolus ci recommended that patients ‘older than 55 yours should undergo routine x-ray eon- trol even in the absence of clinical signs of fractures because ofthe increased incidence of occu fractures in niore advanced age ‘The correct answer is B. The clinical picture is highly suggestive of diabetes insipidus, a disorder due to defi- sleney of antidireic hormone (ADH) secretion from the posterior pituitary gland. Polydipsia is 2 conse ‘quence of polyuria (not an elect) because of abundant Giuresis secondary to ADH deficiency. The vasopressin challenge teste given by administering desmopressin (a synthetic form) by the nasal route and monitoring the ‘urine output 12 hours before and 12 hours afer ad-ninis- ration. If the symptomatology is due to central diabetes insipidus, the patient wil experience an immediate rection in thirst and urine output Diabetes insipidus say manifest in the third trimester of pregnancy or daring the puerperium because ofa circulating enzyme (asoprssinae) that degrades vasopressin, The enryme is not effective against desmopresin. The disorder resolves spontancously Besides the high likelihood of noncompliance, advising the patient to reduce water intake (choice A) would expose her to severe dehydration and hypernatremia {due to lose of water and resultant hemeconcentaton. MRI studies of the pituitery-hypothalamic region (Ghoice ©) ace performed to search for mas lesions thet may be the underiying cause of central diabe wipidus. These investigations should be performed ater the diagnosis has received confirmation by a vaso rosin challenge test Compulsive water drinking may require psychiatric evaluation (choice D), and may mimic diabetes insipidus, but this possibility should be undertaken ‘once diabetes insipidus has been ruled out. ln this par ticular case, the close temporal association with preg nancy and delivery makes the diagnosis of diabetes insipidus more likely 160 2. Scion Two: Qbook Prats Tests Internal Medicine Test Four Evaluation of nephrogenic diabetes insipidus (choice E) is the next appropeate step ifthe patient does not respond to vasopressin challenge test. Nephrogenic dis ‘ates nsipidus in fc, i ue to resistance of renal tubules to the action of vasopressin. A hereditary X-linked form is known, usually associated with hyperuricemia, ‘Acquied forays ace sssocited witha variety of cond tions, eg. pyelonephritis, multiple myeloma, and chron: ic hypercalcemia, ‘The correct answer is C. Acute real failure (ARE) isa ‘vommon occurrence in hospitalized patients, Iis crue ‘ial to determine whether ARF is secondary to prerenal for renal causes. nthe former case, the underlying cause i reduction of blood flow to. normal kidney resulting in decreased glomerular filtration rate. in {he late, intrinse renal damage isthe underlying et- ology. OF the intrinsic causes of ARE, acwe tubular rnecrosisis the most common. Hove can one distinguish between prerenal and renal forms of ARF? Sodium reabsorption is not impaired in prerenal ezotemi whereas creatinine reabsorption is deficient in both prerenal and intrinsic renal zotemis, rational exre- tion of sodium (FE) i therefore a most useful parameter to distinguish between these two condi Sons. I can be calculated by the following formula: Nauanve x Crtasse/Crynans x Napiassin, % 100. In prerenal azotemia, Fy, <1% because the undam~ aged renal tubules will avidly absorb sodium. In acute tubulat necrosis 2s well a othe intrinsic renal causes of szotemia, damaged cubules will allow sodium to leak into urine, and FE, willbe >1%. In this particu lar cose, thecalcultion yields FEy, = 1. Gentamicin is the most nephrotoxic ofthe aminoglycoside antibt tics Its toxicity usually manifests after 5-7 days of treatment. ‘The mos frequent manifestation of allopurinol toxicity (choice A) is 2 pruttic rash due to hypersensitivity. Vasculitis and hepatitis are other, albeit rae, adverse cffets Ampicilin toxicity (choice B) may manifest with renal damage, but usually in the form of acute intrtil nephritis, not acate tubular necrosis. Acute interstitial nephritis is associated with fever, ath, blood easinophil- ia,and leloeyturia with eosinophil Hyperuricemia (choice D) may cause acute tubular necrosis, bot only when serum uric acid levels rise rapidly as a vsult of rapid call turnover. This may develop with hematologic malignancies or germ cell neoplasms treated with chemotherapy Uric acid levels arc oten very high (220 mgyAl) lical a Hypotension (choice Eis the underying pathogenetic {actor of most conditions lading to prerenal azotemia, such as hypovolemic, cardiogenic, or anaphylactic shock, Renal subular function is preserved, bt the fll in glomeraar filtration rate result in oliguria/anuria and ARE Fy, is very low (<1). Clnial history is ‘obviously an important aid in the ferential diagnosis ‘benseen prerenal and renal szotemia Sepsis (choice F) is a frequent cause of prerenal szotemia when associated with shock. kn this cas, the laboratory data rule out this possibility, The correct answer i B. The presentation is clase for central retinal artery ocluson, whichis a blockage of ‘the central retinal artery by embolism oe thrombosis that ceuses pinks, sudden, unilateral blindness, Patients may have underving atherosclerosis, endocanlits, or temporal ertrts The retinal changes iusrated in the question stern are typical and are the esl of failure of blood to flow into the retina. Immediate treatment is imperative Intermittent digital massage over the closed ‘eyelids may dislodge the embolus and allow it to flow into a smaller blood vessel, where twill cause @ smal area of retinal ischemia, If this fl antrior chamber paracentesis wil alsa sometimes dislodge the embolus ‘Age-related macular degeneration (choice A) can also present with sudden, painless, unilateral blindness, but ‘the fundus wil show pigmentary changes with or with ‘out new vese formation bhind the retina ‘Central retinal vein oclusion (ehoice © can also cause painless, unilateral blindness. However, it tends to develop a litle more slowly than central artery oclu sion, and the retinal vesicle appear congested Hypertensive retinopathy (choice D) does not usualy cause sade blindness Retinitis pigmentosa (choice E) develops over a period of years with loss of peripheral vision, ‘The correct answerisE. This patent has mixed connec tive issue disease. Clinically, it appears to be an oveap ‘syndrome with features similar to rheumatoid aris, ‘systemic lupus erythematosus, sleroderms, Siren yn «drome, and polymyositis or dermatomyositis. Iti now considered a separate diseae because of a distinctive autoantibody pattern, with very high levels of satibody rected against rbonudleoprotein (RNP). Rheumatoid ageltinins may also be presen in high titers. Other ant bodies characteristic of the individual discases tht mixed connective dssue disease mimics are usually absent or present ony in ow titers. The clinical presen- tation maybe wildly diverse, depending on which disease pttern manifest fis. Wath tir, however, there should be clinical features suggestive of a variety of diferent 6 autoimmune diseases. Mild disease may be controled ‘wih measures similar to mld sheumatoi arthritis (sali- ‘fits, other NSAIDS, antimalarials, very low dose cor ‘icoteroids): more severe disease (which may be fatal with complications due to vascular lesions, renal lure, ryocardalinfrcsion, diseminated infection, or cee- bral emorshoge) usualy roquires large dose steroids. Associate “ANCA (cholee A) with Wegener granulo Associate dsDNA (choice B) with systemic lupus erythematosus, Associate p-ANCA (choice C) with microscopic polyarerts ‘Associate SCL70 (choice D) with systemic sclerosis, ‘The correct answer is A. Cigarette smoking is consd- cred the principal preventable cause of disease in the Us. and approximately 20% of adults in this country smoke. The guidlines of the US. Agency for Health (Care Policy and Research (AHCER) and the National Cancer isttate recommend that ll primay care phys: cans identify smokers and advise them o quit smoking Sever stds have found tht 10% of all smoke wll uit smoking if they ceceve even 2 3-minute advice 19 ‘uit fom a cinician, However, the very ist step in the ‘Physicians approach tos smoker is to establish hisber ‘willingness quit With respect to smoking cessation, three stages have been identified: prcontomplaion, it ‘there sno intention of quitting; contemplation, if there are some ideas but no clear plans of quitting soon; and ‘ction,f theresa definite intention of setting a quitting ‘ate, The next steps depend on the patient’ response “eying to elicit a quiting date even if the patient appears _unwilig (choice B) would be inappropriate. The pysi- an should elicit a quitting date ifthe patient i already willing t0 do so, ie he‘she is inthe contemplation or action stage. The physician should discus the reasons or ‘obstacles that make a patent unvsling to stop i he/she is in the precontemplation stage. Information about teks of smoking should be provided. Prescribing nicotine patch and gum (choice C) is most useful in smokers wo are nicotine dependent. Several ‘questionnaires have been developed to establish nicotine dependence (the Fagesirom questionnaire is probably the ost widely used). Nicotine dependence s present if 4 patient smokes the fist cgarete within 30 minutes after awakening, smokes more than 20 cigarettes daily, Finds dificult to refrain fom smoking in plaes where making i prohibited, and continues to smoke even ‘during ness. Determining nicotine dependence allows 4 physician to individualze treatment stratepes by pre- scribing note replacement therapy. ”. Section Two: Qhook Practie Tess “Answers and Explanations | Group behavior treatment (eheice D) is mostly indicat- efor smokers who have already tried and failed to quit inthe past. Recent studies have shown that eombina- tions of intensive group counseling with nicotine replacement have he highest succes rate (up to 409%) in obtaining long-term abstinence. Maximal specialized care with intervention ofa psychi- atry specials (chotee E) may be indicated for patients with a high degre of nicotine dependence associated ‘with other forms of drug abuse alcohol being the most frequent). ‘The correct answer is B. The most probable etiology of bacterial endocarditis involving the tricuspid valve is ict TV drug se, which can intzoduce skin organisms into the venous systema that then proceed to attack the tricuspid vahe. Staphylococcus aureus accounts for between 60 and 90% of eases of endacazdtisin TV rug ‘The endocarditis associated with congenital heart dis- cae (choice A) typically involves ether damaged valves for aval or ventricular septal defects The tricuspid valve isnot particulary valnerable Rheumatic fever (choice C) most commonly damages the mitral end aortic vale, and tricuspid damage is usually less severe and seen only when the mitral and toric valves are heavily involved. Consequently, sec- ‘ondary bacterial endocarditis ivolving only the ticus- pid valve in a patient with a history of sheumatic ever ‘would be unusual Rheumatoid arthritis (ehotee D) isnot associated with bacterial endocarditis, Systemic lupus erythematosus (chofe H) can preduce small aseptic vegetations on vales(Libman-Sacks endo carditis) burs nor asociated with bacterial endocat ‘The correct answer Is A. The clinical presentation is highly characteristic of epidural bleeding, which usually is of traumatic origin and most often results From rup- ture ofthe middle meningeal artery, The intl coneus sion leads toa brie los of consciousness, which is fol loved by a lucid interval lasting several hours. As the epidural hematoma progressively enlarges and pushes the underying brain, he patient becomes comatose again and may display sign of unal herniation, The Feriating uncus pushes onthe third ceaial nerve, pro acing pilateral fixed poplar dilatation Pepilledema 4s usoaly alate sign and indicates cerebral edema. A ealvatal acute (choice B) would not cause such a severe neurologic state and evidence of uncal herniation unless associated with an intracranial hematoma. ‘Seaton Twa: Cook Pracie Tests 162 sternal Medicine Test Four Intracerebral hypertensive hemorthage cholee C) ws ally occurs spontaneously and develops within the brain parenchyma (most commonly in the baal gan- slit). Lose of consciousness develops in approximately 50% of patients, Headache, vomiting, and variable net rologic deficits are present. ‘Subarachnoid hemorthage (choice D) characteristically manifests with sudden onset (thunderclgp) of hevdache associated with vomiting and progressive impairment ‘of consciousness. The most frequent causeisraprute of bercy aneurysms. ‘The linical manifestations of a subdural hemor (choice E) may vary depending onthe severity and location. Cerebral atrophy i a predisposing condition, asit leads to stretching” of bridging veins that connect the veins on the cerebral convextes with the superior saital sinus. Minimal trauma may then result in tear- ing of such veins Impaired consciousness andor focal neurologic des follow the traumatic event after an interval of days or weeks. 48. Thecorrect answers A. Anticoagulation with heparin fs the sandand therapeutic fist step for pulmonary cmbols. Thrombolytic agents (choices B and Esl hae avery limited role inthe treatment of pulmonary embolas “Thay may be considered when seeheling gh sided tert fare i life threatening, but beeing reais a formidable potential complication; therefor, they ate ‘containdicatd afer major surgery. When they ae used, ‘systemic adminsvation hake) sas efecve as diet Glatamyt transferase (©) Mean corpuscular ed blood cel volume (D) Serum triglycerides (8) Uricacid ‘A 38-year-old woman with rheumatoid arthritis pre ‘sents to hee physician because of increased joint pain. ‘On physical examination, both passive and active range of motion of the hips are decreased. Her physician increases her dose of nonsteroidal antiinflammatory «drugs (NSAIDS). Which ofthefllowing shouldbe pre- scribed as welt prevent peptic ulcer disease? (A) Cimetidine (B) Clarithromycin (C) Misoprostot (0) Omeprazote (B) Svcrafate ‘A 22-year-old college student has had a nonproductive ‘cough low-grade fever and severe headache for 6 days, aswell set ear pain for | day. His temperature is 383 © (101 P), pulses 7/min, and respirations are 18/min, The left tympanic membrane is erythematous, and there are 90 small blebs present. Crackles are heard cover the right lower lung field. The le lung fle ace ‘lear to auscultation, Laboratory studies reveal a white ‘blood cel count of 6000 with anormal differential. A est xray film reveals right lower lobe inflate with ‘an area of platelke atelectasis. When an anticoagulated tube of blood is cooled wit ie, a precipitate forms, clearing with rewarming. Which of the following is the ‘most appropriate treatment? (4) Amosiclin (8) Bythromycin (©) Gentarycin ©) imipenem Vancomycin ‘A.40-year-old man presents to a physician because he has been experiencing episodes of severe vertigo accompanied by nausea and vomiting, The frst time thishappened, he thought he had picked up gastroin- testinal "bug" but he has now had five of thes episodes ‘over the past 6 months. The episodes frequent begin ‘with a sense of fullness in his ight ea, whieh is offen accompanied by tinnitos and a sense of earing loss in the affected ear Each episode lass hours to days and then resolves, Otoscopic examination of the affered ‘at is within normal limits. Which of the folowing is the most likely diagnosis? (A) Benign pazoxysmal positional vertigo (8) Herpes zoster otis (©) Meniere disease (D) Parulent labyrinthitis (8) Vestibular neuronits A 37-year-old woman complains of pain during inter- ‘course Several months ag, she noted the gradual onset ‘oF increasing vaginal discomfort, ané she snow unable to ave intercourse without significant pain, She uses ‘oral contraceptives and doesnot use condoms or Iubr- ‘ants. During the interview, the patient stops several times to drink fom 3 botle of water that she cates eth ther She reports that she has alwayshad a dry mouth and dy eyes On physica examination, her temperature is 37.2C (989 P), blood pressure is 11082 mm Ti, pulbe {is 74/min, and respirations are 14/min, Thee i iaterl parotid gland swelling, Thor are musiple dental caries And filings. Cardiac examination is significant for a rmidsytlic click. Which ofthe followings the most ike ly dingnosist (A) CREST syndrome (8) Dermatomyositis (©) Raynaud phenomenon (D) Scleroderma Sjigren syndrome Section Two: Qbook Price Tess Internal Medicine Test Fue | 17" 26, A 35-yearold man consults & gastoenterologist, because of chronic heartburn for several years. The heartburn tends to be worse at night and he frequent Iy tastes reflued gastric contents when he goes to bed. $e found that bis symptoms were a ite beter when hae avoided his customary late evening alcoholic drink; however, this modest improvement has subsequently deteriorated, "The gastrocnterologist performs ‘esophageal manometry with pH monitoring, which Aemonstrates decreased pressure of the lower ‘esophageal sphincter and the presence of acid in the fsophagus. Eeophagogastradvodenoseopy demon. ‘rates a very eregular gastroesophageal junction vith long “Binges” of reddened mucosa extending up to 7 ‘em abote the lower esophageal sphincter. Biopsy of the proximal end of one of these fingers shows surface epithelium with regular columnar cells with smal, ‘ovoid nuclei admixed with goblet cel, Which ofthe following i the most key diagnosis? (A) Achalasia (B) Bareet esophagus (©) Corrosive esophagitis (D) Esophageal adenocarcinoma (8) Zenker diverticulum 27, Apreviouly healthy 22-year-old man comes to medic tention becsuse of progressive exertional dyspnea punctuated by episodes of precordial pan. The patient’ father and one of his elder siblings die of chronic heart disease in middle age. The most sguificant findings on ‘Physical examination include presence of aloud S, ana harsh systolic murmur The later increases with the Valera maneuver and decreases with squating. Bas rales are heard on lung auscultation. Blood pressure, pls, and respiratory rate are within normal limits. A chest xy fim show no appreciable alterations in car diac shove, but the ECG shows left acs deviation Echocardiography reveals marked thickening. of the imerventricular septum, atociated with delayed relax ation and Gling of the left ventricle duving diastole ‘Which ofthe following the most appropriate inital tep in management? (A) Beta blockers (B) Digiatis (©) Long-acting nitrates (D) Aortic valve replacement (6) Partial excision of myocardial septum Hitieal m », Section Two: Obook Practice Tests Internal Medicine Test Five A.47year-oldsleoholc man is picked wp by the patie and taken to the emergeney department. The police, ‘oho have had previous encounters with this man, are ‘concerned that he seems particularly lethargic and con fused. Stat blood chemistries demonstrate 4 plasma sodium of 115 mba/L. The emergency department physician orders the administration of hypertonic tales within 4 hours, the plasma sodium rises to 135, imby/L. Over the next few days, the patent develops ‘quadriparesis and weakness of the lower face and tongue. These symptoms never resolve. Damage to which of the following neural structures probably sccounts for these findings? (A) Cerebellum (8) Corel cortex (©) Peviphora nerves (D) Pons (8) Spinal cord ‘A 61-year-old wornan presents to hee physicians office fora routine physica. Sb has a history of diabetes and bbypertension and has a 30-pack-year history of cigarette smoking, Laboratory studies show Sodium 136 mEq Potassium 45 mb Chloride 108 mB. BON 2A ml Creatinine 09 myal aliom U5 mela Albumin sa gla ‘Which of the following isthe most likly diagnosist (A) cess vitamin D intake (B) Occult malignancy (©) Paget disease (D) Parathyroid hormone (PTH) oversecretion ) Sarcoidosis 20. a A35-yoarold man with recurrent, active, genital herpes suddenly develope numerous erythematous macules, ‘papules, wheals, snd vesicles The hands, fet, and face are most extensively invohed. Careful examination ‘demonstrates that many ofthe lesion’ havea “target” Appearance. Some ofthe lesions involve the lips and ‘haccal macoss. Which ofthe following the most ike- ly diagnosis? (A) Exythema multiforme (B) Exythema nodosum (€) Granvloma anmulare (D) Pemphigus (6) Toxic epidermal necrolysis ‘60-year-old woman comes othe physician because of jaundice, pruritus, and anorexia for 2 weeks, he has one ‘two alcoholic drinks on social occasions and has smoked one pick of cigarettes daly for 30 years. She is ‘currently taking a thiazide diuretic for mild byperten sion. Her temperature is 36.8 C (98 F, blood pressure is 150/80 mm Hg, pulse is S/min, and respirations are min. Physical examination contims ieteric disco oration of skin and macosse. Absominal examination is remarkable for sight tenderness in the right upper ‘quadrant, and the iver is palpable 1 em below the right ‘costal ach. The spleen is not palpable, Serum chemistry tests show: ast oun. ALT sour, Alkaline phosphatase 1000 Ur. “Tal bilirubin 55 mg/dl. Direc bilirubin 40 mg/d. Which of the following is sep in dingnosist (A) Abdominal ultrasour (8) Abdominal CT or MRI scan (©) Endoscopic retrograde cholangiopancceatography (D) Percutaneous liver biopsy (&) Percutencous transhepatic cholangiography he most appropriate next 3 3 ‘AA5-year-old woman presents with recent ans of low back pain for the past 3 days She has not sustained any significant trauma. She basa bistoy of systemic lupus exythematostis and has been receiving cheonic cortico- eroid treatment. She is currently afebrile. [Examination reveals tenderness on palpation of the lumbar spine. The pain docs not radiate down the leg ‘Which of the following isthe most likely diagnosis? (A) Ankylosing spondylitis (8) Cauda equine syndrome (©) Compression fractre (D) Herniated intervertebral disk (E) Infection (F) Neoplasm () Spinal stenosis 1 40-year-old man with a history of 1V drug use pre Sens with 2 2-week history of high-grade fever. He has been seen in clinic before and is now found to have & ‘new murmur on cardiac examination. He also has se cra dack painful spots on his fet. Which ofthe fellow ing isthe most common cause of negative blood cul- tues in patients with this lines? (A) Fungal infetion (1) Inadequate culture techniques (©) Pei administration of antibiatics (D) Prosthetic vale seeding (2) Right-sided endocarditis ‘A A5.yeanold woman is taken to the emergency department with severe colicky right upper quadrant pain for the past 2 days. She is on medications and ‘Senies sleohol se. Ultrasound studies show gallstones, land the patient is taken for choleestecomy the next ‘morning. Intraoperative, its noted tat the liver has yellowish coloe and liver wedge biopsy is submitted slong with the gallbladder to pathology The patholo. ‘gst reports steutosis with a predominately macrovesc: ular pater inthe Liver. Which ofthe following i the ‘most likely explanation for thes findings? (A) Breast cancer (8) Chronic pancreatitis (©) Diabetes melitus (D) Peptic ulcer disease () Sjstemic lupus erythematosus 6. Sdn Two: Qbook Practie Tess Internal Medicine Tet Fue | 178 [A 52-year-old woman consults a physician because of severe epigastric pain of several months’ duration. Endoscopy demonstrates thickened gastric mucosa ‘with several peptic ulcers in the stomach and duode= ‘nur, The batl acid secretin rat is 75% ofthat fo lowing a mavimal stimulating dose of histamine Serum gastrin levels are markedly elevated (1200 pg/ml). Endoscopic ultrasound demonstrates & pan czeatic mass which is later resected. Following the resection, serum gastrin levels decrease to 50 p/m La addition to the tumor that was resected, this patent ‘would most likely develop which ofthe following di ing her lifetime? (A) Macfinoidl habitus (8) Medullary carcinoms of the thysoid (©) Mucosal neuromas (D) Pheodhromocytoma (©) Pituitary adenoma {A confused elderly man is taken to the emergency ‘department. The patient was found wandering and ‘eompaining of severe abdominal pain, but is unable to give coherent history. Urine dipstick demonstrates Positive Ketones but no glucosuria, Stat blood hemistries show a plasma glucose of 100. mgd. Which ofthe folowing isthe mest Hkely explanation far these findings? (a) Alcoholism (8) Congestive heart flure (©) Emphysema (D) Inflammatory bowel disease (©) Rheumatoid arthritis 1m a. 38. Section Two: Obook Practice Tests Internal Medicine Test Five A.70-year-old woman with metastatic hung cancer is brought to the hospital for increasing confusion and btundation ever the past several weoks. Her family denies recent head teeuma, The woman is currently receiving chemotherapy for her malignancy, but 20 ‘other medications. On physical examination, her vital sins ae stable er jugular venous pressure (JVP) is 7 fem HO, and her lungs are clear. Her heart rhythm i regulat and she has no edema inher legs. Laboratory analysis shows: Sodium 124 meg Potassium 433 mEq Chloride 109 mEq Bicarbonate 25 mEq/L ACT scan of her head indicates no brain metastases. What isthe most appropriate next step in manage- (A) Administer diuretics and water (B) Administer isotonic seine (©) Administer water (D) Restrict intake of salt (2) Restrict intake of water [An ISyear-ld female athlete undergoes succes arthroscopic repair of a torn right medial meniscus She has been armbulating with the aid of crutches and now ‘complains of profound weakness of hee right arm. Physical examination i normal except for an inability t0 actively extend the right arm at the bow or the right Dhan atthe wrist The muscle ton, reflexes flexors, and intrinsic hand muscles are normal. Theres minor sen- sory deficit over the dorsolateral area of the righthand, ‘Which ofthe followings the most key diagnosis? AA) Axillary nerve palsy (B) Dorsal scapular necve palsy (©) Median nerve palsy (D) Radial nerve palsy (E) Ulnar nevve palsy 2. ‘A.45year-ld woman presets witha yellowish discal tration of her body, fst noted by her husband last ‘week, Since then, she as been having severe itehing at night, which disturbs her sleep, and complains of atin sling sensation of her hands and fet. On examination, anthelasmat are seen around the eyes. The liver ie firmly palpable 4 cm below the costal margin. Scratch marks are noted om her abdomen and limbs. Clubbing is observed in all the digits. Serum creatinine is 03 mit, bilirubin is2.3 mg, albumin 43 gf al nine aminotransferase is 92 UIL, and alkaline phos phatase is 410 U/L. Which ofthe following is the most appropriate next step in diagnosis? (A) Anti-mitochondrial antibody assay (8) Anti-smooth muscle antibody assay (©) Endoscopic retrograde cholangiopanereatography ceRce) (D) Serum protein electrophoresis (©) Technetium (Te) liver spleen san 4 ‘A.AL-year-old maa with alcoholic circhosi is admsited to the hospital for evaluation of increasing abdominal rth. The patent his a3-yeat history of cirhosis de to 20 yeas of ethanol consumption, He has no known car dite disease but has had two upper gastrointestinal bleeds secondary to portal hypertension from his cit- thos, He underwent variceal banding on both ooce ‘sions He reports that sine his as admission for ascites months ago, he has continued to rink ethanol but has ‘been taking his daily spionolactone and furosemide On physical examination, his abdomen is distended with occasional spider angiomata and a uid wave on palpation. A diagnostic paracentesis is planned. Monitoring which ofthe following laboratory values is ‘mort important in the continued cate of this patient? (A) Bleeding time (B) Hematocrit (©) Paral thromboplastin time (PTT) (D) Platelet count (8) Prothrombin time (FT) A S3.year-old man collapses while in the checkout Kine athe supermarket. A bystander administers CPR until fn ambulance arrives after approximately 10 minutes. Daring the ride to the hospital, the pulse slot rice, snd further resuscitation and dlectric deflation are ‘eauired. At the emergency department, he hasa steady sinus Hhythm and normal, table vital signs but he isin seep coma, with bilateral fixed, dated pupils His past redial and surgical history are unknown, Which of the fllowing might ultimately improve his neurologic couteome? (A) Anticoagulation with heparin drip (B) High-dose systemic steroids (dexamethasone) (C)Hyperventilaton toa PCO, of 25 mm Hg {D) Moderate hypothermia to 33.0 C (914 F) core temperature (8) Sedation with barbicurates 2. ®. Section Tao: Ghook Practice Tests Internal Medicine Test Five | "75 A28-year-old man comes to medical attention because ‘of sustained hypertension fr the past year. During the sume perio, he also had paraxysmal episodes charac- tered by profuse sweating, headache, and a sensation of increase! heat rat. At thie time his blood pressure is 160/95 mm Hg, with a pulse of &6imin on supine positon. On standing his blood pressure is 120/70 mm Higand pulse is 10min, A dipstick test reveals glucose in the urine. Which of the following isthe most appro- rite next step in diagnosis? (A) CT andlor MRI studies ofthe abdomen (B) Measurement of epinephrine and norepinephrine in the blood (©) Measurement of fisting glucose levels in plasma or (D) Phareacologic provocative tests () Urinary aisay for fee catecholamines and vanilly- mandelie acid |A27-year-okd man is brought to the hospital with a gun shot wound of the abdomen, and he is prepared quickly for emergency laparotomy, The endotracheal intubation is achieved with the help of suecinycholine, and be is then switched to nondepolaizing agents and inhaled halothane. Shortly thereafter, the anesthesiologist noties tachycardia, hypertension, and increased CO; production; subsequently the core ody temperature Fises to 406 C (1050 F) and hypeckalemia develops. ‘Which ofthe following isthe intrinsic pathophysiology responsible for these abnormalities? (A) Bacteremia (8) Massive release of eatechol (©) Masive release of thyroid hormone (D) Sustained muscular contracture (B) Unrecognized adrenal insuficiency inédical | Sein Two: Qbook Practice Tests 176 | intemal Medicine Test Five ‘The response options for items 44-45 are the same. You wil be required to select one answer foreach item inthe se. (A) Aneie dissection (H)_ Microscopic polyngits (i) Atherosclerotic disease ()Polyarteritis nodose (©) Churg-Struss syndrome (D)Sophilitc aortiis (D)_Cryoglobutinemic vasculitis (K)_Takayas arteritis (8) Giant cell arteritis (1) Thromboangits obliterans (F)_Henoch-Schanlein purpura (M0) Wegener granulomatosis (G)_ Kawasski syndrome or each patent with signs and symptoms of peripheral vascular disease selec the mos kay diagaosis 44, A 25-year-old woman presents with 2 2-month history of recurrent transient episodes of visual blurring, dizines, and focal neurologic deficits, sach as limb weakness and paresthesias. She also reports sumbness and coldness of her fingers Physical examination discloses marked weakening ofthe pulses in the upper extremities. An aortic angiogram reveals pax {mal narroseing ofthe brachiocephalic, let common carotid, a let stbcavian arteries, The aortic arch appears normal 45, A:34-year-old woman presents with a 6-month history of sinusitis and ofits media unresponsive te common antibiotic and ant-inflammatory treatments She recently developed increasing malsise and a persistent cough productive of blood tinged sputum, Her temperature is 38.2 C (100.8 F), blood pressure is 126/81 mm Hg, pus is 80/min, and respirations are 16/min, Laboratory studies show mild anemia and leukocytosis an erythrocyte sedimentation cate (ESR) of B7/miny and the presence of circulating antineutrophil eytoplasmic antibodies of ANCA type Urinalysis shoves more than 5 ed blood callethigh-power Geld and red cll casts Section Two: Obook Practice Tests Internal Medicine Tet Five | "77 ‘The response options fr items 46-48 are the same You will be required to select one answer foreach tem inthe st. (A) Allergic contact dermatitis (8) Nummolar eczema (8) Atopic dermatitis ()Proriasis (©) Hypottyroiaism (D) Seabies (D)_Iramunodeiciency state () Sebortheic dermatitis (@) Intent contact dermatitis @) Tinea corporis (8) Langerhans cell histiocytosis (M0) Vitamin deficiency (G)_ Lupus erythematosus For each patent with arash selec the most key diagnosis 46, A2-yearold boy is brought by his parents to the physician ecanse ofan itchy rash that hasbeen a persistent problem since ‘6 months of age. The rash is dry, erythematous, and scaly, and chiefly involves the fce and dorsal surfaces of ands and feet The lesions undergo alternating periods of emission and exacerbations. Local steroid treatment has provided tr porary reli. Examination also reveals dy skin and pronounced skin markings on plantar and palmar surfaces. Family his tory i significant for allergic rhinitis and asthe in several relatives. The child's development is otherwise normal 47. A.50-year-old man comes to his physician because ofa 2-week history of pruritic rash onthe extensor surfaces of legs. ‘ums. The patient reports no family or personal history of allergic disorders. The rash is characterized by erythematous, ‘rusted, coin-shaped plagues. Skin scrapings treated with KOH do not revel any spores or hyphae on microscopic exam- ination, Physical examination is otherwise ncemarkable. 48. A.35-year-old man with AIDS develope a scaling, ily erythema involving the scalp, eyebrows, nasolabial folds ack, chest, and umbilicus. | Secion Two: Obook Practice Tests 178 | intemal Medicine Test Five ‘The response options fo items 49.50 are the same. You will be required to select one answer foreach in the set. (A) Chest x-ray film (©) Plasma lipids (B) Dexamethasone suppression test (F) Plasma renin level (©) Growth hormone glucose suppression test (©) Thyroid function tests (D) Measurement of urinary eatecholamines (H) eo-dimensionalechocardiogeam For each patent with hypertension select an appropriate initial diagnostic test 49, 50-year-old man comes tothe physician fora health maintenance examination. On this ecasion, and onto subsequent vst, his blood pressure is found tobe elevated. On the ast visit his blood pressure i 145/94 mm Hig. He denies any si nificant medical problems, but his family history i significant for high blood pressun and diabetes mellitus, Physical examination reveals no abnormalities. + premature coronary artery disease, 50, A 30-year-old woman comes to the physician fora health maintenance examination. Her Blood pressure is 145/68 mm He, Her pulse is ieregulay, with an average eae of 100/min, She admits having increasing anxiety, leplesness and palpitations in the past 3 months. She also had a4-hg (8.8 1b) weightloss over the same period, Examination reveals fine tremors ofthe hands, most skin, and hyperrflesia. medical Section Two: Qbook Practice Tests | 179 Internal Medicine Test Five: Answers and 26. 2. 28 2. 30. aL 32. 3. 34 3 36. a. 38. 40. a 2. 6. “4 46. ”. 4 ». 50. cme mooceecoee Oe R me EZR om om Explanations Secon Tn: Qbook Practice Tests | intemal Medicine Test Five “The correct answer is D, Characteristics of infectious ‘mononucleosis include sgos and symptom similar to influenza with lymphadenopathy (especially in the cer ‘vical chain) and splenomegaly Skin rsh is infrequent, ‘but ampiciin administration is followed by a difuse retlopupulsr rah in 90% of cases. Atypical Iympho- cytes are easily identified on blood smears. Granuloeytopena fe present intially, and thrombocy- topenia develops frequent. IgM antibodies to Epstein Barr virus appear during the acute phase. The Monospot testis based on heteophil (sheep cll gglu- tinaton) antibody tests and becomes positive before the th week aftr the onset ofthe disease ‘Acute cytomegalovirus (CMV) disease (choice A) may be indistinguishable in its clinical an laboratory man- ifestaions fom infectious mononucleosis, but the het crophil antibody test is negative in CMV infection. ‘Most CMV infections, however, rem asymptomatic in immunocompetent hosts, producing severe dissemi- hated infections in immunocompromised patients, Acute lymphocytic leukemia (choice B) is a disease of childsen (peak between 3 and? years) and is character- Jed by pancytopenia and the presence of circulating blasts. Drug-induced thrombocytopenia (choice C) is not associated with other hematologic zbnormalities ors temic illness, Currently most cases are due to heparin. ‘Other common drugs that may cause thrombocytope- nia include sulfonamides, tharkes, and cimetidine. Sureptococcal pharyngitis with a leokemoid reaction {choice B) would be associated with marked ‘rophilic leukocytosis with the presence of numerous drculating granulocytic precurios. Leukemoid reac tions may ive rise 10 leukocyte counts up © 50,000/mm’, but are not associated with the presence ‘of blasts or darombocytopenia ‘The correct answer is D. The proper placement ofthe needle is atthe superior edge ofthe seventh eb between the tip of the scapula and the posterior ailary Hine (rather than the midclavicular line, choice C). The superior border ofthe rib is used to avoid the neu rovascular bundle, which runs along the inferior edge ‘of each tb, Before the tap is done, a lateral decubius fil should be pefatmed to contirm thatthe effusion Js fee flowing. The lang shouldbe percussed, and the cede should be placed below the point where dui is etected. This usually occurs tthe seventh rb, At east, 200 mL of fluid should be present ro be successfully {upped (otherwise it should be done with ultrasound guidance) Insertion at the inferior edge (choices A and B) ca. result in damage o the neurovascular bundle, A. The diagnosis is chronic hrm= ).CLLisa neoplastic dscuder of B lymphocytes, characterized by marked peripheral lymphocytosis. Circulating lymphocytes are extremely similar to normal lymphocytes and tend to accumulate progressively in the marrow and blood because of inactivation of the apoptoss-inhibting Bel-2 gene. The manifestations are due to increasing mamunosop- pression, bone matrow replacement, and organ infltra ‘ion. Lymphocytosis due to CLL is often incidentally discovered in otherwise healthy older people Lymphadenopathy, however is requendy present. CLL follows an indolent course nd aggressive chemother py seems to have litle impact on survival. Inpatients who present with only’ lymphocytosis and lym ‘Phadenopathy (stage I acording t the Rai system), no treatment is necessary Chlorambucil (choice B) isthe standard initial teat ‘ment for symptomatic CLL, ic. patients with progres sive fatigue and organomegaly (stage I) severe anemia (tage IID), and. thrombocytopenia (stage IV) Chorambel is well oeraed Fludazabine (choice C) is usually sed asa second-line teeatment for patients who do not respond any longer to chlorambucil. It is associated with significant immunosappresson Prednisone (choice D is useful for certain autoimmune- mediated manifestations of CLL, namely autoimmune hemolytic anemia. or thrombocytopenia Bone marrow transplantation (choice B) is used only for the rare young patient who presents with an aggresive form of CLL It is nota weatmet option for most CLL patients These are erly persons who mos often wil die of causes untlated to CLL. The correct answers F. This typical presentation of. type I diabetes melita. Patients ae typically diagnosed in childhood or adolescence, and this form of diabetes te the most prevalent type in patients younger than 30, ‘Type 1 ditbcice ae particularly prone to develop cit- etic ketoacidosis, as they produce litle or no insulin. In ‘Gavcasian patients, there fa strong association between type L diabetes melts and the specific HLA phenotypes HLA-DR3, HLA-DRé, and the beterozygote form HLA- DRS/HLA-DRE. These patients tend to have detectable serum islet cell cytoplasmic antibodies or antibodies to ‘lutamic acid decarboxylase and to insulin, I is thought that thee ciabetes melts tthe result ofan iamne- ‘mediated selective destruction ofthe islet beta cls that. usualy secrete insulin, Pancreatic biopsies in these patents (usually done in esarch rather than clinical se tings) show a dense lymphocytic inflrat in pancreatic islets, with TT lymphocytes, B lymphocytes, macro Pages, and a loss of most bea cel HLA-DRI and HLA-DR2 (choices Ato D) are not aso ciated with an increased incidence of type | dabetesand tr actually clatively "protective" HLA types, since most type | abetic instead ave HLA-DR of HLA-DRA. “The correct answers B. Te disgnosis of acobolichep- sti as enough social implications tha itis important ta be reasonably sure itis correct before suggesting it. (One ofthe more helpful ules of thumb in making the iagnosis i that the ratio of serum aspartate amino- ttansferae (AST) to serum alanine aminotransferase (ALT) is usually greater than 2 in alobolic hepatitis. (Other indices are affected by alcoholic hepatitis bu are ot as specific forthe disease ‘ery high ALT levels (choice A) ate more characteristic of viral hepatitis. In aleoholic hepatitis, elevation to ‘round 250 U/L is more rypical “The gamma-glutamyl transferase (GGT) level (choice (© provides another helpful ce to alcoholic hepa in that iti often markedly elevated in these patients. ‘Normal levels for males acles than or equal 065 U/L; for females, ss than or equal o 45 Ul “The erythrocyte mean corpuscular volume (MCV) (choice Dis frequently elevated and can be used as a marker of alcoholic hepatitis because it gradually returns to normal with drinking cessation, Platelet count (choice F) is often decreased a ether 8 direct toxic est or secondary to hypersplenismn. ‘Thecorrect answer is The clinical manifestations and family history are consistent with Huntington disease, whichis due to an autosomal dominant mutation of tgne on chromosome 4. The mutation consists of an ‘unstable expansion of a CAG trinucleotide repeat in gene encoding « novel protein named Huntingtin. The tage of clinical onset is commonly betwoon 30 and $0 years, but may be as early as 5 years. Behavioral abnor: malities often precede the characteristic choreiform ‘movernents. The patient may experience ieitabilty, restlessness, and dificult in concentration. Dementia subsequently develops CCreutafeldt-Jacols disease (choice A) manifests with a ‘rapidly progressive dementia and mental status changes associated with myoclonic movements. A family histo ‘yi usually absent Gilles de l Tourette syndrome (choice B) hasan erly 180 mg/dL in problem drinkers, ‘who also exhibit high levels of uric acid (choice F) in the blood (>7 mg/dL). Neither test is useful in screen ing for problem drinking because of low sensitivity. “The correct answer ie C. NSAIDs inhibit the produc- tion of prostaglandins, which are eseential for protect ing the stomach and duodenum fom ulers, In patients ‘who absolutely require NSAIDs, misoprostol adminis: tration wil decrease the incidence of peptic ulcer dis ‘ease (PUD) and upper gastrointestinal bleeding. In patients witha history ofthese conditions other agents should be used for pain control if posible ‘Cimetidine (choice A) is an H2 blocker that has not proven beneficial fr treating NSAID- induced ulcers iiedical Section Two: Obook Praice Tests 186 | intemal Medicine Test Five Clarithromycin (choice B) is an antibiotic active ‘gaint Helicobacter pylori, known pathogen in PUD. “The drug has not been proven beneficial in NSAID- induced ulcer. Omeprazole (choice D) is a proton-pump blocker. Is nat appropriate for tenting NSAID-Anduced ulcers Sucrllate (choice B) isa viscous liquid that coats weet beds It has not been proven beneficial in NSAID. inked ulers 23, The correct answer i B. Erythromycin i the treatment ‘of choice for Mycoplasma pnewmonia. Bullous myring tis, hich occur in a smal percentage of patients with “Mycoplasma infection, is characterized by the presence ‘of erythematous and painfal papules on the surface of the tympanic membranes, This patient aio hae cold agglutinins (classically asocated with Mycoplasma), ‘which precipitate on cooling and car with warming. The IgM anthodies made against Mycoplasma spat nate RBCs at low temperatures, causing hemolysis in Tes than 199 of eases. This patent has classic signs and symptoms of Mycoplasma: he is young and has headaches, nonproductive cough, low-grade fever, and Infrae or atelectasis on chest x73. 2h, Thecorrect answers Chis is Menire disease 2 disease ‘of poorly understood pathophysiology in which a gener sized dilation ofthe membranous labyrinth ofthe inner ‘ear (endolymphatic yops) sasociated with tacks of ‘vertigo, tinnitus, and intially Suctuating and later pro agresve hearing loss. The dina description lustated inthe question tem is typical, Meniere disease can act any age or sex, witha (broad) peak inthe dt and Sth decades of Bie. Treatment is pharmacologic and often ‘requires some experimentation before medications (eg, anticholinergcs, antihistamines, barbiturates, and ‘iazepa) effete ina particular individual are found. Benign paroxysmal positional vertigo (choice A) Is characteriod by violent vertigo induced by moving the head to certain positions. Herpes zoster oticus (choice B) has prominent pain symptoms in addition to hearing low, vertigo, and sometimes paralysis ofthe facial nerve. Purulent labyrintits (choice D) ia bacterial infection of the inner ear and occurs aka complication of acute tts media or purufent meningitis Vestibular neuronitis (choice E) presents with an ini til persistent, severe episode of vertigo that eventually fades toa paroxysmal form, which usually completely appears within a yest or two, Ps 25. ‘The correct answer is B. Sjogren syndrome is an autoimmune disorder resulting in dysfunction of exocrine glands, which lads to deynest ofthe eyes and mouth, dental caries, dysphagia, and parotid enlarge- ‘ment because of infiltration of these structures with Iymphocytes and plasma ces. Dyspareunia may deve- ‘py asin this case. There i an association with systemic lupus erythematosus, cheumatoid arthritis, sceroder: sma, polymyositis, and autoimmune thyroid disease. Treatment is supportive, with artificial tears and dental hygiene. The disease i usually benign, but lymphoma may develop. [CREST syndrome (choice A) difers from scleroderma (systemic sclerosis) in that there isa lower tsk of renal involvement, a higher rik of pulmonary hypertension, and an overall beter prognosis. CREST patients have thickened skin only on their hands and fics, as opposed to the more difuse skin involvement in slerodeema Both diseases are characterized by an immune- mediated Gibrosis of internal organs and skin. Dermatomyositis (chole B) is a systemic disease of unknown etology characterized by proximal muscle ‘weakness and a “heliotrope” rash, It is asociated with underlying malignancy. Raynaud phenomenon (choice C) usually affects younger women. The peripheral vasculature exhibits an lbnormal rerponse to the cold. Pstients experience cyanosis inthe fingertips in response to cold or to emo: tions. Treatment includes Keeping the patents hands warm with ploves, baving the patent rfain from smoking, and, in extreme cases, prescribing calcium channel blockers Scleroderma (choice D), or systemic sclerosis, causes hardening and contsction of the connective issue. The skin is tough and thik and bas patches of pigmentation, 26, The correct answer is B The patent ha reflux exophagh- tis that has become complicated by metaplasia of the Squamous mucosa to intestnal-type mucosa (as indie ted by the goblet els). This change, called Baretesoph gus or Barret moss, considered premalignant since ‘Adenocarcinoma ofthe esophagus can arisen these ares. Recommended treatment of gastroesophageal refx dis ‘ease includes elevation of the ead ofthe bed avoidance of acid stimulators, such as coffe and alohok avoidance ‘oF agen that decrease lower esophageal sphieser pres sure, such as anticholinergic fats and cocoate; use of nai after meas and at bedime: and use of 2 block- rs, prokinetic agents or proton-pump inhibitors, such as ‘omeprazole, Once Barret esophagus has been demon- strated endoscopic surveillance to detect developing ade- nacarcinomais recommended every 1 oF 2 yrs ». Achalsia (choice A) would show aperistalsis and rreased lower esophageal sphincter pressure on ‘manometry. Corrosive esophagitis choice C) is seen after accidental of suicidal ingestion of caustic poisons, such a strong leaning solutions Psophageal adenocarcinoma (choice D) is a major complication of Baret esophagus, but itis not indicat ‘edhe by the biopsy since the cells seen are regular and ‘nonmalignant Zenker diverticulum (choice E) is a posterior out pouching of the esophageal mucosa and subimucoss through the cricopharyngeal muscle ‘The correct answer is A. The clinical presentation is consistent with hypertrophic cardiomyopathy, fe ‘query inherited disorder due to mutations in one of the genes ensoding mofibrillary proteins. Lefe ventric ular dysfunction is due to impaired diastolic filing. The septum is dsproportonstely thick compared with the feee wall (asymmetric hypertrophy), resulting in out flow obstruction. The latter is the cause of the systolic ‘murmur, which intensifies with deceased left ventrcu- lar filing, such as during the Valsalva maneuver. Beta blockers or cakiam channel blockers shouldbe the in til ereatment Digitalis (chole B) is contraindicated in hyperiophic cardiomyopathy because it enhances myocardial con tractlity and, consequently, the degree of outflow ‘obstruction secondary to increase septal vonttaction Longacting nitrates (chotee ©) are used for treatment of angina and related coroaary artery syndromes, but ‘ot for chest pain secondary to hypertrophic cardiomy- ‘pathy. The late, in fact, isnot due to vascular steno but to a combination of deceased loft ventricular ‘output and increased oxygen demands of the hyper teophie myocardium. Aortic valve replacement (choice D) is indicated for treatment of aortic stenosis, but the aortic valve isnot ‘alin ths condition, Outflow obsiraction is the result ‘of a thick septum that partially obstructs che outflow Partial excision of the myocardial septum (choice E) has heen succesful in some centers but should be rescrved for severe cases in which pharmacologic ther apy has aed, “The correct answer sD. This is central pontine myel lysis, which is a feared complication of severe hyponatremia. This demyelinating condition of the pons often produces permanent damage, which may Imunifest as istated in the question stem, More », 30. Section Two: Qhock Practice Tess ‘Answers and Explanations | '®7 severe cases can even cause a locked-in syndrome, in hich the patient isn an avake and sentient state but has complete generalized motor paralysis, leaving only perhaps a limited ability to communicate by coded eye movements. (Fiction aficionados may recall that the ‘character Noier in The Count of Monte Crist had this syndrome. He could communicate only with his grand daughter, who was the only person patient enough to alto him and interpret his eye blinks.) Controversial evidence suggests that overly rapid correction of severe hyponatremia increases the chances thatthe complica- tion will develop, posibly because of fluid shifts in the confined azea ofthe pons. Ii now recommended that plasma sodiam be raised no faster that 1 mEq/L, ‘with an uppes limit of 10 mEq/L24 he. Although demyelination can also involve other areas of the brain to much lesser degrees, the cerebellum (choice A), cerebral cortex (choice B), peripheral nerves {ehoiee ©, and spinal cord (choice E are not usually significantly damaged as the pons inthis condition The correct answer is D. The most common cause of asymptomatic hypercalcemia is primary hyperpacathy roidism, In older women, 85% of cases ate due to ade ‘noma ofa single gland, 15% are caused by hyperplasia. ofall our parathyroid glands, and about 1% are asoci- ated with carcinoma, Most patients have asymptomatic Iaypercalcemia that i found incidentally during a row ‘ine laboratory examination Excess vitamin D intake (chotee A) is an uncommon cause of hypercalcemia. (Occult malignancies (choice B) are responsible for most, ‘eases of “in hospital” hypercalcemia, Udelying mecha isms inclide release ofa PTHfike substance and local ‘oxteoiti hypeealemi Paget disease (choice C) is associated with an increase in alkaline phosphatase and normal calcium and phos phate Sarcoidosis (choice B is asocited with hypercalcemia ‘due to increased production of 1.25-(OH), vitamin D, by alveolar macrophages, but itis less common, This [Phenomenon also occurs in other chronic granuloma: tous disorers, in ymphonsas, and in idigpathie hyper calciuria, ‘The correct answeris A. The patient has eythema mul- tiforme, which an indammatory eruption characterized by lesions showing a variety of morphologies that may involve both skin and macous membranes. Erythema multiforme can occur idiopathic as a comptiation of viral infections (notably herpes sales viruses, but also casacievrus and echovirus, Mycoplasma prenmoniae 31 Section Two: Qbook Pract Tests Internal Medicine Test Five infection, or fangal infections (histoplasmosis); or as reaction to. drug (pencils, sulfonamides, barbiturate) ‘or vaccine (BCG, vaccinia, poliomyelitis). Erythema mat ‘forme tends ta have a siden onset, with a predilection for most severe involvement of the hands (notably palm), fet (notably soles, and face (notably near oF favolving mucous membranes) The most hepfl sion morphology isthe target lesion, with an erythematous ting, mid pllos, and central erythematous macules it ‘may, in practice, he necessary to look at a number of Jesions before a target Ison is identified. Attacks of ery thema multiforme tend to last 2-4 weeks, treatment usualy primarily directed against any undesiving cond tion or drug that triggered the skin reaction, Erythema nodosum (choice B) usualy causes tender red nodules of the prebial areas of the legs, Granuloma annulare (choice C) causes a peripheral tng of nodules around normal to slightly depressed sin, Pemphigus (choice D) causes acid Blisters of skin and mucous membranes. “Toxic epidermal necrolysis (choice B) causes shoes of skin to pel off, analogous tothe peling skin of sun burn, “The correctansweris A. The mos crucial step in begin ning investigations on jaundiced patent is to deter- rine whether dhe jaundige is due t hemolytic disease, hepatocellular damage, or biliary obstruction. ‘Generily, jaundice resulting from hemolysis or hepato: ‘olalar damage must be treated with medical measures, whereas obstructive jaundice requiees surgical teat- ‘ment. Obnructvejaundioe is characterized by a high proportion of dire (Conjugate) bilirubin, elevated serum alkaline phosphatase levels, and normal or mild- Iy elevated aminotransferases. This cas is therefore de to billary obstruction, How to proceed? The most appropriate investigation is ultrasound examination to tluate the hepatobiliary system and pancreas, dete- rine whether extrahepatic bile dots ae dilated, estab Tish the presence of gallstones, and identify hepatic or pancreatic masses. CCT or MRI seans (choice B) are also adequate but aze ire expensive thus, ultrasound constitutes the most convenient intl diagnostic test Endoscapic retrograde cholangiopancreatography (ERCP; choice © isthe method of choice o study pan creatc and smpellar lesions, carryout stone extraction fom the common bile duc, or insert a stent. Percutaneous liver biopsy (choice D) isthe definitive study for hepatocellola o in lrative diseases. tis not médical 22 used ae an initial diagnostic procedure because ofits invasiveness and potential adverse eflects. Percutaneous transhepatic cholangiography (choice E) ‘can identify the location of biliary obstruction, but may be atsociated with serious complications, such as bac {crema bile peritonitis, and hemorrhage. At any rat this procedure should fllow other noninvasive radio logic methods ‘The correct answer is C From a primary care perspec: tiv, the great majority of cases of low back pain are due to degenerative joint disease, However the primary care physician should be alert to “red flags that may signal the presence of mote serious pathology. A history of corticosteroid teatment may be asocated with osteo porosis of the vertebral column, which predisposes patients to compression fracture ofthe vertebral bod ies. This type of fracture may present with back pain ‘without any apparent history of preceding trauma Ankylosing spondylitis (choice A) is characterized by ‘onset before 0 years of age, progressive ankylosis of the vertebral columa, and bjateral involvement ofthe sacroiliac joints. More than 90% of cases ae associated with HLA-B27 histocompatblty antigen. This disease should be suspected in a young person complaining of (0 retold increase in AST or ALT. Tamoxifen therapy of breast cancer (choice A) is an ‘unusual cause of fty liver. CChronie pancreatitis (cholee B) does not cause fatty liver although both can coexist inaleobolics. Peptic ulcer disease (choice D) does not cause fatty liver Glucococticoid therapy in systemic Lupus erythemato- sas (choice E i an unusual cause of far iver 38. 36. Scion Two: Qbook Practice ests ‘Answers and Explanations | "89 “The correct answer is ET fst sep in solving this _quetion eo recognize thatthe patent has severe peptic ler disease inthe sting of markedly increased basal acid secretion by the stomsich and markedly elevated lev- dls of serum gastrin. This suggests Zllinger-Flison ym drome, in which an endocrine tumor secretes gastrin All Zollinger-Eison patients have serum gastrin levels treater than 150 pg/mL. Markey lvated lees (greater than 1000 pg/mL) inthe seting of compatible clinical features and gastric acd bypersecetion (greater than 60% of the amount of acid alter a maximal stimulating dose of histamine) is considered dixgnost. In this case, the tamor comes fom a pancreatic endocrine tumor; other stes can include the dvodenum, splenic hilum, mesentery stomach, oF even lymph nodes or ovary. More than 5006 of gastrinomas ate malignant. About hal the patients with Zellinger-lison syndrome have mult tumors, wsualy as pat of multiple endocrine neoplasia type (MEN I or Wermer syndrome). In MEN ,pancre- ati it cell tumors are found in 30 to 75% of patients parathyroid adenomas in more than 90%; and pituitary denomas in 50 to 65%, MEN I patients may also rarely have duodenal gastinomas MEN IIA, or Sipple syndrome, is characterized by medullary carcinoma of the thyroid (choice B; more than 90% of affected patients, pheochromocytomas (choice D;50%),and parathyroid adenomas (25%). MEN IIB, or macosal neuroma syndrome is character- ized by macosal neuromas(ehoiee C; close to 100% of affected patents), Marfanoid habitus (choice ; close to 100%), medullary carcinoma ofthe thyroid (choice 'B; more than 90%), and rarely parathyroid adenomas. ‘The correct answer is A. You should be aware that ketoacidosis can be seen in conditions other than dia- betes mellitus. The #0 most important are starvation and chronic alcoholism, The mechanism of ketone pro Action in chronic alcoholics appears to bea combina tion of alcohol withdrawal and starvation with increased free faty acid release. The typical history (Goshen it can be obtained) i that of a alcoholic binge that ended with vomiting, after which the patient con- sumed neither food nor water for 24 outs oF mo Abdominal pain is usually @ prominent complaint. Many of thee patients have pancreatitis, and impaired iucose tolerance or even mild type 2 diabetes mellitas tne often demonstrated after the patients recover ftom their acute episode Congestive hear failure (choice B) emphysema (ch ©), and theamatoid arthritis (choice B) do not cause nondiabetic ketoacidosis, ”. Section To: Qhaok Pracice Tess Intemal Medicine Test Five Although severe inflammatory bowel disease (choice D) ‘ould in theoey cause enough “staration” t induce ketoacidosis in practice shis isnot usually seen “The correct answer is E. This patient has euwolemic Iyponatremia, which mesns that her effective circa {ng wolume ie normal. Her euwolemic stat ie support «by her normal gular venous pressure (TVP) of 7 em ater an indication of her intravascular satus. Patients Jn congestive heart alore would have an elevated IVP Furthermore, er lungs are cleat ad she has no le tclema, Euvolemic hyponatremia can be caused by @ _shucocorticnd defect, hypothyroidism, or inappropriate ‘cretion of arginine vasopresin (antdiaretic hor- mone; ADH), SIADH can be caused by many things, including tau, infections, medications and certain neoplasms (ectopic production) ncuding lng canoer Excess ADH secteion leads to retention of free water, producing, hyponatremia. The frst step in managing this would be to restrict the intake of water, producing a rgative water balance that results in gradual, daily rection in weight and a progressive rise in serum sodium. Giving diuretics and water (choice A) is therapeutic ‘option for trating hypernatremis. The diuretic would lead to the removal of excess sal, which would not be desirable ‘Administering isotonic saline (cholce B) isthe equiva- lent of giving more free water, which wil be retained tnd the ssit will be excreted, ths worsening the hyponatremia ‘Administering water (choice ©) will worsen the hyponetremia since the patent is simply being given fee water. This treatment would be appropriate in tweating hypernatremia. “This patient is hyponatromic since her effective salt load is lve. Restricting the sal intake would th not Inlp (choice D). Infact, allowing her to take salt is also not likely to be effective in controling the bypenstrem iasince ADH leads to excess retention of free water at the cost of sodium excretion, ‘ThecorrectanswerisD. The radial nerve originates fom. Gy, and innervate the triceps, brachioradiais, wrists, and finger and thumb extensors. The lesion sully affects the spiral groove ofthe amerus. Clinical features include weist drop and inability to extend the elbow and fingers because of severe denervation ofthe dorsolateral hand. Radial nerve palyy is sometimes known 38 “Saturday night pls” since es found in patients who hve boen drinking and fll aeep with an arm banging lover a chai. Patents on crutches may abo pinch the nerve with their rurches, producing 2 similar palsy. The axillary nerve, CyG, (choice A), innervates the 20) and elevat- 4 blood glucose. The first step in the care ofall DKA patents is prompt restoration of their volume status. "This the priority as the ongoing diuresis from theele- ‘ated glacose wil only worsen ther acidosis Administration of broad coverage antibiotic therapy (choice A) would be appropriate if the patient has an tindeeiying infection, However, this intervention has no place in the acute management of DKA. ‘Administation of IV glucose and insulin (choice B) is ‘propriate once the blood glucose fils below 250 mg/d. fis important at all times to have inulin on board fr diabetic patents, Once the glucose falls below 250 magi. during therapy, glucose must be given with the insulin to prevent hypoglycemia and asist with clearance ofthe ketone bodies. Hypertonic saline should not be administered (choice (©), since this patient's corrected sodium is 133 mg/L. (any extra glucose in the sample of blood used to calc late scrum clctolytes will decease the measured serum sodium by 2.6 mBq/L per 100 mg/dl of glucose), which is acceptable, M n Scion Two: Qbook Practie Tes ‘Answers and Explanations | 2" Administration of IV potassium (choice D) will be esd as the patients acidosis begins to correct and the serum potassium begins co decline. In DKA patients, total body potassium beoomes depleted 25 a result of diuresis, “he correct answer is C. The features Hustrated are typical of acromegaly which is due to excess growth hormone produced y's pituitary adenoma. Othe ‘cal features that may be sce include thickened, some times darkly pigmented skin, a barrel chest, tongue enlargement, and an increase in hat sz. Joint symp- toms, which may include « crippling degenerative arthritis, re frequent. ther problems include periph- ‘ral neuropathies de to nerve compression, headaches, vial chang (elated tothe pituitary rumor), cardiac disease, hypertension, and inereased cancer risk. The ‘agnosis is usually made clinically and hen substan ated with skull x-rays (showing cortical thickening and enlargement of the ela turcica) and plasms growth hormone levee ‘Adrenal tumors (choice A) can cause hypo- or hyper- secretion of mineralocorticids, glucocorticoids, and pinephrinenorepinepheine. Parathyroid tumors (choice B) can alter calcium metabolism, Testicular tumors (choice D) can secrete androgens, ‘estrogens, or other steroid hormones. Thyroid tumors (chole E) can cause hypothyroidism; they rarely cause hyperthyroidism or disturbances in cal- jum metabolism (medullary carcinoma). ‘The correct answer's A. There isclea linia evidence of corticospinal tract disease. Inflammation and ‘destruction ofthe alantaxil joints isa crucial clinical iagnosis to make in the management of patients with severe rheumatoid artis. Leal pain may be minimal, tnd the progression may be so slow that dysfunction tnd disability are atuibuted to pexipheral acthitis Fpisodes of deficit progression may occur and may be precipitated by unusual neck movements. Any kind of ‘manipulations absolutely contraindicated, Management requires urgent neuroradiologic (MRI) and neurosur gical evaluations. Though cervical spondylotic myelopathy (choice B) is statistically the most eommon cause of cervical pinal ‘ord compression in an ad, onsets typically after the lage of 50, The clinical features may’ be inistinguish- able, but radiologic examination is disgnostc. The sulantosxal articulation is nota site for degenerative arthritis. a4 Section Ta: ObookPrctie Tests Internal Medicine Test Six “The patient is at rsk for severe osteoporosis because of long-term nse of prednisone. Osteoporotic spinal frac tures (choice C) are astocited with back pain and loss ‘of vertebral height and predominantly involve the tho- raci spine. Cervical level compression cannot occu in this situation “There is no convincing clinical evidence of peripheral neuropathy (choice D) mild glove-and-stocking en sory neuropathy is relatively common in rheumatoid arthritis however, this neuropathy is usually benign ‘and doesnot imply inflammation ofthe nerves, Patchy distinct sensory Toss may also be due to spinal cord compression, carpalitersal tunnel syndromes, oF ‘mononeurts multiplex. Reflexes are usually hypoae- tive or absent, rather than hyperactive. Epidural fat (choice B) especialy at the thoracic level _may increase sufficiently during chronic use of gluco- corticoids to resalt in spinal cord compression. Tt ‘Should aways be considered if bony compression snot docunnented and spinal cord symptoms remain unex plained. MRI is the diagnostic modality of choice. es ‘rate condition, ‘The correct answer is D. Cough productive of foul smeling sputum should immediately suggest a pul ‘monary infection involving anaerobic bacteria. Thi ‘often associated with poor dental hygiene and/or co: ditions favoring aspiration, such as neurologic disor ders, depressed level of consciousness, oF tracheal/nasogastric tubes. Prevorlla melaninogenica, Fusobacterium nucleatum, vod anaerobic streptococci are among the most common pathogens In this case, the patient has developed a pulmonary absoess, asthe sry finding ofa “eavty wit air-fuid level” stongly suggests cchrichia cli (chee A) isa common cause of rose ‘nial poeummonia, defined as pneumonia occurring in hospitaied patients mote than 48 hous after admission Kleela prewmonine (choice B) is prtcuaty asc ed with conmmuniy-acqured pneumonia in patients with tleohol abuse or diabetes mellitus, Ie may also cate ‘nosocomial (hosptal-aequred) preumoni. Lagionella prewnophila (choice ©), one of the most common causes of community acquired pneumonia, prefeentnlly afees immunocompromised. patients, Incayy smokers and patients with chronie obstructive pplmonary disease (COPD). Outbreaks of legionellosis result from exposure o contaminated sources, such as air conditioning towers or showerheads Pseudomonas aruginasa (choice E) is frequent isolt- cd in cases of nosocomial pneumonia, as well as in ‘pneumonia occuring in eystic fibrosis patients, 1s, Staphylococcus aureus (choice F) is one of the pathogens mos frequently asocated with nosocomial peumonia Sirepoconus preumonize (choice G) isthe most com- mon ‘cause of ‘community acquired pncumonia. Preumococal pneumonia typically causes lobar cons idaton and frequently follows an upper respiratory rat infection, ‘The correct answer is C. All medications are dared cither bythe liver, the kidney, or the lang The majority are cleared by one ofthe first two routes, whereas inal ional anestheis are largely cleaved by the tied. Many patients especialy inpatients on current day medical and surgcl wards, have impaired renal function liver fare tion, or both and very earful consideration to dosing adjustments must be made. In ficty most house-saff mamas have lst of drugs that cominoaly have altered 4 mim in sie, and be on any aspect ofthe body. ‘Squatmous cal eacinoma (choice E isthe econd most ‘common form of skin cance. They commonly occur ‘on the lower lip ‘The correct answer is E. This patent has the signs and symptoms of obstructive sleep apnes. This condition is characterized by poor nighttime sleeping resulting in daytime somnolence and the psychological conse ‘quences of prolonged sleep deprivation, Patients tend to be erable, have dficalty concentrating on daytime tasks and have an inappropriate lack of energy given their activity level. Although only 50% of obstructive sleep apnes patients are obese, obese patents tend 19 have an increased amount of redundant nasal and Seaton Two: Gbock Pract Tests ‘Answers and Explanations | 725 nasopharyngeal tissue, which makes their upper airway more likely to obstruct in the supine postion. Diagnosis of obstructive sleep apnea is initially made by the appropriate history and symptoms and is con firmed by polysomnography, ot sleep study, which doc: tumentsapnee or hypopneain addition to oxygen desat- urstons during these obstructive episodes. Empiric tril of weight loss and exercise (choice A) is pat of the treatment plan in patients who have ‘obstcuctive sleep apnea, but it does not confirm the ‘diagnosis, Alo, anatomic abnormalities independent of the increase in soft tissues associated with obesity may be the source of the problem, which would not resolve with weight loss oF exercise, Laboratory tests for hypothyroidism (choice B) are not indicated a this time, given the lack of other symptoms consistent with a diagnosis of hypothyroidism, Mask continuous postive airway pressure (CPAP) (choice C) should not be administered unless the patient has been evaluated clinically for the presence of Obstructive sleep apnea, cither with polysomoography for continous oxygen saturation measurements. In sddition, CPAP administration will not be funded ‘without documented evidence of seep apres [An MRI ofthe head and neck for sof tissues (choice D) is not indicated and may not provide any information regarding the pathology of the insomnia. In addition, obstructive sleep apnea is dynamic proces that would ‘ed ta he viewed radiologically in realtime. The pres- nce of redundant soft issues in the nasopharynx and ‘oropharynx does not necessarily confirm the diagnosis of obstructive sleep apnea. ‘The correct answer is G. In thi as, oliguria and other signs of acute real failure may result fom either sus- tained hypotension (econdary to Bud los) oF ate tubular necrosis. ‘The later may result. fom bbypexicfischemic damage tothe tubular cells inthe st ting of shock or sepsis. Thus, prerenal azotemia is dv to decreased renal perfusion, whereas ena azotemia is due te intisic damage to ronal tubules. The treatment difrs ‘depending on the underlying cause The pararsetes that allow the differential dignosis berween prerenal ad ‘eal azotemia are principally the fractional exretion of sodium (FRq) and the cato between BUN and serum creatinine. Real failure due to hypovolemic shock, asin this case, wl manifest with prerenal azotemia, associated with FR, 20. Usually, urine is [hyperosmolar compared with plasma. Cardiogenic shock (choice ©) may also give rise to acute renal failure becuse of preenal azotemia (hypoperfusion). In this ‘ase however the mos likely mechani is hypovolemic shock secondary to los of fui, a4 "7. Scion Two: Qbook Practice Tess Internal Medicine Test Six “The correct answeris L. The dnc pictares consistent swite acute urinary retention secondary to prostatic Iyperplasia, which is an example of posreal azote. Acute onset of anuria is usually accompanied by signs of acute renal fale, soch as mauten and vomiting, malaise, and obtunded sensorium. This acute elnial pice is ‘usualy precede by a long history of urinary symptoms duc to. prostatic hyperplasia, namely progressively Increasing hesitancy, decreased force of stream, and postioid dribbling Use of drugs with anticholinergic properties, such as tricyelic antidepressants, may precip> Hate acute urinary retention, The reent history of inta- venous pyelography (IVP) should not mislead you. Radiologic contrast media may act as direct nephrotox- ins, However, acute renal failure secondary to radio: srephic contest toxicity (choice M) develops within 4 hours after administration of IV radiocontrast. Furthermore, iis obvious that amura in this case is due to obstrction end not to intrinsic renal damage. Aminoglycoside toxicity (choice A) i a common cause ‘of acute renal failure de to acute tubular necrosis, in Which case FR is >1% since damaged tubular cells are tunable to reabsorb sodium. In addition, the BUN:Ce ratio ¢<20,and urine osmolality approaches that of the plasma (250-300 mOsm/kg) Analgesic toxicity (choice B) is due to chronic ingestion of large amounts of such agents as NSAIDs, apiin, and paracetamol. t may result i papillary necrosis and pro ressive renal interstitial damage. Such damage will even tually lead to chronic interstitial nephritis, manifesting swith chronic renal ale Polyura due to inability ofthe Kidneys to concentrate urine sa typical erly sign Goodpasture syndrome (choice D) i 2 chronic ifm: ‘matory disorder afacting the lungs and Kidneys. I is mediated by antibodies aguns the collagen of the base ‘ment membranes ofthe lung nd glomeruli. Hemoptsis And nephritic emndrome often progressing ores failure axeitslinical manifestations. Henech-Schonlein purpura (choice B) isa disase of huldreo. Purpura, hematuria, abdominal pain, melena, and arthralgas constitute its clinical picture, and TeA- mediated vasculitisits pathologic substrate yperuricemi (choice F) may caus acute renal failure, wich develops in patients sith rapid. cell tumover, namely patients with leukemia or lymphoma who are ‘undergoing chemotherapy. ‘Malignant hypertension (eholee H) may ceuse hema ‘ura, proteinuria, and loss of renal function because of arteriolar damage. Blood pressure i very high, usually 5220/120 nen Hg, 9. Multiple mysloma (choice 1) i a plasma cll neoplasia associated with production of a monoclonal immunoglobulin. Fragments of ths immunoglobulin (sual dimers of light csi) aze filtered through the tlomeralus and may precipitate in the tubules. This wll Fesult in proteinuria, hypertension, and progressive renal failure [Myoglobinaria (choice 1) may produce acute tubular necrosis in the setting of extensive crush injuries that ‘cause necrosis of skeletal muscle Postinfectious glomerulonephrits (choice K) usually follows pharyngitis or impetigo due to. group A Streptococcus and manifests with typical nephritc sy ‘drome, Rarely this condition may manifest with rapidly progresive glomeralonephrits (ie. crescenic type), leading to acute rena flue. Sickle cell disease (choice N) is one ofthe causes of renal papillary necrosis, ter cases include analgesic toxicity, obstructive uropathy with recurrent urinary ‘wact infections and diabetes. Unilateral ureteral stone (choice O) would manifest with acute onset of colicky flank pan, associated with spross or microscopic hematuria. Acute renal lure toes not ensue inthis situation, unles the conttalateral kidney is already impaired. ‘The correct answer is G. These clinical manifestations, especially the characteristic ofthe sol muro, are consistent with aortic stenosis, which soften associated with anginal pain. The patent also shows signs of impeding eft ventricular failure, namely exertional dys- paca and left ventricular hypertrophy. Aortic stenosis ‘anifestng in younger individuals (<50 yeas) i aslly ue to-a congenitally abnormal aortic valve. In elderly palients,eacifcation of aortic valve caps isthe most ‘The correct answer Is U The clinica history and objec tive findings are highly characteristic of spontaneous (primary) pneumothorax, which often affects tall thin men between 20 and 40 yeatsof age. Chest pain and respiratory distcess begin suddenly, usually at rest and ‘often during sleep Patient often sek medical attention days after the onset of symptoms. If the pneumothorax is large diminished breath sounds, reduced tactile fremitus and hyperresonance are present. The condi- tion is thought to arise from spontaneous ruprure of subpleural alle in otherwise normal lungs. “The correct answer is S. Despite the vague familia history of heart problems the clinical symptomatology is lacie for gastroesophageal reflux or veux esophagitis ‘The onset of pain in the recumbent position and soon L after meals, along with the reli provided by antacids or till virtually diagnostic Upper endoscopy tthe diag nostic procedure of choice to establish a definitive diagnosis and to obtain biopsy specimens of the sophageal mucoss. Acatecholeeystiis(choiceA) manifests with abdominal pin in the right upper quadrant radiating to the back and right shoulder fever, and leukocytosis. Occasionally, ‘itmay present with precordial pain. Acute pericarditis (choice B) is associated with precordial pain, fever, and evidence of pericardial effusion, eg. pericardial rub. f pericardial effesion i abundant, car- diac tamponade may ensue Acute pleuitis (choice C) i characterized by latralized chest pain that intensifies with deep breathing. (Objective signs of pleural effusion, as well a pewritie rub, ae usually present Angina pectoris (choice D), in its mest common form, ‘manifests with precordial pain that has a crashing oF squeezing quality. The pain is triggered by exercise oF ‘envotional stress and relieved by et or vasodilators Aortic section (choice ) is an emergency manifesting, ‘with excruciating chest pain radiating to the back, pulse and presure discrepancies, onset of aortic regurgitation, and shock A history of hypertension is often present Aortic regurgitation (choice Fs associated with dias tolie murmur and a wide diferential hetween systolic and diastolic pressure, Costochondrits (choice H), also known as Tietze syn- drome, is die to inflammation of the choadrocostal junctions, which are swollen and sore sophages! carcinoma (choice 1) may produce chest pain but usually manifests with progressive dysphagia fnorera, and weight los, Alcohol and smoking habits arc often present inthe history. Bsophagel spasm (choice J) is 2 rare cause of chest pain, but it usually associated with swallowing diff clic. An xray afer barium may show evidence of spasm, Herpes zoster (choice K) is characterize by a vesicular ruption along dermatome, which may be followed by ‘chronic burning pin, especially in elderly patients. Mita valve prolapse (choice L) is de to myzomatous involve the ‘renal gland despite the sll sie ofthis organ ‘Tuberculosis (choice F) was once the most common cause of adrenal insufficiency, and itll the most com ‘mon cause in areas in which tuberculosis is endemic. "The correct answer is A. Constritve pericarditis isthe fuse thickening of the pericardium in resction to prior inflammation, which results in reduced dstensi- Diy of the cardiac chambers, Cardia output is limited, a wo. Section Two: Qhook Practice Tess Internal Medicine Test Seven and filling pressures are increased to match the external ‘anstriive fore plaed onthe heart bythe pericardium, Inboth constrictive pericarditis and cardiac tamponade, the distlic pressures are equal in all four chambers of the hear. The jugular veins are distended, indicating, systemic venous hypertension. Tis neck vein disten- ‘on increases with inspiration and is called Kusmauls sign ‘The first and second heart sounds (SI and $2) are reduced in intensity because of reduced sound trans- ‘mission though the thickened pericardium (choice B). Patients with congestive heart flue have an extra, ‘third sound (choice C). This occurs during rapid filing of the left ventricle, The fourth heart sound (choice D is heard in patients in sinus rhythm and with heart failure. in elderly patients, it may indicate reduced compliance ofthe stiff ventricle In most cases of constrictive pericarditis, the elnieal findings of right-sided failure are more prominent than those of left-sided failure (choice F). Thus ascites, jun dice, and edema will be commonly seen ‘The correct answer iD. An isolated thyroid nodule is a frequent finding in asymptomatic adults, and most of such nodules are beniga, Fine needle aspiration allows 2 diagnosis in most cases. The material aspirated with needle is smeared on aside and stsned. In only 1596 of ‘eases is the aspirated material “non-diagnostic” Suspicious cases are followed with repeated fine needle aspiration, Malignant nodules are ually large (>3 em) andlor fixed to the surrounding parenchyma, Papillary Carcinoma it the most common malignant thyroid neoplasia, Ultrasonography may ako be of value in distinguishing solid from eystc nodules and is preferred to MERI sean (choice A) or CT scan (choice B) because ofits high Sensitivity and lower cost However, CT and MRI are valuable in defining the extent of malignant tumor, ‘once the diagnosis is made Radioactive iodine scan (choice ©) is needed when a tolitery thyroid nodule is escociated with symptoms of Uyrotoicois. Radioactive iodine scan heips to dstin- uish a toxic adenoraa from Graves dicate, in which high uptake is sen in the whole gland. Excision of a thyroid nodule (chotceB) is performed if it proves tobe malignant, oF in case of «hot i, hyper functioning) nodule causing thyroroxcosis ‘The correct answer is C. IV drug users are prone to developing bacteremia, which can lead toa brain abscess Patients with brain abscesses are typically afbrile and can exhibit progressive neurologic dys function, 1V drugabusers are prone to developing bacterial endo carditis (choice A), but they are typically febrile. This patients echocardiogram, and physical examination les aveay from the diagnosis. However, inthe manage ‘mest ofthis patient, blood cultures should be deawn, and suspicion of endocarditis must be high. Patents with bacterial meningitis (choice B) are typi cally toxic appearing and febrile, and have positive signs of meningeal ivtation. Thay ago often exhibit nuchal rigidity. Elevated white count in the cere- Drospinal ud, decreased gloose lve, clevated protein, and a predominance of neutrophils are indicative of this diagnosis. HIV meningitis causes a headache and meningeal i tation. Focal neurologic deficits donot occur Ceyprococeal meningitis (choice D) typically presents wth altered behavior and a headache. However, stroke ke event are rare, IV drug use can lead toa foreign body embolus (choice B) and apoplectc neurologic problems. An embolus ‘may reach the bra va a rightto-let cardiac shunt or pulmonary arteriovenous malformation ifthe injection is venous Embolic phenomens are more common in the setting of endocarditis, “The correct answer is D This woman isin a nonketoti hyperglycemic hyperosmolar coma, a feared complica tion of type 2 diabetes mellitus that i associated with 3 5% mortality rate. The basic problem is that wien extreme hyperglycemia occurs, glucose spills into the ‘rine and can cause profound debdation, nce the ‘ose acts as an osmorie diuretic. Feautes of thie syn drome include CNS alterations, extreme hyperglyvria Atypical valves in the range of 1000 mg), ad dehy ration. These features in urn lead to hypersmilality (the eoerect choice in this case; normal values are les than bout 290 mOsrmolkg), mild metabolic acidesi no etoncmia to minimal hyperketonemi, and prerenl szoremia. Diabetic ketonidoss, which isthe ate dag ‘osis that shoud he considered is uncommon in type 2 dlibetics and ie associated with lower blood ghicose levels than nonketotic byperlycemic hyperosmolar coma. Blood urea nitrogen (choice A) and serum creatinine (choice ©) are usually elevated rather than decreased in nonketotic hyperglycemic hyperosmolar coma ‘Strongly positive ketones in blood (choice B) are a fea ture of diabetic ketoacidosis. Minimally elevated Ketones inthe blood are sometimes see in nonketotic lnypergycemic hyperosmolar coma 2 13 ‘Serum sodium (choice ) is usally normal increased in nonketotic hyperglycemic byperosmelar coma. “The correct answer is E. Chronic constipation is a ‘common problem inthe elderly: however, before ras: suring the patient, the physician has an obligation to ‘exclude serious disease uch aseolon cancer Once the physician is reasonably convinced that there is no ser ‘out underiying pathology, the next steps are to suggest increasing fiber in the det and to discontinue any med ications that may be causing the constipation. If these ‘steps fal, then the addition of bulking agents (bran, pyilium, calcium polycarbophil, or methycellulose) is ‘warranted, Longeterm ue of other types of laxatives is ‘ot recommended, Docusate (choice A) isa weting agent (detergent lx tive) that soften tole by increasing their water content Lactulose (choice B) and magnesium phosphate (Choice C) are osmotic agents sometimes used to pre- pare patients for diagnostic bowel procedures. ‘Mineral oil (choice D) softens fecal matter but is not recommended for long-term use because it may decrease absorption of fat sluble vitamins ‘The correct answer is A This patient has aplastic ane- ‘mia, A characteristic feature ofthis condition i that the growth of erythrocyte precursors, granulocyte precur- Sore, and megakaryocyte precursors is. markedly impaired. The marrow ie usally replaced by adipose tissue. Aplastic anemia typically develops insidious, Fhat may have a more rapid couse. In about haf of ‘cases, no cause is ever identified. In the remainder, ‘causes may inclade chemical exposures (e4, benzene and inorganic arsenic), radiation, or drug reactions (eg. antineoplastic agents, antibiotics, anticonvulsants, And nonsteroidal antiinflammatory drugs), and par vovinae B19 in patents with hemoglobinopathies or spherocytosis, Historically the condition has required marrow transplantation, but this therapy is now reserved for patients who fil o improve with equine antthymocyte globulin or eyelosporine therapy. Tron deficiency anemia (choice B) produces microcytic Folate deficiency anemia (choice ©) and vitamin B,, deficiency anemia (choice B) produce a megaleblastic ‘Myelophthisic anemia (choice D) can clinically resem: ble aplastic anemia, but bone marrow studies would demonstrate tumor, granulomatous disease, or fibrosis replacing the normal marrow. ry 16, Scion Two: Qbook Pratce Tess “Answers and Explanations | 245 ‘The correct answer is E.This question asks you to dif ferentate between diferent types of peripheral motoe ‘neuropathies. Ramsay Hunt syndrome, which i caused by herpes zorter infection ofthe geniculate ganglion, resuls in ficial aly. It difers from the other new ropathie in that there is usually a vesicular eruption, ‘peal of herpes infections. Bel palsy (hotee A), the most common form of facial paralysis, idiopathic. The onact i abrupt, with maximal ‘weakness in the firt 48 hours. Fighly percent of patients fll ecover in afew weeks. Guillsin-Bareé syndrome (choice B) isan acute in lamn- _matory polyradiculoneuropathy. It causes bilateral facial paloy and usualy produces arcllexic motor paralysis. A ‘ira ines often precedes the onset of neuropathy. Horner syndrome (choice C) affects the oculosympe: thetic nerves, usually ipsilateral, specially caus unilateral miosis and ptosis, with normal pupillary response t0 light. Hlemianhidrosis of the face aso Menire disease (choice D) manifests with recurrent ‘vertigo and is associated with tinnitus and progressive ‘deafness. Tere i facial paralysis ‘The correct answer is B. The patient has pityriass roces, and the lege lesion is known as the herald patch” Any description ofthis disease in a question will probably ether use thi term oF describe the inital, larger lesion. The scaly lesions tend to involve the trunk and may be ether oval or crcnate increasing an inital impression of ringworm). The lesions on the back may fallow the lines of cleavage of the skin, producing a “Chustinas tec” appearance ofthe lesion distribution. ‘The condition i selt-limited, but may persist more than 2 months. It's suspected tobe infectious in nature, with potential causative species including a picornavirus herpes virus 7, and Mycoplasma, Molluscum contagiosum (choice A) causes multiple smal papules with umbilcated centers. Pityriasis rubra pilaria (choice C) has predilection for involving the hands and soles and does not produce a herald patch Rosacea (choice D) causes telangiectasia, erythema, ‘papules, and pustules ofthe nose and cheeks Scabies (choice E) would be suggested in a question stem f the physician found small skin burrows in sd tion to papules. Also itching is prominent with scabies “The correct answer is E. This patent has a bilateral loss of pain ad temperature sensation, with preserved pro prioception, below tbe TIO dermatome. This implies ddamage at the spinal, rather than the brainstem, lve

You might also like